LinAlg2/LinAlg2.tex

3842 lines
175 KiB
TeX

\documentclass[12pt, a4paper]{report}
\PassOptionsToPackage{dvipsnames}{xcolor}
\usepackage{tikz}
\usepackage{amsmath}
\usepackage{amsfonts}
\usepackage{enumitem}
\usepackage{amssymb}
\usepackage{marvosym}
\usepackage{mathtools}
\usepackage[colorlinks=true, linkcolor=magenta]{hyperref}
\usepackage{cancel}
\usepackage[ngerman]{babel}
\usepackage{harpoon}
\usetikzlibrary{tikzmark,calc,arrows,angles,math,decorations.markings}
\usepackage{pgfplots}
\usepackage{framed}
\usepackage[hyperref,amsmath,amsthm,thmmarks,thref,framed]{ntheorem}
\usepackage{tcolorbox}
\pgfplotsset{compat=1.17}
\title{Lineare Algebra 2}
\date{Sommersemester 2022}
\author{Philipp Grohs \\ \small \LaTeX-Satz: Anton Mosich}
\newcounter{textbox}
\def\tl{\stepcounter{textbox}\tikzmarknode{a\thetextbox}{\strut}}
\def\br{\tikzmarknode{b\thetextbox}{\strut}\begin{tikzpicture}[overlay, remember picture]
\draw ($(a\thetextbox.north west)+(-0.4\arraycolsep,0ex)$) rectangle
($(b\thetextbox.south east)+(0.2\arraycolsep,0ex)$);\end{tikzpicture}}
% https://tex.stackexchange.com/questions/481978/how-to-write-the-block-matrix-in-latex
\newcommand*{\vect}[1]{\overrightharp{\ensuremath{#1}}}
\newcommand\R{\ensuremath{\mathbb{R}}}
\newcommand\C{\ensuremath{\mathbb{C}}}
\newcommand\K{\ensuremath{\mathbb{K}}}
\newcommand\mapsfrom{\rotatebox{180}{$\mapsto$}}
\theoremstyle{break}
\theoremseparator{\smallskip}
\theoremindent=1em
\theoremheaderfont{\kern-1em\normalfont\bfseries}
\theoreminframepreskip{0em}
\theoreminframepostskip{0em}
\newtcbox{theoremBox}{colback=NavyBlue!17,colframe=NavyBlue!87,boxsep=0pt,left=7pt,right=7pt,top=7pt,bottom=7pt}
\def\theoremframecommand{\theoremBox}
\newshadedtheorem{theo}{Theorem}[section]
\newshadedtheorem{lemma}[theo]{Lemma}
\newshadedtheorem{defin}[theo]{Definition}
\newshadedtheorem{satz}[theo]{Satz}
\newshadedtheorem{korollar}[theo]{Korollar}
\newshadedtheorem{folgerung}[theo]{Folgerung}
\DeclareMathOperator{\sgn}{sgn}
\DeclareMathOperator{\rg}{rg}
\DeclareMathOperator{\spec}{spec}
\DeclareMathOperator{\spur}{sp}
\DeclareMathOperator{\Hom}{Hom}
\DeclareMathOperator{\adj}{adj}
\DeclareMathOperator{\id}{id}
\DeclareMathOperator{\diag}{diag}
\DeclareMathOperator{\eig}{Eig}
\DeclareMathOperator{\nxn}{n \times n}
\DeclareMathOperator{\im}{im}
\DeclareMathOperator{\GL}{GL}
\newcommand\homkv{\Hom_\K(V, V)}
\newcommand\homk{\Hom_\K}
\newcommand\inner[2]{\langle #1, #2 \rangle}
\newcommand\norm[1]{\lVert #1 \rVert}
\newcommand\ontop[2]{\genfrac{}{}{0pt}{0}{#1}{#2}}
\newcommand\abs[1]{\lvert #1 \rvert}
\begin{document}
\definecolor{pastellblau}{HTML}{5BCFFA}
\definecolor{pastellrosa}{HTML}{F5ABB9}
\definecolor{weiss}{HTML}{FFFFFF}
\tikzset{%
-||-/.style={decoration={markings,
mark=at position 0.5 with {\draw[thin, -] (-.2,-.2) -- (0, .2);\draw[thin, -] (0, -.2) -- (.2, .2);}},
postaction={decorate}},
}
\begin{titlepage}
\begin{tikzpicture}[remember picture, overlay]
% Trans pride flag
\foreach[count=\i] \col in {pastellblau,pastellrosa,weiss,pastellrosa,pastellblau}
\node (back names) [shape=rectangle,
fill=\col,
minimum width=\paperwidth / 5,
anchor=south west,
minimum height=\paperheight] at ([xshift=(\i - 1)*(\paperwidth / 5)]current page.south west) {};
% The Loss
% Panel 1
\draw[very thick, <-] ([xshift = 85pt, yshift = -60pt]current page.north west) -- +(0, -.35\paperheight);
% Panel 2
\draw[very thick, <-, -||-] ([xshift = -85pt, yshift = -.1\paperheight - 60pt] current page.north east) --
+(0, -.25\paperheight);
\draw[very thick, <-, -||-] ([xshift = -185pt, yshift = -60pt]current page.north east) --
+(0, -.35\paperheight);
% Panel 3
\draw[very thick, ->, -||-] ([xshift = 40pt, yshift = 60pt] current page.south west) -- +(0, .35\paperheight);
\draw[very thick, ->, -||-] ([xshift = 175pt, yshift = 60pt] current page.south west) -- +(0, .35\paperheight);
% Panel 4
\draw[very thick, ->] ([xshift = -175pt, yshift = 60pt] current page.south east) -- +(0, .35\paperheight);
\draw[very thick, <-] ([yshift = 120pt, xshift = -25pt] current page.south east) -- +(-.38\paperwidth,0);
\draw[very thick] ([xshift = -175+40pt, yshift = 120]current page.south east)
arc [radius=40pt, start angle=0, end angle=90];
% Title, Author & Date
\node at ([yshift = -.45\paperheight]current page.north) {\Huge{ \textbf{Lineare Algebra 2} }};
\node at ([yshift = -.52\paperheight]current page.north) {\Large{Philipp Grohs}};
\node at ([yshift = -.55\paperheight]current page.north) {\large{\LaTeX-Satz: Anton Mosich}};
\node at ([yshift = -.60\paperheight]current page.north) {\large{Sommersemester 2022}};
\end{tikzpicture}
\end{titlepage}
\tableofcontents
\chapter{Determinanten}
\section{Permutationen}
\begin{defin}
Sei $n \in \mathbb{N} \setminus \{0\}, [n] := \{1, 2, \dots, n\}$. \\
Eine bijektive Abbildung $\pi:[n]\to[n]$ heißt \underline{Permutation} von $[n]$.
Wir definieren die \underline{symmetrische Gruppe}
$S_n := \{\pi\text{ Permutation von }[n]\}$
mit der Hintereinanderausführung als Gruppenoperation.
\end{defin}
\subsubsection{Bemerkung}
\begin{itemize}
\item $(S_n, \circ)$ ist eine Gruppe.
\item $\pi\in S_n$ ist eindeutig durch das Tupel $(\pi(1), \dots, \pi(n))$ definiert.
\item Fixpunkte $(\pi(i)=i)$ werden oft weggelassen.
\end{itemize}
\begin{defin}
$\pi\in S_n$ heißt \underline{Transposition} wenn es $i, j\in [n]$ gibt mit
\[\pi(k) = \begin{cases} k & k\notin\{i, j\}\\ i & k = j\\ j & k=i \end{cases}\]
Wir schreiben $\pi = (ij)$.
\end{defin}
\begin{satz}
\label{theo:1.1.3}
Es gilt $\lvert S_n \rvert = n!$.
\end{satz}
\begin{proof}
Vollständige Induktion
\begin{itemize}
\item $n=1: S_1 = \{\id\}\implies\lvert S_1\rvert = 1 = 1!$
\item $n-1\to n:$\\ Angenommen $\lvert S_{n-1} \rvert = (n-1)!$.
Dann gilt $\lvert\{\pi \in S_n: \pi(n) = n \}\rvert = (n-1)!$. Sei allgemein $i \in [n]$.
Dann gilt $\pi(n)=i \iff (in)\circ\pi(n)=n$. Also gilt
\begin{align*}
& \lvert\{\pi\in S_n: \pi(n)=i\}\rvert = \lvert\{(in)\circ\pi: \pi(n)=n\}\rvert \\
& = \lvert\{\pi: \pi(n)=n\}\rvert = (n-1)!
\end{align*}
Weiters gilt
\begin{align*}
& S_n = \bigcup_{i\in[n]}^\bullet\{\pi\in S_n: \pi(n)=i\} \implies \\
& \lvert S_n\rvert = \sum_{i\in[n]}\lvert\{\pi \in S_n: \pi(n) = i\}\rvert
= n\cdot(n-1)! = n!
\end{align*}
\end{itemize}
\end{proof}
\begin{satz}
\label{theo:1.1.4}
Für $n\in \mathbb{N}_{\ge2}$ ist jedes $\pi \in S_n$ das Produkt von (endlich vielen) Transpositionen.
\end{satz}
\begin{proof}
\begin{itemize}
\item $n=2: S_2 = \{\id, (2 1)\}$
\item $n-1\to n$\\
Sei $\pi \in S_n$. Dann gilt (siehe Beweis von Satz \ref{theo:1.1.3}) mit $i=\pi(n)$, dass
\[\underbrace{(i n)\pi}_{\pi_i}(n) = n\]
Sei $\pi_i = (\underbrace{\pi_i(1) \dots \pi_i(n-1)}_{\in S_{n-1}} n)
\underset{\text{Induktions VS}}{\implies} \pi_i = (i_1 j_1) \dots (i_k j_k)$.\\
Außerdem gilt $\pi = (i n)\pi_i$, also $\pi = (i n)(i_1 j_1) \dots (i_k j_k)$
\end{itemize}
\end{proof}
\subsubsection{Bemerkung}
\begin{itemize}
\item Produktdarstellung ist nicht eindeutig, zum Beispiel:\\ $(3 1 2) = (2 1)(3 1) = (3 1)(3 2)$
\item $f\in \mathbb{Z}[X_1, \dots, X_n], \pi \in S_n$ \\
$\pi f(X_1, \dots, X_n) := f(X_{\pi(1)}, \dots, X_{\pi(n)})$
\end{itemize}
\subsubsection{Beispiel}
$\pi = (2 3 1), f(X_1, X_2, X_3) = X_1-X_2+X_1X_3 \implies \pi f(X_1, X_2, X_3) = X_2 - X_3 + X_2X_1$
\begin{lemma}
\label{theo:1.1.5}
Sei
\[
f(X_1, \dots, X_n) = \prod_{\substack{i, j\in[n]\\ i < j}} (X_j-X_i)\in \mathbb{Z}[X_1, \dots, X_n]
\]
Dann gilt \begin{enumerate}[label=\alph*)]
\item Zu jedem $\pi \in S_n$ existiert eine eindeutig Zahl $s(\pi) \in \{-1, 1\}$ mit $\pi f = s(\pi)f$.
\item Für $\pi$ eine Transposition gilt $s(\pi) = -1$.
\end{enumerate}
\end{lemma}
\begin{proof}
\begin{enumerate}[label=\alph*)]
\item \begin{align*}
\pi f(X_1, \dots, X_n) & = \prod_{i<j}(X_{\pi(j)}-X_{\pi(i)}) \\
& =\Bigl(\prod_{\substack{i<j \\
\pi(i)<\pi(j)}}
(X_{\pi(j)}-X_{\pi(i)})\Bigr)
\Bigl(\prod_{\substack{i<j \\
\pi(j)<\pi(i)}}(X_{\pi(i)}-X_{\pi(j)})\Bigr) \\
& = (-1)^{\lvert\{(i, j)\in[n]\times[n]:i<j\land\pi(i)>\pi(j)\}\rvert}
\prod_{i<j}(X_j-X_i) \\
& = s(\pi)f(X_1, \dots, X_n) \text{ mit } \\
s(\pi) & = (-1)^{\lvert\{(i, j)\in[n]\times[n]:i<j\land\pi(i)>\pi(j)\}\rvert}
\end{align*}
\item $\pi = (i j), i<j, k\in\{i+1, \dots, j-1\}$:
$\pi(i, j) = (j, i), \pi(i, k) = (j, k), \pi(k, j) = (k, i)$\\
Für diese Paare gilt $x<y \land \pi(x) > \pi(y)$\\
Für alle anderen Paare gilt $x<y \land \pi(x)<\pi(y)$\\
Erstere sind $2(j-i-1)+1$ Paare. Daraus folgt $\pi f=(-1)^{2(j-i-1)+1}f$, also $s(\pi)=-1$.
\end{enumerate}
\end{proof}
\begin{defin}
\leavevmode
\begin{itemize}
\item Die durch Lemma \ref{theo:1.1.5} bestimmte Größe $s(\pi)$ heißt \underline{Signum} von $\pi \in S_n$.
Wir schreiben $\sgn(\pi)$.
\item $\pi$ heißt \underline{gerade} falls $\sgn(\pi)=1$ und \underline{ungerade} falls $\sgn(\pi)=-1$.
\end{itemize}
\end{defin}
\begin{satz}
\label{theo:1.1.7}
Für $\pi, \sigma \in S_n$ gilt \[\sgn(\sigma\pi)=\sgn(\sigma)\sgn(\pi)\]
\end{satz}
\begin{proof}
Nach Satz \ref{theo:1.1.5}(a) gilt:
\begin{align*}
& f(X_1, \dots, X_n) = \prod\limits_{i<j}(X_j-X_i) \implies \\
& \sigma\pi f(X_1, \dots, X_n) = \sgn(\sigma\pi)f(X_1, \dots, X_n)
\end{align*}
Andererseits gilt:
\begin{align*}
\sigma\pi f(X_1, \dots, X_n) & = \sigma[\pi f(X_1, \dots, X_n)] \\
& = \sigma[\sgn(\pi)f(X_1, \dots, X_n)] \\
& = \sgn(\pi) \sigma f(X_1, \dots, X_n) \\
& = \sgn(\pi)\sgn(\sigma)f(X_1, \dots, X_n)
\end{align*}
\end{proof}
\begin{satz}
\leavevmode
\begin{enumerate}[label=\alph*)]
\item $\sgn(\pi)=1\iff\pi$ ist Produkt gerader Anzahl Transpositionen
\item $\pi$ Produkt von k Transpositionen $\implies \sgn(\pi)=(-1)^k$
\end{enumerate}
\end{satz}
\begin{proof}
Folgt direkt aus Satz \ref{theo:1.1.5}(b) und Satz \ref{theo:1.1.7}
\end{proof}
\begin{folgerung}
Es gibt genau $\frac12n!$ gerade und $\frac12n!$ ungerade Permutationen in $S_n$
\end{folgerung}
\begin{proof}
Folgt aus Satz \ref{theo:1.1.3}
\end{proof}
\begin{defin}
Die geraden Permutationen bilden eine Untergruppe $A_n$ von $S_n$, die man
\underline{alternierende Gruppe} nennt.
\end{defin}
\section{Multilinearformen}
\begin{defin}
Seien $V_1, \dots, V_n, W$ \K-Vektorräume. Eine Abbildung $\varphi: V_1 \times \dots \times V_n \to W$
heißt \underline{n-linear}, wenn für alle
$v_1, v'_1 \in V_1, \dots, v_n, v'_n\in V_n, i \in [n], \lambda\in\K$ gilt, dass
\begin{itemize}
\item $\varphi(v_1, \dots, v_i+v'_i, \dots, v_n)=
\varphi(v_1, \dots, v_i, \dots, v_n)+\varphi(v_1, \dots, v'_i, \dots, v_n)$
\item $\varphi(v_1, \dots, \lambda v_i, \dots, v_n)= \lambda\varphi(v_1, \dots, v_i, \dots, v_n)$.
\end{itemize}
Ist $W=\K$ und $V_1, \dots, V_n=V$, so heißt $\varphi$ \underline{n-Linearform}. \\
($n=2 \to$ \underline{Bilinearform})
\end{defin}
\subsubsection{Beispiel}
\[
\varphi:
\begin{cases}
\K^2\times \K^2 & \to \K \\
\left(\begin{pmatrix}a_{11}\\a_{21}\end{pmatrix}, \begin{pmatrix}a_{12}\\a_{22}\end{pmatrix}\right)
& \mapsto a_{11}a_{22} - a_{12}a_{21}
\end{cases}
\]
\begin{defin} \label{theo:1.2.2}
Eine n-Linearform von $V$ heißt
\begin{itemize}
\item \underline{nicht ausgeartet}, falls
$(a_1, \dots, a_n)\in V\times\dots\times V$ existiert mit \\
$\varphi(a_1, \dots, a_n) \neq 0$.
\item \underline{alternierend}, falls $\varphi(a_1, \dots, a_n)=0$ für $a_1, \dots, a_n$ linear abhängig.
\end{itemize}
\end{defin}
\subsubsection{Bemerkung}
$\varphi$ alternierend und $a_i = a_j$ für $i\neq j \implies \varphi(a_1, \dots, a_n) = 0$.
\begin{lemma}
\label{theo:1.2.3}
Sei $\varphi$ alternierende n-Linearform von V und $\pi \in S_n$. Dann gilt für\\
$a_1, \dots, a_n\in V$:
\[\varphi(a_{\pi(1)}, \dots, a_{\pi(n)})=\sgn(\pi)\varphi(a_1, \dots, a_n)\]
\end{lemma}
\begin{proof}
Wegen Satz \ref{theo:1.1.4} und Satz \ref{theo:1.1.7} genügt es anzunehmen, dass $\pi$ Transposition ist.
Sei also $\pi=(ij)$. Es gilt
\begin{align*}
0 & =\varphi(a_1, \dots, \underbrace{a_i+a_j}_{i}, \dots, \underbrace{a_i+a_j}_{j}, \dots, a_n) \\
& =\underbrace{\varphi(a_1, \dots, a_i, \dots, a_i, \dots, a_n)}_{0} +
\underbrace{\varphi(a_1, \dots, a_j, \dots, a_j, \dots, a_n)}_{0} \\
& \;\; + \varphi(a_1, \dots, a_i, \dots, a_j, \dots, a_n) +
\varphi(a_1, \dots, a_j, \dots, a_i, \dots, a_n) \\
& \implies \varphi(a_1, \dots, a_j, \dots, a_i, \dots, a_n)=
\underbrace{(-1)}_{=\sgn{\pi}}\varphi(a_1, \dots, a_i, \dots, a_j, \dots, a_n)
\end{align*}
\end{proof}
\begin{lemma}
\label{theo:1.2.4}
Sei $V$ ein $\K$-VR mit $\dim(V)=n$ und $\varphi$ nicht ausgeartete und alternierende n-Linearform von V.
Dann gilt
\[
a_1, \dots, a_n \text{ linear abhängig} \iff \varphi(a_1, \dots, a_n) = 0
\]
\end{lemma}
\begin{proof}
\begin{itemize}
\item[$\implies$]: folgt aus Definition \ref{theo:1.2.2}
\item[$\impliedby$]: z.Z.: $\varphi(b_1, \dots, b_n)\neq0\impliedby b_1, \dots, b_n \text{ Basis von } V$.
Da $\varphi$ nicht ausgeartet ist, gibt es $a_1, \dots, a_n\in V$ mit $\varphi(a_1, \dots, a_n)\neq0$.\\
Da $b_1, \dots, b_n$ Basis gibt es $\lambda_{ij}\in\K$ mit $a_i=\sum\limits_{j=1}^n{\lambda_{ij}b_j}$\\
Wegen n-Linearität gilt
\begin{align*}
0\neq\varphi(a_1, \dots, a_n) & =\sum_{j_1=1}^n{\dots}\sum_{j_n=1}^n{\varphi(b_{j_1}, \dots, b_{j_n})
\lambda_{1j_1}\cdots\lambda_{nj_n}} \\
& \underbrace{=}_{\mathclap{\varphi\text{ alternierend}}}
\sum_{\substack{j_1, \dots, j_n \\
\text{paarweise verschieden}}}
{\varphi(b_{j_1}, \dots, v_{j_n})\lambda_{1j_1} \cdots \lambda_{nj_n}} \\
& = \sum_{\pi\in S_n} \varphi(b_{\pi(1)}, \dots, b_{\pi(n)})
\lambda_{1\pi(1)} \cdots \lambda_{n\pi(n)} \\
& \underbrace{=}_{\mathclap{\text{Lemma \ref{theo:1.2.3}}}}
\varphi(b_1, \dots, b_n)\left(\sum_{\pi\in S_n}
\sgn(\pi)\lambda_{1\pi(1)}\cdots\lambda_{n\pi(n)}\right) \\
& \implies\varphi(b_1, \dots, b_n)\neq 0
\end{align*}
\end{itemize}
\end{proof}
\begin{satz}
\label{theo:1.2.5}
Sei V $\K$-VR mit $\dim(V)=n$ und Basis $a_1, \dots, a_n$.
\begin{enumerate}[label=\alph*)]
\item Für $\varphi$ alternierende nicht ausgeartete n-Linearform gilt für\\
$b_i = \sum\lambda_{ij}a_j$, dass
\[
\varphi(b_1, \dots, b_n) =
\varphi(a_1, \dots, a_n)(\sum_{\pi \in S_n}\sgn(\pi)\lambda_{1\pi(1)}\cdots\lambda_{n\pi(n)})
\]
\item Sei $c\in\K\setminus\{0\}$. Dann ist die Abbildung
\[
\varphi(b_1, \dots, b_n) = c(\sum_{\pi \in S_n}\sgn(\pi)\lambda_{1\pi(1)}\cdots\lambda_{n\pi(n)})
\]
eine alternierende nicht ausgeartete n-Linearform.
\end{enumerate}
\end{satz}
\begin{proof}
\begin{enumerate}[label=\alph*)]
\item folgt aus dem Beweis von Lemma \ref{theo:1.2.4}.
\item Man verifiziert leicht, dass $\varphi$ n-linear ist. Weiters ist $\varphi$ nicht ausgeartet, da
\[
\varphi(a_1, \ldots, a_n) =
c(\sum_{\pi\in S_n}\sgn(\pi)\delta_{1\pi(1)}, \cdots, \delta_{n\pi(n)}) = c \cdot 1 \neq 0
\]
z.Z.: $\varphi$ alternierend. Seien $b_1, \dots, b_n$ linear abhängig.\\
O.B.d.A. $b_1=\mu_2b_2+\cdots+\mu_nb_n$. Dann gilt
\[\varphi(b_1, \dots, b_n) = \sum_{j=2}^{n}\mu j \varphi(b_j, b_2, \dots, b_n)\]
Es genügt also zu zeigen, dass
$\varphi(b_1, \dots, b_n) = 0$ falls $b_1 = b_i, i\in\{2, \dots, n\}$.
Dann gilt aber $\lambda_{1j}=\lambda_{ij} \forall j$.
\begin{align*}
\varphi(b_i, \dots, b_i, \dots, b_n) & = c\cdot\sum_{\pi\in S_n} \sgn(\pi) \lambda_{i\pi(1)}\cdots\lambda_{i\pi(i)}\cdots\lambda_{n\pi(n)} \\
& =c\cdot \Bigg(\sum_{\pi\in A_n}\sgn(\pi)\lambda_{i\pi(i)}\cdots\lambda_{i\pi(i)}\cdots\lambda_{n\pi(n)} \\
& +\sum_{\pi\in A_n}\underbrace{\sgn(\pi\circ(1i))}_{=-\sgn(\pi)}\lambda_{i\pi(i)}\cdots\lambda_{i\pi(i)}\cdots\lambda_{n\pi(n)}\Bigg) \\
& =c\cdot\sum_{\pi\in A_n}(\sgn(\pi)-\sgn(\pi)) \cdot \cdots \\
& \cdot \cdots \lambda_{i\pi(i)}\cdots\lambda_{i\pi(i)}\cdots\lambda_{n\pi(n)}=0
\end{align*}
\end{enumerate}
\end{proof}
\subsubsection{Bemerkung}
Es gibt also zu jedem $\K$-VR V mit $\dim(V)=n$ eine nicht ausgeartete alternierende n-Linearform.
\begin{satz}
\label{theo:1.2.6}
Sei V $\K$-VR mit $\dim(V)=n$ und $\varphi_1, \varphi_2$ nicht ausgeartete alternierende n-Linearformen.
Dann existiert $c\in\K\setminus\{0\}$ mit $\varphi_2=c\cdot\varphi_1$.
\end{satz}
\begin{proof}
Sei $a_1, \dots, a_n$ Basis von V. Nach Lemma \ref{theo:1.2.4} ist
$\varphi_i(a_1, \dots, a_n)\neq0, i=1, 2$.\\
Sei $c:=\dfrac{\varphi_1(a_1, \dots, a_n)}{\varphi_2(a_1, \dots, a_n)} \in \K\setminus\{0\}$.\\
Sei $b_1, \dots, b_n$ mit $b_i=\sum\lambda_{ij}a_j$.\\
Dann gilt nach Satz \ref{theo:1.2.5}(a), dass für $i=1, 2$
\begin{align*}
& \varphi_i(b_1, \dots, b_n) =
\varphi_i(a_1, \dots, a_n)\underbrace{\sum_{\pi \in S_n}\lambda_{1\pi(1)}\cdots\lambda_{n\pi(n)}}_
{\text{unabhängig von $i$!}} \\
& \implies \frac{\varphi_1(b_1, \dots, b_n)}{\varphi_2(b_1, \dots, b_n)}=
\frac{\varphi_1(a_1, \dots, a_n)}{\varphi_2(a_1, \dots, a_n)}=c
\end{align*}
\end{proof}
\section{Determinanten}
\begin{defin}
Sei $B=(a_1, \dots, a_n)$ Basis des \K-Vektorraums V.
Sei $\varphi$ nicht ausgeartete n-Linearform und $\alpha \in \homkv$.
Dann ist die \underline{Determinante von $\alpha$} definiert durch \[
\det(\alpha):=\det{}_\K(\alpha)
:=\frac{\varphi(\alpha(a_1), \dots, \alpha(a_n))}{\varphi(a_1, \dots, a_n)}
\]
\end{defin}
\begin{satz}
\label{theo:1.3.2}
$\det(\alpha)$ ist unabhängig von der Wahl der Basis B und der der Form $\varphi$.
\end{satz}
\begin{proof}
\begin{itemize}
\item[1. Fall] $\alpha$ nicht bijektiv\\
$\implies \alpha(a_1), \dots, \alpha(a_n) \text{linear unabhängig} \implies \det(\alpha) = 0$
\item[2. Fall] $\alpha$ bijektiv. Sei $B=(a_1, \dots, a_n)$.
Dann ist auch $\alpha(a_1), \dots, \alpha(a_n)$ Basis und,
da $\varphi$ nicht ausgeartet,
\[\varphi(\alpha(a_1), \dots, \alpha(a_n))\neq0\]
Sei $\varphi_\alpha(b_1, \dots, b_n) := \varphi(\alpha(b_1), \dots, \alpha(b_n))$.
Dann ist $\varphi_\alpha$ alternierend und nicht ausgeartet. Wegen Satz \ref{theo:1.2.6} folgt,
dass $c\in\K\setminus\{0\}$ existiert mit
\begin{equation}\label{eq:constantphi}
\varphi_\alpha=c\cdot\varphi
\end{equation}
und (durch Einsetzen von $a_1, \dots, a_n$), dass $c=\det(\alpha)$.
Da \ref{eq:constantphi} unabhängig von B ist also $\det(\alpha)$ unabhängig von B.
Sei nun $\psi$ eine zweite alternierende, nicht ausgeartete n-Form und
$\psi_\alpha(b_1, \dots, b_n) := \psi(\alpha(b_1), \dots, \alpha(b_n))$.
Dann ist $\psi_\alpha$ alternierend und nicht ausgeartet. Nach Satz \ref{theo:1.2.6}
gibt es $d\in\K\setminus\{0\} \text{ mit }d=\frac\psi\varphi$.
Also gilt:
\[
\det(\alpha)=\frac{\varphi_\alpha(a_1, \dots, a_n)}{\varphi(a_1, \dots, a_n)}=
\frac{d\varphi_\alpha(a_1, \dots, a_n)}{d\varphi(a_1, \dots, a_n)}=
\frac{\psi_\alpha(a_1, \dots, a_n)}{\psi(a_1, \dots, a_n)}
\]
also ist $\det(\alpha)$ auch von der n-Form unabhängig.
\end{itemize}
\end{proof}
\begin{korollar}
\label{theo:1.3.3}
Sei V ein n-dimensionaler \K-Vektorraum. Dann gilt
\begin{enumerate}[label=\alph*)]
\item $\alpha\in \homkv \text{ bijektiv } \iff \det(\alpha)\neq0$
\item $\alpha, \beta \in \homkv \implies \det(\alpha, \beta) = \det(\alpha) \det(\beta)$
\item $\det(\id)=1$
\item Ist $\alpha\in \homkv$ invertierbar, so gilt $\det(\alpha^{-1})=\det(\alpha)^{-1}$.
\end{enumerate}
\end{korollar}
\begin{proof}
Sei $B=(a_1, \dots, a_n)$ Basis und $\varphi$ n-Form mit \[
\det(\alpha) = \frac{\varphi(\alpha(a_1), \dots, \alpha(a_n))}{\varphi(a_1, \dots, a_n)}
\text{[ unabhängig von $B$ und $\varphi$ nach Satz \ref{theo:1.3.2}]}
\]
\begin{enumerate}[label=\alph*)]
\item $\alpha$ bijektiv $\iff \alpha(a_1), \dots, \alpha(a_n) \text{ linear unabhängig}$\\
$\underbrace{\iff}_{\mathclap{\text{Lemma \ref{theo:1.2.4}}}}
\varphi(\alpha(a_1), \dots, \alpha(a_n))\neq0\iff \det(\alpha)\neq0$
\item 2 Fälle:\begin{enumerate}[label=\arabic. Fall]
\item[1. Fall:] $\alpha$ oder $\beta$ ist nicht bijektiv: o.B.d.A $\alpha$ nicht bijektiv.\\
$\implies \det(\alpha)=0\implies \det(\alpha)\det(\alpha)=0$\\
Weiters folgt, dass $\alpha\beta$ nicht bijektiv, also $\det(\alpha\beta)=0$.
\item[2. Fall:] $\alpha, \beta$ bijektiv.
Dann ist auch $(\beta(a_1), \dots, \beta(a_n))$ Basis und
\begin{align*}
\det(\alpha\beta) & = \frac{\varphi(\alpha(\beta(a_1)), \dots, \alpha(\beta(a_n)))}
{\varphi(a_1, \dots, a_n)} \\
& =\frac{\varphi(\alpha(\beta(a_1)), \dots, \alpha(\beta(a_n)))}
{\varphi(\beta(a_1), \dots, \beta(a_n))}\cdot \cdots \\
& \cdots \frac{\varphi(\beta(a_1), \dots, \beta(a_n))}
{\varphi(a_1, \dots, a_n)}\underbrace{=}_{\text{Satz \ref{theo:1.3.2}}}
\det(\alpha)\det(\beta)
\end{align*}
\end{enumerate}
\item $\det(\id)=\frac{\varphi(a_1, \dots, a_n)}{\varphi(a_1, \dots, a_n)}=1$
\item $1\underbrace{=}_{\text{c)}}\det(\id)=\det(\alpha\alpha^{-1})\underbrace{=}_{\text{b)}}
\det(\alpha)\det(\alpha^{-1})$
\end{enumerate}
\end{proof}
\begin{satz}
\label{theo:1.3.4}
Sei $\alpha\in \homkv, B=(b_1, \dots, b_n)$ Basis und $A=(a_{ij}) = {}_B M(\alpha)_B\in\K^{n\times n}$.
Dann gilt
\[\det(\alpha)=\sum_{\pi\in S_n}\sgn(\pi)a_{1\pi(1)}\cdots a_{n\pi(n)}\]
\end{satz}
\begin{proof}
Es gilt $\alpha(b_i)=\sum\limits_{j=1}^na_{ij}b_j \text{ für }i=1, \dots, n$.
Nach Satz \ref{theo:1.2.5}(a) gilt
\[
\varphi(\alpha(b_1), \dots, \alpha(b_n)) =
\varphi(b_1, \dots, b_n)\cdot\sum_{\pi\in S_n}\sgn(\pi)a_{1\pi(1)}\cdots a_{n\pi(n)}
\]
und daraus folgt die Behauptung direkt.
\end{proof}
\begin{defin}
Für $A=(a_{ij})\in\K^{n\times n}$ definieren wir die \underline{Determinante von A} als
\[
\det(A)=\sum_{\pi\in S_n} \sgn(\pi)a_{1\pi(1)}\cdots a_{n\pi(n)}\in\K
\]
\end{defin}
\subsubsection{Bemerkung}
Schreibweise für $A=(a_{ij})$:
\[
\det(A)=\begin{vmatrix}
a_{11} & \dots & a_{1n} \\
\vdots & \ddots & \vdots \\
a_{n1} & \dots & a_{nn}
\end{vmatrix}
\]
\section{Rechenregeln}
\begin{satz}
\label{theo:1.4.1}
Sei $A=(a_1, \dots, a_n)\in\K^{n\times n}$. Dann gilt
\begin{enumerate}[label=\alph*)]
\item $\det(A)=\det(A^T)$
\item $\forall i, j\in[n]: i<n:
\det((a_1, \dots, \underbrace{a_j}_{i}, \dots, \underbrace{a_i}_{j}, \dots, a_n))=\det(A)$
\item $\forall i\in[n]: \lambda_1, \dots, \lambda_n\in\K: \det((a_1, \dots, a_i+
\sum\limits_{\substack{j=1\\j\neq i}}^n\lambda_ja_j, \dots, a_n))=\det(A)$
\item $\forall i\in[n]: \lambda\in\K: \det((a_1, \dots, \lambda a_i, \dots, a_n)) = \det(A)$
\item $\exists i, j\in[n]: i\neq j\land a_i=a_j \implies \det(A)=0$
\item $\forall \lambda \in \K: \det(\lambda A)=\lambda^n \det(A)$
\item $A$ invertierbar $\implies \det(A^{-1})=\det(A)^{-1}$
\item $\forall B \in \K^{n\times n}: \det(AB)=\det(A)\det(B)$
\item $\det(I_n)=1$
\end{enumerate}
\end{satz}
\begin{proof}
Nur a) explizit:
\begin{enumerate}[label=\alph*)]
\item \begin{equation*}\begin{aligned}
\det(A^T) & = \sum_{\pi\in S_n}\sgn(\pi)a_{\pi(1)1}\cdots a_{\pi(n)n} \\
& =\sum_{\pi\in S_n}\sgn(\pi)a_{1\pi^{-1}(1)}\cdots a_{n\pi^{-1}(n)} \\
& \underbrace{=}_{\substack{\sgn(\pi^{-1})=\sgn(\pi) \\
\pi^{-1}\mapsto\pi}} \sum_{\pi\in S_n} \sgn(\pi)a_{1\pi(1)}\cdots a_{n\pi(n)}
\end{aligned}\end{equation*}
\item[b) - i)] folgt daraus, dass für \[\alpha:\begin{cases}\K^n\to\K^n\\x\mapsto A\cdot x\end{cases}:
\det(A)=\dfrac{a}{b}\text{ (Satz \ref{theo:1.3.4})}\] und, dass $\varphi$ alternierende n-Form ist,
beziehungsweise Korollar \ref{theo:1.3.3}
\end{enumerate}
\end{proof}
\begin{satz}
Seien $A, B\in\K^{n\times n}$ ähnlich,
das heißt $\exists P\in\K^{n\times n}$ invertierbar mit \\
$B=P^{-1}\cdot A\cdot P$. Dann gilt
\[\det(A)=\det(B)\]
Weiters ist A genau dann invertierbar wenn $\det(A)\neq0$.
\end{satz}
\begin{proof}
\[\det(B)=\det(P)\underbrace{\det(P^{-1})}_{=\det(P)^{-1}}\det(A)=\det(A)\]
Rest folgt, da $\det(A)=\det(\alpha)$ mit $\alpha:\begin{cases}\K^n\to\K^n \\x\mapsto A\cdot x
\end{cases}$.
\end{proof}
\subsubsection{Berechnungsverfahren}
Gaußalgorithmus führt 1) Zeilenvertauschungen und 2) Additionen von\\
Vielfachen einer Zeile zu einer anderen durch. Raus kommt eine obere Dreiecksmatrix.
\begin{equation}\label{eq:dreiecksmatrix}
B=\begin{pmatrix}
b_{11} & \dots & \dots & b_{1n} \\
0 & b_{22} & \dots & b_{2n} \\
\vdots & & \ddots & \vdots \\
0 & \dots & \dots & b_{nn}
\end{pmatrix}
\end{equation}
Operationen 2) ändern die Determinante nicht, Operationen 1) ändern das Vorzeichen.
\begin{satz}
Sei $A\in\K^{n\times n}$ und $B$ wie \ref{eq:dreiecksmatrix} das Resultat des Gaußalgorithmus auf $A$
angewendet mit $k$ Zeilenvertauschungen. Dann gilt
\[
\det(A)=(-1)^kb_{11}\cdot\dots\cdot b_{nn}
\]
\end{satz}
\begin{proof}
Für Matrizen der Form \ref{eq:dreiecksmatrix} ist die Determinante das Produkt der Diagonalelemente.
Rest folgt aus der Definition des Gaußalgorithmus, sowie Satz \ref{theo:1.4.1}.
\end{proof}
\subsubsection{Regel von Sarrus}
Sei $A=\begin{pmatrix}
a_{11} & a_{12} & a_{13} \\
a_{21} & a_{22} & a_{23} \\
a_{31} & a_{32} & a_{33}
\end{pmatrix}\in\K^{3\times3}$
\[
\begin{array}{ccccccccc}
a_{11} & & a_{12} & & a_{13} & & a_{11} & & a_{12} \\
& \color{ForestGreen}\diagdown & & \color{ForestGreen}\diagdown \color{red} \mathllap \diagup & & \color{ForestGreen}\diagdown \color{red} \mathllap \diagup & & \color{red}\diagup \\
a_{21} & & a_{22} & & a_{23} & & a_{21} & & a_{22} \\
& \color{red} \diagup & & \color{ForestGreen}\diagdown \color{red} \mathllap \diagup & & \color{ForestGreen}\diagdown \color{red} \mathllap \diagup & & \color{ForestGreen}\diagdown \\
a_{31} & & a_{32} & & a_{33} & & a_{31} & & a_{32}
\end{array} \color{ForestGreen} + + + \color{red} - - -
\]
$A=\begin{pmatrix}a_{11} & a_{12} \\ a_{21} & a_{22}\end{pmatrix}
\in\K^{2\times2} \implies \det(A)=a_{11}a_{22}-a_{12}a_{21}$\\
$n>3 \to $ Gaußalgorithmus
\begin{defin}
Sei $A\in\K^{n\times n}$ und $i, j\in[n]$. Sei $M_{ij}\in\K^{n\times n}$ die Matrix,
welche durch Ersetzung der j-ten Spalte durch den i-ten Einheitsvektor $e_j$ entsteht.\\
$A_{ij}:=\det(M_{ij})$ heißt \underline{Kofaktor} (zum Indexpaar $(i, j)$).
\begin{equation*}
\bordermatrix{
&&&&j \cr
&a_{11}&\dots &a_{1i-1}&0&a_{1i+1}&\dots&a_{1n} \cr
&\vdots&\ddots&\vdots&\vdots&\vdots&\ddots&\vdots \cr
i&a_{ji}&\dots&a_{ji-1}&1&a_{ji+1}&\dots&a_{jn}\cr
&\vdots&\ddots&\vdots&\vdots&\vdots&\ddots&\vdots \cr
&a_{n1}&\dots &a_{ni-1}&0&a_{ni+1}&\dots&a_{nn}
}
\genfrac{}{}{0pt}{0}{= M_{ij}}{=(a_{\_1}, \dots, \underbrace{e_i}_{j}, \dots, a_{\_n})}
\end{equation*}
\end{defin}
\subsubsection{Bemerkung}
Es gilt
\begin{equation}\label{eq:1.4.4.1}
A_{ij}=\begin{vmatrix}
a_{11} & \dots & a_{1i-1} & 0 & a_{1i+1} & \dots & a_{1n} \\
\vdots & \ddots & \vdots & \vdots & \vdots & \ddots & \vdots \\
a_{ji} & \dots & a_{ji-1} & 1 & a_{ji+1} & \dots & a_{jn} \\
\vdots & \ddots & \vdots & \vdots & \vdots & \ddots & \vdots \\
a_{n1} & \dots & a_{ni-1} & 0 & a_{ni+1} & \dots & a_{nn}
\end{vmatrix}
\end{equation}
da obige Matrix aus $M_{ij}$ durch Spaltenadditionen hervorgeht.
\begin{lemma}
Sei $\tilde{A_{ij}}\in\K^{(n-1)\times(n-1)}$ die Matrix, welche aus A durch Streichung der i-ten Spalte und
j-ten Zeile hervorgeht und $D_{ij}:=\det(\tilde{A_{ij}})$. Dann gilt \[A_{ij}=(-1)^{i+j}D_{ij}\]
\end{lemma}
\begin{proof}
Transformiere durch ($i-1$) Spaltenvertauschungen und ($j-1$) Zeilenvertauschungen die Matrix
\ref{eq:1.4.4.1} auf
\[
B_{ij} = \begin{pmatrix}
1 & 0 & \dots & 0 \\
0 & & & \\
\vdots & & \tilde{A_{ij}} & \\
0 & & &
\end{pmatrix}
\]
Es gilt $\lvert B_{ij}\rvert=D_{ij}$ und $\lvert B_{ij}\rvert=(-1)^{(i-1)+j(-1)}A_{ij}$,
woraus die Behauptung folgt.
\end{proof}
\begin{satz}
[Entwicklungssatz von Laplace]
Sei $A\in\K^{n\times n}$ und $i, j\in[n]$. Dann gilt
\begin{enumerate}[label=\alph*)]
\item $\det(A) = \sum\limits_{l=1}^na_{il}A_{il} = \sum\limits_{l=1}^n(-1)^{l+i}a_{il}D_{il}$
\item $\det(A) = \sum\limits_{l=1}^na_{lj}A_{lj} = \sum\limits_{l=1}^n(-1)^{l+j}a_{lj}D_{lj}$
\end{enumerate}
\end{satz}
\begin{proof}
b) \[\begin{aligned}
\det(A) & = \det(a_{\_1}, \dots, a_{\_n})= \\
& =\det(a_{\_1}, \dots, \underbrace{\sum_{l=1}^na_{lj}e_l}_{=a_{\_j}}, \dots, a_{\_n})= \\
& =\sum_{l=1}^n a_{lj}\det(a_{\_1}, \dots, \underbrace{e_l}_{j}, \dots, a_{\_n}) = \\
& = \sum_{l=1}^n a_{lj}A_{lj}
\end{aligned}
\]
a) analog (angewendet auf $A^T$).
\end{proof}
\begin{satz}
[Cramer'sche Regel]
Sei $\adj(A)=(A_{ji})_{i, j\in[n]}$. Dann gilt
\[A\cdot \adj(A) = \det(A)\cdot I_n\]
\end{satz}
\begin{proof}
Sei $B=A\cdot\adj(A)\implies$
\[\begin{aligned}
b_{ij} & = \sum_{k=1}^n a_{ik} A_{jk} \\
& = \sum_{k=1}^n a_{ik} \det(a_{\_1}, \dots, \underbrace{e_j}_{k}, \dots, a_{\_n}) \\
& = \sum_{k=1}^n a_{ik}
\bordermatrix{
& & & k & & \\
& a_{11} & \dots & a_{1k} & \dots & a_{1n} \\
& \vdots & \ddots & \vdots & \ddots & \vdots \\
j & 0 & \dots & 1 & \dots & 0 \\
& \vdots & \ddots & \vdots & \ddots & \vdots \\
& a_{n1} & \dots & a_{nk} & \dots & a_{nn} \\
} \\
& = \det\left(\bordermatrix{ & \\
& a_{1\_} \\ & \vdots \\ j \to & a_{i\_} \\ & \vdots \\ & a_{n\_}}\right) \\
& = \begin{cases}0& i\neq j \\ \det(A) & i=j\end{cases}
\end{aligned}\]
\end{proof}
\begin{folgerung}
Sei $A\in\K^{n\times n}$ invertierbar. Sei $x\in\K^n$ die eindeutige Lösung des linearen Gleichunssystems
$Ax=b$. Dann gilt
\[
x_i= \det(A)^{-1} \det(a_{\_1}, \dots, \underbrace{b}_{i}, \dots, a_{\_n})
\]
\end{folgerung}
\begin{proof}
\[\begin{aligned}
& A^{-1}=\frac{1}{\det(A)}(A_{ji}) \\
& \implies \det(A)x_i=\sum_{j=1}^n A_{ji}b_j
& = \sum_{j=1}^n b_j \det(a_{\_1}, \dots, \underbrace{e_j}_{i}, \dots, a_{\_n}) \\
&
& =\det(a_{\_1}, \dots, \underbrace{b}_{i}, \dots, a_{\_n})
\end{aligned}\]
\end{proof}
\subsubsection{Blockmatrizen}
\begin{defin}
$A\in\K^{n\times n}$ heißt \underline{obere Blockmatrix} wenn $\exists p\in \{1, \dots, n-1\}$ mit $a_{ij}=0$
für $p+1\le i\le n, 1\le j\le p$, d.h.
\begin{equation}
\label{blockmatrix}
A=\bordermatrix{
\ &\overbrace{}^{p} & \overbrace{}^{n-p} \cr
p\{\ & P & D \cr % } TODO geschwungene Klammern besser machen
n-p\{\ &0&Q} % }
\end{equation}
Analog sind \underline{untere Blockmatrizen} definiert.
\end{defin}
\begin{satz}
\label{theo:1.4.10}
Sei $A$ obere Blockmatrix wie in \ref{blockmatrix}. Dann gilt $\det(A)= \det(P) \det(Q)$
\end{satz}
\begin{proof}
Sei $A = \begin{pmatrix} P & D \\ 0 & Q \end{pmatrix}$.\\
Wende elementare Zeilenumformungen der ersten $p$ Zeilen an, sodass $P$ obere Dreiecksform hat
(mit $s$ Zeilenvertauschungen) und elementare Zeilenumformungen der letzten $n-p$ Zeilen sodass
$Q$ obere Dreiecksform hat (mit $t$ Zeilenvertauschungen).
Bezeichne das Ergebnis mit $A'= \begin{pmatrix} P' & D \\ 0 & Q' \end{pmatrix}$, wobei $P', Q'$ obere
Dreiecksform haben.\\
Es folgt, dass $A', P', Q'$ obere Dreiecksform hat. Da die Determinante oberer Dreiecksmatrizen
das Produkt der Diagonalelemente ist, gilt $\det(A')=\det(P')\det(Q')$.\\
Weiters gilt $\det(A')=(-1)^{s+t} \det(A)$ (insgesamt $s+t$ Vertauschungen)
und $\det(P')= (-1)^s \det(P), \det(Q') = (-1)^t \det(Q)$. Daraus folgt die Behauptung.
\end{proof}
\chapter{Eigenwerte und Eigenvektoren}
\section{Diagonalisierbarkeit}
\begin{defin}
$D\in \K^{n\times n}$ heißt \underline{Diagonalmatrix} wenn $\forall i\neq j: d_{ij}=0$.
Wir schreiben auch
\[
\diag(\lambda_1, \dots, \lambda_n):=\begin{pmatrix}
\lambda_1 & 0 & \dots & 0 \\
0 & \lambda_2 & \dots & 0 \\
\vdots & \vdots & \ddots & \vdots \\
0 & 0 & \dots & \lambda_n
\end{pmatrix}
\]
\end{defin}
\subsubsection{Bemerkung}
\begin{itemize}
\item $A\in \K^{n\times m} \implies \diag(\lambda_1, \dots, \lambda_n)A = \begin{pmatrix}
\lambda_1 a_{1\_} \\
\vdots \\
\lambda_n a_{n\_}\end{pmatrix}$
\item $\diag(\lambda_1, \dots, \lambda_n)^k = \diag(\lambda_1^k, \dots, \lambda_n^k)$
\end{itemize}
\begin{defin}
\leavevmode
\begin{enumerate}[label=\alph*)]
\item $\alpha \in \homkv, \dim(V)<\infty$ heißt \underline{diagonalisierbar} (bzgl. $B$)
wenn eine geordnete Basis $B$ existiert mit ${}_B M(\alpha)_B$ Diagonalmatrix
\item $A\in\K^{n\times n}$ heißt diagonalisierbar wenn eine invertierbare Matrix $P\in\K^{n\times n}$
existiert mit $P^{-1}AP$ Diagonalmatrix.
\end{enumerate}
\end{defin}
\begin{lemma}
Sei $V$ ein $\K$-Vektorraum mit $\dim(V)=n<\infty$. \\
Dann gilt für $\alpha\in\homkv$ und $C$ Basis:
\[\alpha \text{ diagonalisierbar} \iff {}_C M(\alpha)_C \text{ diagonalisierbar}\]
\end{lemma}
\begin{proof}
\begin{itemize}
\item[$\implies$] Sei $\alpha$ diagonalisierbar und $B$ eine Basis mit $_B M(\alpha)_B$ Diagonalmatrix.
Dann gilt
\begin{align*}
{}_B M(\alpha)_B & = {}_B M(\id)_C \cdot {}_C M(\alpha)_C \cdot {}_C M(\id)_B \\
& = {}_C M(\id)_{B^{-1}} \cdot {}_C M(\alpha)_C \cdot {}_C M(\id)_B
\end{align*}
Also ist ${}_C M(\alpha)_C$ diagonalisierbar.
\item[$\impliedby$] Sei ${}_C M(\alpha)_C$ diagonalisierbar und $P$ invertierbar mit
$P^{-1}\cdot {}_C M(\alpha)_C \cdot P$ Diagonalmatrix.
Sei $B$ Basis mit $P={}_C M(\id)_B$.
Dann gilt ${}_B M(\alpha)_B$ ist Diagonalmatrix.
\end{itemize}
\end{proof}
\begin{lemma}
\label{theo:2.1.4}
\leavevmode
\begin{enumerate}[label=\alph*)]
\item $\alpha \in \homkv$ ist diagonalisierbar genau wenn es eine Basis
$B=(b_1, \dots, b_n)$ und $\lambda_1, \dots, \lambda_n\in\K$ gibt mit
$\forall i=1, \dots, n:\alpha(b_i)=\lambda_i b_i$.
\item $A\in\K^{n\times n}$ ist diagonalisierbar genau wenn es eine geordnete Basis $B= (b_1, \dots, b_n)$
von $\K^n$ gibt mit $\forall i=1, \dots, n: A b_i = \lambda_i b_i$.
\end{enumerate}
\end{lemma}
\begin{proof}
\begin{enumerate} [label=\alph*)]
\item die Bedingung ist äquivalent zu ${}_B M(\alpha)_B$ diagonalisierbar.
\item Spezialfall von a).
\end{enumerate}
\end{proof}
\section{Eigenwerte und Eigenvektoren}
\begin{defin} \label{theo:2.2.1}
\leavevmode
\begin{enumerate}[label=\alph*)]
\item Sei $\alpha \in \homkv$. $\lambda\in\K$ heißt \underline{Eigenwert} von $\alpha$ wenn es einen Vektor
$v\in V\setminus\{0\}$ gibt mit $\alpha(v)=\lambda v$. $v$ heißt \underline{Eigenvektor} zu $\lambda$.\\
Die Menge aller Eigenwerte von $\alpha$ heißt \underline{Spektrum} von $\alpha; \spec(\alpha)$
\item Sei $A \in \K^{n\times n}$. $\lambda\in\K$ heißt \underline{Eigenwert} von $A$ wenn es
$v\in \K^n\setminus\{0\}$ gibt mit $A v = \lambda v$. $v$ heißt \underline{Eigenvektor} zu $\lambda$.\\
Die Menge aller Eigenwerte von $A$ heißt \underline{Spektrum} von $A; \spec(A)$
\end{enumerate}
\end{defin}
\begin{lemma}
\label{theo:2.2.2}
\leavevmode
\begin{enumerate}[label=\alph*)]
\item $\alpha \in \homkv$ diagonalisierbar $\iff \exists$ Basis aus Eigenvektoren.
\item $A \in \K^{n\times n}$ diagonalisierbar $\iff \exists$ Basis aus Eigenvektoren.
\end{enumerate}
\end{lemma}
\begin{proof}
Folgt direkt aus Lemma \ref{theo:2.1.4} und Definition \ref{theo:2.2.1}
\end{proof}
\begin{defin}
\leavevmode
\begin{enumerate}[label=\alph*)]
\item Sei $\alpha \in \homkv$ und $\lambda \in \spec(\alpha)$.
Dann heißt $\eig_\alpha(\lambda):=\{v\in V: \alpha(v) = \lambda v \}$ der zugehörige \underline{Eigenraum}.
\item Sei $A \in \K^{n\times n}$ und $\lambda \in \spec(A)$.
Dann heißt $\eig_A(\lambda):=\{v\in \K^n: A v = \lambda v \}$ der zugehörige \underline{Eigenraum}.
\end{enumerate}
\end{defin}
\begin{lemma}
Sei $\alpha \in \homkv / A\in\K^{n\times n}$ und $\lambda \in \spec(\alpha)/\lambda\in\spec(A)$.\\
Dann ist $\eig_\alpha(\lambda)/\eig_A(\lambda)$ ein Unterraum von $V/\K$.
\end{lemma}
\begin{proof}
Nur für $\alpha\in\homkv$
\begin{itemize}
\item $ 0 = \alpha(0) = \lambda \cdot 0 \implies 0 \in \eig_\alpha(\lambda) $
\item $v, w\in \eig_\alpha(\lambda) \implies \alpha(v+w)
= \alpha(v) + \alpha(w) = \lambda v + \lambda w
= \lambda(v + w) \implies v + w \in \eig_\alpha(V)$
\item $\mu \in \K, v \in \eig_\alpha(\lambda) \implies \alpha(\mu v) =
\mu \cdot \alpha(v) = \mu \cdot \lambda \cdot v =
\lambda \cdot (\mu \cdot v) \implies \mu \cdot v \in \eig_\alpha(\lambda)$
\end{itemize}
\end{proof}
\begin{satz}
Sei $\alpha \in \homkv$ und $B$ Basis. Dann gilt
\begin{align*}
& \spec(\alpha) = \spec({}_B M(\alpha)_B) \\
& {}_B\Phi(\eig_\alpha(\lambda)) = \eig_{{}_B M(\alpha)_B}(\lambda)
\end{align*}
\end{satz}
\begin{proof}
Sei $\lambda \in \spec(\alpha)$ und $v\in\eig_\alpha(\lambda)$. Dann gilt
\[
\alpha(v) = \lambda v \iff {}_B M(\alpha)_B \cdot {}_B v = \lambda \cdot {}_B v
\]
\end{proof}
\begin{defin}
\leavevmode
\begin{enumerate}[label=\alph*)]
\item Sei $\alpha \in \homkv, \dim(V)<\infty$ und $B$ Basis. Dann heißt die Funktion
\[
\chi_\alpha:\begin{cases}\K \to \K \\
\lambda \mapsto \det({}_B M(\alpha)_B - \lambda \cdot I_n)\end{cases}
\]
\underline{charakteristisches Polynom} von $\alpha$.
\item Sei $A \in \K^{n\times n}$. Dann heißt die Funktion
\[
\chi_A:\begin{cases}\K \to \K \\
\lambda \mapsto \det(A - \lambda \cdot I_n)\end{cases}
\]
\underline{charakteristisches Polynom} von $A$.
\end{enumerate}
\end{defin}
\subsubsection{Bemerkung}
$\genfrac{}{}{0pt}{0}{\chi_\alpha}{\chi_A}$ ist Polynom vom Grad
$\le\genfrac{}{}{0pt}{0}{\dim(V)}{n}$, da
\begin{align*}
& \chi_A(\lambda)=
\sum_{\pi \in S_n} \tilde{a}_{1\pi(1)}^{(\lambda)} \cdots \tilde{a}_{n\pi(n)}^{(\lambda)} \text{ mit} \\
& \tilde{a}_{ij}^{(\lambda)} = \begin{cases} a_{ij} & i\neq j \\ a_{ij}-\lambda & i=j
\end{cases} \dots \text{ Polynom von Grad $0$ oder $1$}
\end{align*}
\begin{lemma}
\label{theo:2.2.7}
\leavevmode
\begin{enumerate}[label=\alph*)]
\item $\chi_\alpha$ ist unabhängig von der Wahl der Basis.
\item $\chi_A = \chi_B$ wenn $A, B$ ähnlich (das heißt $\exists P \in \K^{n \times n}: B = P^{-1}AP$)
\end{enumerate}
\end{lemma}
\begin{proof}
\begin{enumerate}[label=\alph*)]
\item Sei C weitere Basis.\\
Dann gilt $\underbrace{{}_C M(\alpha)_C}_{B}
= \underbrace{{}_C M(\id)_B}_{P^{-1}} \underbrace{{}_B M(\alpha)_B}_{A}
\underbrace{{}_B M(\id)_C}_{P}$. \\
Man kann also alles auf b) zurückführen.
\item \[\begin{aligned}
\chi_A(\lambda) & = \det(A-\lambda I) \\
& = \det(P)^{-1} \det(A - \lambda I) \det(P) \\
& = \det(P^{-1}) \det(A - \lambda I) \det(P) \\
& = \det(P^{-1}(A - \lambda I)P) \\
& = \det(P^{-1}AP-\lambda I) \\
& = \det(B - \lambda I) \\
& = \chi_B(\lambda)
\end{aligned}\]
\end{enumerate}
\end{proof}
\begin{lemma}
\leavevmode
\begin{enumerate}[label=\alph*)]
\item Sei $\alpha\in\homkv$. Dann gilt \[\spec(\alpha) = \{\lambda \in \K: \chi_\alpha(\lambda)=0\}\]
\item Sei $A\in \K^{\nxn}$. Dann gilt \[\spec(A) = \{\lambda \in \K: \chi_A(\lambda)=0\}\]
\end{enumerate}
\end{lemma}
\begin{proof}
Nur b)
\[\begin{aligned}
\lambda \in \spec(A) & \iff \exists v\in V \setminus \{0\}: A v = \lambda v \\
& \iff \exists v \in V \setminus \{0\}: (A - \lambda I) v = 0 \\
& \iff \ker(A - \lambda I) \neq \{0\} \\
& \iff A - \lambda I \text{ nicht injektiv} \\
& \iff \det(A - \lambda I) = 0
\end{aligned}\]
\end{proof}
\subsubsection{Beispiele}
\begin{alignat*}{3}
& A = \begin{pmatrix}\bar3 & \bar4 \\ \bar1 & \bar1 \end{pmatrix} \in \mathbb{Z}_5^{2\times2} & & \\
& \chi_A(\lambda) = \begin{vmatrix} \bar3 - \lambda & \bar4 \\ \bar1 & \bar1 - \lambda \end{vmatrix}
& & = (\bar3 - \lambda)(\bar1 - \lambda) - \bar4 \\
& & & = \bar3 - \bar4 \lambda + \lambda^2 - \bar4 \\
& & & = \bar4 - \bar4 \lambda + \lambda^2 = (\bar2 - \lambda)^2 \\
& \implies \spec(A) = \{2\} \\
& \eig_{\bar2}(A) = ? \\
& v \in \eig_{\bar2}(A) \iff \mathrlap{(A - \bar2 I)v = 0} \\
& \iff \mathrlap{\left(\begin{array}{c c | c}
\bar3 - \bar2 & \bar4 & \bar0 \\
\bar1 & \bar1 - \bar2 & \bar0
\end{array}\right)} \\
& \left(\begin{array}{c c | c}
\bar1 & \bar4 & \bar0 \\
\bar1 & \bar4 & \bar0
\end{array}\right) \\
& \left(\begin{array}{c c | c}
\bar1 & \bar4 & \bar0 \\
\bar0 & \bar0 & \bar0
\end{array}\right) \\
& \implies \eig_{\bar2}(A) = \left\langle\begin{pmatrix}\bar1 \\ \bar1\end{pmatrix} \right\rangle \\
& \implies A \mathrlap{\text{ nicht diagonalisierbar [Lemma \ref{theo:2.1.4} (b)]}}
\end{alignat*}
\begin{lemma}
Sei $A \in \C^{n\times n}$ mit reellen Einträgen. Dann gilt:
\begin{enumerate}[label=\alph*)]
\item $\lambda \in \spec(A) \implies \overline{\lambda} \in \spec(A)$
\item $v \in \eig_\lambda(A) \implies \overline{v} \in \eig_{\overline{\lambda}}(A)$
\end{enumerate}
\end{lemma}
\begin{proof}
\begin{enumerate}[label=\alph*)]
\item Klarerweise ist $\chi_A(\lambda)$ ein Polynom mit reellen Koeffizienten, also
$\chi_A(\lambda)=a_0+a_1 \lambda + \cdots + a_n \lambda^n, a_0, \dots, a_n\in\R$\\
Sei $\chi_A(\lambda)=0 \implies 0 = \overline0
= a_0 + a_1 \overline\lambda + \cdots + a_n \overline{\lambda} ^ n = \chi_A(\overline\lambda)$
\item $v\in\eig_\lambda(A) \implies A v = \lambda v \implies \overline{A V}
= \overline{\lambda v} \implies A \overline{v} = \overline\lambda \overline{v}$
\end{enumerate}
\end{proof}
\begin{lemma}
\label{theo:2.2.10}
Eigenvektoren zu unterschiedlichen Eigenwerten sind linear unabhängig.
\end{lemma}
\begin{proof}
Seien $v_i \in \eig_{\lambda_i}(A), i=1, \dots, r, \lambda_i \neq \lambda_j \text{ für } i\neq j$.
Induktion nach $r$
\begin{itemize}
\item[$r=1$:] $v_1$ ist linear unabhängig.
\item[$r-1\mapsto r$:] \begin{equation}\label{eq:2.2.10.1}
\mu_1 v_1 + \cdots + \mu_1 v_1 = 0 \end{equation}
\[ \implies A(\mu_1 v_1 + \cdots + \mu_r v_r) = 0 \]
\begin{equation}\label{eq:2.2.10.2}
\implies \lambda_1\mu_1 v_1 + \cdots \lambda_r \mu_r v_r = 0
\end{equation}
Weiters folgt durch Multiplikation von \ref{eq:2.2.10.1} mit $\lambda_r$,
dass \begin{equation}\label{eq:2.2.10.3}
\lambda_r \mu_1 v_1 + \cdots + \lambda_r \mu_r v_r = 0 \end{equation}
\[ \begin{aligned}
\text{\ref{eq:2.2.10.3}} - \text{\ref{eq:2.2.10.2}}
& \implies \underbrace{(\lambda_r - \lambda_1)}_{\neq0} \mu_1 v_1 + \cdots +
\underbrace{(\lambda_r - \lambda_{r-1})}_{\neq0} \mu_{r-1} v_{r-1} = 0 \\
& \implies v_1, \dots, v_{r-1} \text{ linear abhängig. \Lightning}
\end{aligned} \]
\end{itemize}
\end{proof}
\begin{lemma}
Sei $\alpha \in \homkv, \dim(V)=n \text{ oder } A \in \K^{\nxn}$ mit $n$ verschiedenen Eigenvektoren.
Dann ist $\alpha/A$ diagonalisierbar.
\end{lemma}
\begin{proof}
Wegen Lemma \ref{theo:2.2.10} gibt es Basis von Eigenvektoren.
Daher ist $\alpha/A$ diagonalisierbar wegen Lemma \ref{theo:2.2.2}.
\end{proof}
\begin{defin}
Sei $\spec(A) = \{\lambda_1, \dots, \lambda_r \}$ und
$(\lambda_1 - \lambda)^{k_1} \cdots (\lambda_r - \lambda)^{k_r} p \in\K[X]$ mit $p$
nicht durch Linearfaktoren teilbar (also keine Nullstellen in $\K$).\\
$k_i$ heißt \underline{algebraische Vielfachheit} des Eigenwerts $\lambda_i$.
Wir schreiben $k_i = m_a(\lambda_i)$.\\
$\dim(\eig_A(\lambda_i))$ heißt \underline{geometrische Vielfachheit} des Eigenwerts $\lambda_i$.
Wir schreiben $\dim(\eig_A(\lambda_i)) = m_g(\lambda_i)$
\end{defin}
\subsubsection{Beispiel}
\begin{itemize}
\item $\chi_A(\lambda) = \lambda^4 - 2 \lambda^3 + 2 \lambda^2 - 2\lambda + 1 \in \R[X]$\\
$\implies \chi_A(\lambda) = (1 - X)^2 \underbrace{(1 + \lambda^2)}_{p(\lambda)}$ \\
$\implies m_a(1) = 2$
\item Für $\K=\C$ zerfällt jedes Polynom in Linearfaktoren, also ist $p$ immer konstant.
\end{itemize}
\begin{satz}
Sei $\mu\in\spec(A)/\spec(\alpha)$. Dann gilt \[ 1\le m_g(\mu) \le m_a(\mu) \]
\end{satz}
\begin{proof}
Klarerweise gilt $1\le m_g(\mu)$ da $\mu$ Eigenwert ist.
Sei $r:= m_g(\mu)$ und $b_1, \dots, b_r$ Basis von $\eig_\alpha(\mu)$. Sei $B=(b_1, \dots, b_n)$ Basis.
Dann ist
\[ {}_B M(\alpha)_B =
\bordermatrix{
& & & & r & & \cr
& \mu & 0 & 0 & 0 & * & \dots & * \cr
& 0 & \mu & 0 & 0 & * & \dots & * \cr
& \vdots & & \ddots & \vdots & \vdots & & \vdots \cr
r & 0 & 0 & 0 & \mu & * & \dots & * \cr
& \vdots & \vdots & \vdots & \vdots & \vdots & & \vdots \cr
& 0 & 0 & 0 & 0 & * & \dots & *
}
\text{, also}
\]
\begin{align*}
\chi_\alpha(\lambda) & = \left\lvert \begin{array}{c | c}
\begin{smallmatrix}
\mu - \lambda & & \\
& \ddots & \\
& & \mu - \lambda
\end{smallmatrix} & A \\
\hline \\
0 & B
\end{array} \right\rvert \underbrace{=}_
{\text{Satz \ref{theo:1.4.10}}} \det
\begin{pmatrix}
\mu - \lambda & & 0 \\
& \ddots & \\
0 & & \mu - \lambda
\end{pmatrix} \cdot \det(B) \\
& = (\mu - \lambda)^r \det(B) \\
& \implies r \le m_a(\mu)
\end{align*}
\end{proof}
\begin{lemma}
Seien $A, B$ ähnlich und $\mu \in \spec(A) (=\spec(B) \text{ nach Lemma \ref{theo:2.2.7}})$.
Dann stimmen die geometrischen Vielfachheiten überein, das heißt $\dim(\eig_\mu(A)) = \dim(\eig_\mu(B))$.
\end{lemma}
\begin{proof}
Sei $B = P^{-1} A P$. Dann gilt
\begin{align*}
\eig_{\mu}(B) & = \ker(B - \mu I) = \ker(B - \mu P^{-1} P) \\
& = \ker(P^{-1} (A - \mu I) P) \\
& \underbrace{\implies}_{\mathclap{\text{
Für ähnliche Matrizen stimmen die Dimensionen der Kerne überein
}}} \dim(\eig_\mu(B)) = \dim\eig_\mu(A)
\end{align*}
\end{proof}
\begin{satz}
$A/\alpha$ diagonalisierbar $\iff$
\begin{enumerate}[label=\roman*)]
\item $\chi_{A/\alpha}$ zerfällt in Linearfaktoren, das heißt
\[
\chi_{A/\alpha}(\lambda)= (\lambda_1 - \lambda)^{k_1} \cdots (\lambda_r - \lambda)^{k_r},
\sum k_i = n
\]
\item algebraische und geometrische Vielfachheiten stimmen überein, das \\
heißt $m_a(\lambda_i) = m_g(\lambda_i), i=1, \dots, r$
\end{enumerate}
\end{satz}
\begin{proof}
\begin{itemize}
\item[$\impliedby$:] Aus i), ii) folgt, dass \begin{equation}\label{eq:2.2.15.1}
\sum_{i=1}^r \underbrace{\dim(\eig_\alpha(\lambda_i))}_{=m_g(\lambda_i)=:d_i} = n \end{equation}
Sei $b_i^1, \dots, b_i^{d_i}$ Basis von $\eig_\alpha(\lambda_i)$.
Wir zeigen, dass $B=\{b_i^1, \dots, b_i^{d_i}: i=1, \dots, r\}$ Basis ist.
\begin{enumerate}[label=\arabic*)]
\item $\lvert B \rvert = n$ folgt aus \ref{eq:2.2.15.1}
\item Ang. $\sum\limits_{i=1}^r
(\underbrace{\mu_i^1 b_i^1 + \cdots + \mu_i^{d_i} b_i^{d_i}}_{v_i}) = 0$ \\
$\underbrace{\implies}_
{\mathclap{\substack{v_i \text{Eigenwerte zu} \\ \text{verschiedenen Eigenvektoren} \\
+ \text{Lemma \ref{theo:2.2.10}}}}}
v_i = 0 \forall i=1, \dots, r \underbrace{\implies}_
{\mathclap{\substack{b_i^1, \dots, b_i^{d_i} \\ \text{Basis von }
\eig_\alpha(\lambda_i)}}} \mu_i^1, \dots, \mu_i^{d_i} = 0 \forall i=1, \dots, r$ \\
$ \implies B $ ist Basis aus Eigenvektoren
$\underbrace{\implies}_{\mathclap{\text{Lemma \ref{theo:2.2.2}}}} \alpha $ diagonalisierbar.
\end{enumerate}
\item[$\implies$:] Sei $\alpha$ diagonalisierbar. \[\begin{aligned}
& \implies \exists \text{ Basis } \{b_1, \dots, b_n\} \text{ aus Eigenvektoren} \\
& \implies {}_B M(\alpha)_B = \diag(\lambda_1, \dots, \lambda_n) \\
& \implies \chi_B(\lambda) = (\lambda_1 - \lambda) \cdots (\lambda_n - \lambda)
\end{aligned}\]
\end{itemize}
\end{proof}
\subsubsection{Diagonalisieren}
\begin{enumerate}[label=\arabic*)]
\item Zerlegung in Linearfaktoren
\[
\chi_A(\lambda) = (\lambda_1 - \lambda)^{m_a(\lambda_1)} \cdots (\lambda_r - \lambda)^{m_a(\lambda_r)}
\]
\item Bestimme Basis $B_i$ der Eigenräume
\[ \eig_A(\lambda_i) = \ker(A - \lambda_i I) \]
\item Ordne Basis $B= \bigcup\limits_{i=1}^n B_i$ zu $B= (b_1, \dots, b_n)$
\item Mit $S = (b_1, \dots, b_n)$ gilt dann
\[
\diag(\underbrace{\lambda_1, \dots, \lambda_n}_{
\mathclap{\substack{\text{Eigenwerte werden nach} \\
\text{Vielfachheit gezählt!} \\
\lambda_i \text{ ist Eigenwert von } b_i \text{!}}}
}) = S^{-1} A S
\]
\end{enumerate}
\subsubsection{Beispiel}
$A = \begin{pmatrix}
1 & 2 & 2 \\
2 & -2 & 1 \\
2 & 1 & -2
\end{pmatrix}$
\begin{enumerate}[label=\arabic*)]
\item \begin{align*}
\chi_A(\lambda) = & \begin{vmatrix}
1 -\lambda & 2 & 2 \\
2 & -2 -\lambda & 1 \\
2 & 1 & -2 -\lambda
\end{vmatrix} \\
\underbrace{=}_{\mathclap{\substack{\text{Entwicklung} \\ \text{nach 1. Zeile}}}}
& (1-\lambda) \begin{vmatrix} -2 -\lambda & 1 \\ 1 & -2 -\lambda \end{vmatrix}
+ (-2) \begin{vmatrix} 2 & 1 \\ 2 & -2-\lambda \end{vmatrix} \\
& + 2 \begin{vmatrix} 2 & -2 - \lambda \\ 2 & 1 \end{vmatrix} \\
= & \dots= -\lambda^3 - 3 \lambda^2 + 9\lambda + 27 = (3-\lambda)(-3-\lambda)^2
\end{align*}
\item $\lambda = 3$
\begin{align*}
& \left( \begin{array}{c c c | c} 1-3 & 2 & 2 & 0 \\ 2 & -2-3 & 1 & 0 \\ 2 & 1 & -2-3 & 0 \end{array} \right)
= \left( \begin{array}{c c c | c} -2 & 2 & 2 & 0 \\ 2 & -5 & 1 & 0 \\ 2 & 1 & -5 & 0 \end{array} \right) \\
& \sim \left( \begin{array}{c c c | c} -1 & 1 & 1 & 0 \\ 0 & -3 & 3 & 0 \\ 0 & 3 & -3 & 0 \end{array} \right)
\sim \left( \begin{array}{c c c | c} 1 & -1 & -1 & 0 \\ 0 & 1 & -1 & 0 \\ 0 & 0 & 0 & 0 \end{array} \right)
\sim \left( \begin{array}{c c c | c} 1 & 0 & -2 & 0 \\ 0 & 1 & -1 & 0 \\ 0 & 0 & 0 & 0 \end{array} \right) \\
& \implies \eig_A(3) = \left\langle \begin{pmatrix} 2 \\ 1 \\ 1 \end{pmatrix} \right\rangle
\end{align*}
$\lambda = -3$
\begin{align*}
& \left( \begin{array}{c c c | c} 1+3 & 2 & 2 & 0 \\ 2 & -2+3 & 1 & 0 \\ 2 & 1 & -2+3 & 0 \end{array} \right)
= \left( \begin{array}{c c c | c} 4 & 2 & 2 & 0 \\ 2 & 1 & 1 & 0 \\ 2 & 1 & 1 & 0 \end{array} \right) \\
& \sim \left( \begin{array}{c c c | c} 2 & 1 & 1 & 0 \\ 0 & 0 & 0 & 0 \\ 0 & 0 & 0 & 0 \end{array} \right) \\
& \implies \eig_A(-3) = \left\langle \begin{pmatrix} -1 \\ 0 \\ 2 \end{pmatrix}, \begin{pmatrix} -1 \\ 2 \\ 0 \end{pmatrix} \right\rangle
\end{align*}
\item \begin{align*}
& S = \begin{pmatrix} 2 & -1 & -1 \\ 1 & 0 & 2 \\ 1 & 2 & 0 \end{pmatrix} \\
& \implies S^{-1} A S = \begin{pmatrix} 3 & 0 & 0 \\ 0 & -3 & 0 \\ 0 & 0 & -3 \end{pmatrix}
\end{align*}
\end{enumerate}
\begin{lemma}
\label{theo:2.2.16}
Sei $A\in\K^{\nxn}$ und $\underbrace{\spur(A)}_{\mathclap{\color{red}\text{\dq Spur von $A$ \dq}}}
:= \sum\limits_{i=1}^n a_{ii}$
\[\chi_A(\lambda) = (-1)^n\lambda^n + (-1)^{n-1} \spur(A) \lambda^{n-1} + \cdots + \det(A)\]
\end{lemma}
\begin{proof}
$\chi_A(\lambda) = \sum\limits_{\pi \in S_n} \sgn(\pi) \prod\limits_{i=1}^n \tilde{a}_{i\pi(i)}$ mit
$\tilde{a}_{ij} = \begin{cases} a_{ij} & i\neq j \\ a_{ij} - \lambda & i=j\end{cases}$. \\
Wenn $\pi\neq \id$ gilt $\deg\left(\prod\limits_{i=1}^n \tilde{a}_{i\pi(i)}\right)\le n-2$,
da mindestens zwei Elemente vertauscht werden. Die Koeffizienten von Grad $n, n-1$ kann man also aus
$\prod\limits_{i=1}^n \tilde{a}_{ii} = \prod\limits_{i=1}^n (\tilde{a}_{ii} - \lambda)$ ablesen.
Daraus folgt die Behauptung für die höchsten beiden Koeffizienten.
Weiters gilt $\chi_A(0)=\det(A)$, was die Aussage für den konstanten Koeffizienten zeigt.
\end{proof}
\[
\sigma_j := (-1)^j \sum\limits_{\substack{S\subset [n] \\ \lvert S \rvert = n-j}}
\prod\limits_{s \in S} \lambda_s
\]
\begin{korollar}
\leavevmode
\begin{enumerate}[label=\alph*)]
\item $A\sim B \implies \spur(A)=\spur(B)$
\item A diagonalisierbar $\implies \spur(A)=\lambda_1 + \cdots + \lambda_n$ mit
$\lambda_1, \dots, \lambda_n$ Eigenwerte von $A$, nach Vielfachheit gezählt.
\item A diagonalisierbar $\implies \det(A)=\lambda_1 \cdot \dots \cdot \lambda_n$ mit
$\lambda_1, \dots, \lambda_n$ Eigenwerte von $A$, nach Vielfachheit gezählt.
\end{enumerate}
\end{korollar}
\begin{proof}
Folgt daraus, dass das charakteristische Polynom (und damit seine Koeffizienten) unter Ähnlichkeit
invariant sind (Lemma \ref{theo:2.2.7}) und Lemma \ref{theo:2.2.16}
\end{proof}
\begin{satz}
[Cayley-Hamilton]
\dq$\chi_A(A) = 0$\dq, das heißt sei $A\in \K^{\nxn}$ mit charakteristischem Polynom
$\chi_A(\lambda)=c_n \lambda^n + c_{n-1} \lambda^{n-1} + \cdots + c_0$.
Dann gilt
\[
\chi_A(A):=c_n A^n + c_{n-1} A ^{n-1} + \cdots c_0 I = 0 =
\begin{pmatrix}0 &\dots &0 \\ \vdots& \ddots &\vdots \\ 0 & \dots & 0\end{pmatrix} \in \K^{\nxn}
\]
\end{satz}
\begin{proof}
Sei $B := A^T - \lambda I =
\begin{pmatrix}
a_{11} - \lambda & a_{21} & \dots & a_{n1} \\
a_{12} & a_{22} - \lambda & \dots & a_{n2} \\
\vdots & \ddots & \ddots & \vdots \\
a_{1n} & a_{2n} & \dots & a_{nn} - \lambda
\end{pmatrix}
= (a_{ji} - \delta_{ij} \lambda)_{ij}$
und $C:= \adj(B)$, sodass
\begin{equation}
CB = \det(B) I_n = \chi_A = I_n [\chi_A = \chi_{A^T}
\label{eq:2.2.18.1}
\end{equation}
\ref{eq:2.2.18.1} heißt komponentenweise, dass
\begin{flalign}
& \sum_{i=1}^{n}
\underbrace{c_{ki}}_{\mathrlap{\text{Polynome, in die $A$ eingesetzt werden kann}}}
\underbrace{b_{ij}}
= \delta_{ij} \cdot \underbrace{\chi_A} \forall k, j \in [n] \nonumber \\
= & \sum_{i=1}^{n}c_{ki}(A) b_{ij}(A) = \delta_{jk}\chi_A (A) \label{eq:2.2.18.2}
\end{flalign}
Wegen $b_{ij}(A) = a_{ji} I_N - \delta_{ij}A$ gilt weiters
\begin{equation}
\forall i \in [n]: \sum_{j=1}^{n} b_{ij}(A) e_j = (\sum_{j=1}^{n} a_{ji} e_j) - A e_i = 0
\label{eq:2.2.18.3}
\end{equation}
Es folgt $\forall k \in [n]$
\begin{align*}
\chi_A (A) e_k & = \sum_{j=1}^{n} \delta_{jk} \chi(A) e_j & \\
& \underbrace{=}_{\mathclap{\text{\ref{eq:2.2.18.2}}}}
\sum_{j=1}^{n} \sum_{i=1}^{n} c_{ki}(A) b_{ij}(A) e_j & \\
& = \sum_{i=1}^{n} c_{ki}(A) (\sum_{j=1}^{n} b_{ij(A) e_j}) & \\
& \underbrace{=}_{\mathclap{\text{\ref{eq:2.2.18.3}}}} 0 & \\
& \implies \chi_A(A) = 0
\end{align*}
\end{proof}
\subsubsection{Berechnung der Koeffizienten von $\chi_A$}
Sei $f(\lambda) \underbrace{=}_{\text{(*)}} \prod\limits_{j=1}^{n}(\lambda_j - \lambda) =
\underbrace{c_n\lambda^n}_{=(-1)^n} + c_{n-1}\lambda ^{n-1} + \cdots + c_0$
Wie können wir $c_j$ effizient bestimmen?
\begin{itemize}
\item [Bemerkung 1:] $\displaystyle { c_j = (-1)^{j} \sum_{\substack{S\subseteq [n] \\
\lvert S \rvert = n-j}} \prod_{s \in S} \lambda_s =:
\sigma_{n-j}^n (\lambda_1, \dots, \lambda_n)}$ \\
Dies folgt aus (*) durch Ausmultiplizieren \\
Sei nun weiters $p_j^n(\lambda_1, \dots, \lambda_n) := \sum\limits_{i=1}^{n}\lambda_i^j$
\item [Bemerkung 2:] $\sigma_j^n, p_j^n$ sind symmetrisch, das heißt
\[
\begin{aligned}
& \sigma_j^n(\lambda_{\pi(1)}, \dots, \lambda_{\pi(n)}) =
\sigma_{j}^n (\lambda_1, \dots, \lambda_n) \\
& p_j^n(\lambda_{\pi(1)}, \dots, \lambda_{\pi(n)}) = p_{j}^n (\lambda_1, \dots, \lambda_n)
\end{aligned} \text{ für } \pi \in S_n
\]
\end{itemize}
\begin{lemma}
[Newtonidentität] \label{theo:2.2.19}
Es gilt für $k\le n$
\[k\sigma_k^n+\sum_{j=0}^{k-1}\sigma_j^n p_{k-j}^n=0\]
\end{lemma}
\begin{proof}
Induktion.
\begin{itemize}
\item [$k=n$:] Wegen
\begin{equation*}
0= \sum_{i=1}^{n} =
\sum_{i=1}^{n} \sum_{j=0}^n c_j \lambda_i^j =
\sum_{j=0}^n c_j p_j^n =
\sum_{j=0}^n \sigma_{n-j}^n p_j^n =
\sum_{j=0}^n \sigma_j^n p_{n-j}^n
\end{equation*}
folgt $\sigma_n^n p_0^n + \sum\limits_{j=0}^n \sigma_j^n p_{n-j}^n = 0$ was mit
$p_0^n = n$ die gewünschte Aussage liefert.
\item [$k<n$:] Betrachte das (symmetrische) \[
q(\lambda_1, \dots, \lambda_n) :=
k \sigma_k^n + \sum_{j=0}^{k-1} \sigma_j^n p_{k-1}^n
\]
Es gilt \[q(\lambda_1, \dots, \lambda_n) =
\sum_{j_1, \dots, j_n} c_{j_1 \dots j_n} \lambda_1^{j_1} \cdots \lambda_n^{j_n}\]
und wir müssen zeigen, dass alle Koeffizienten $c_{j_1 \dots j_n}=0$ sind.
Dazu bemerken wir, dass $c_{j_1 \dots j_n}$ immer $0$ ist,
wenn mehr als $k$ $j_i$'s ungleich $0$ sind.\\
Sei also $c_{j_1 \dots j_n}$ ein solcher Koeffizient mit $j_{k+1}, \dots, j_n=0$.
Dann gilt
\begin{align*}
& \underset{\rotatebox{90}{$=$}}
{q(\lambda_1, \dots, \lambda_k, 0, \dots, 0)} =
\sum_{j_1, \dots, j_k} c_{j_1 \dots j_n 0 \dots 0}
\lambda_1^{j_1} \cdots \lambda_n^{j_k} \\
& k \sigma_k^n + \sum_{j=0}^{k-1} \sigma_j^k p_{k-1}^k = 0
\text{ nach Voraussetzung}
\end{align*}
Aufgrund der Symmetrie gilt dasselbe Argument für alle anderen Koeffizienten
mit höchstens $k$ vielen $j_i$'s ungleich $0$.
\end{itemize}
\end{proof}
\begin{satz}
Sei $A \in \K^{\nxn}$ diagonalisierbar. Dann gilt für
\begin{align*}
\chi_A(\lambda) & = c_{n}\lambda^{n} + c_{n-1} \lambda ^{n-1} + \cdots + c_0 \\
& c_n = (-1)^n \\
& c_{n-k} = -\frac1k \sum_{j=0}^{k-1} c_{n-j} \spur(A^{k-j})
\end{align*}
\end{satz}
\begin{proof}
Folgt direkt aus Lemma \ref{theo:2.2.19} und der Bemerkung dass für $A$ diagonalisierbar \\
$\spur(A^k) = \lambda_1^k + \cdots + \lambda_n^k$ gilt.
\end{proof}
\subsubsection{Bemerkung}
$\underset{\mathrlap{\text{\dq fast alle Matrizen sind diagonalisierbar\dq}}}
{\text{Gilt}}$ auch für $A$ nicht diagonalisierbar. \dq Beweis\dq Stetigkeit
\subsubsection{Triangulierbarkeit von Matrizen}
\begin{defin}
\leavevmode
\begin{enumerate}[label=\alph*)]
\item $\alpha \in \homkv, \dim(V)=n$ heißt \underline{triangulierbar} wenn es eine Basis $B$ gibt,
sodass ${}_B M(\alpha)_B$ obere Dreiecksgestalt hat.
\item $A\in\K^{\nxn}$ heißt \underline{triangulierbar} wenn es eine reguläre Matrix $P\in\K^{\nxn}$ gibt,
mit $P^{-1} A P$ obere Dreiecksgestalt.
\end{enumerate}
\end{defin}
\begin{satz}
$A \in \K^{\nxn} / \alpha$ ist triangulierbar $\iff \chi_A / \chi_\alpha$ zerfällt in Linearfaktoren.
\end{satz}
\begin{proof}[Beweis]
\begin{itemize}
\item[$\implies$:] $\chi_A$ ist invariant
bezüglich Ähnlichkeitsumformung (Lemma \ref{theo:2.2.7}).
Sei $P^{-1} A P = \begin{pmatrix} \lambda_1 & & * \\
& \ddots & \\
0 & & \lambda_n\end{pmatrix}$
, dann folgt\\
$\chi_A(\lambda) = \chi_{P^{-1} A P}(\lambda)
= \prod\limits_{i=1}^n (\lambda_i - \lambda) $
\item[$\impliedby$:] Induktion nach $n$
\begin{itemize}
\item[$n=1$:] Jede $1\times1$ Matrix ist obere Dreiecksmatrix.
\item[$n-1\mapsto n$:] Sei $\chi_A(\lambda) =
\prod\limits_{i=1}^n (\lambda_i - \lambda)$ und sei
$b_1 \in \eig_{\lambda_1}(\alpha)$.
Sei $B=(b_1, \dots, b_n)$ Basis von $\K^n$. Dann gilt
\[\begin{aligned}
& A
= {}_B M(\alpha)_B
= \begin{pmatrix}
\lambda_1 & a_{12} & \dots & a_{1n} \\
0 & \tl & & \\
\vdots & & \tilde{A} & \\
0 & & & \br\end{pmatrix} \\
& \text{Sei }\beta: \begin{cases}
\overbrace{\langle b_2, \dots, b_n\rangle}^{V}
& \to \langle b_2, \dots, b_n\rangle \\
b_i
& \mapsto \Phi^{-1}_{\tilde{B}}(C\cdot
{}_{\tilde{B}}v)
\end{cases}
\end{aligned}\]
Es gilt $\chi_\alpha(\lambda) =
\lambda_1 - \lambda) \cdot \chi_\beta(\lambda)$,
daher zerfällt $\chi_\beta$ in Linearfaktoren.
Nach Induktionsvoraussetzung existiert eine Basis $\tilde{B} =
(\tilde{b}_2, \dots, \tilde{b}_n)$ von $\tilde{V}$ mit
\begin{equation}
{}_{\tilde{B}} M(\beta)_{\tilde{B}} =
\begin{pmatrix} \lambda_2 & & * \\
& \ddots & \\
0 & & \lambda_n\end{pmatrix}
\label{eq:2.2.22.1}
\end{equation}
Weiters ist $\alpha(b_i) = a_{1i} b_1 + \beta(b_i), i=2, \dots, n$.
Sei $\tilde{b}_i = \sum\limits_{j=2}^n \mu_{ij} b_j$.
Wegen \ref{eq:2.2.22.1} gilt
\begin{equation}
\beta(\tilde{b}_i) \in
\langle \tilde{b}_1, \dots, \tilde{b}_i \rangle
\label{eq:2.2.22.2}
\end{equation}
Wir zeigen nun, dass für die Basis $C=(c_1, \dots, c_n)$ mit
$c_1 = b_1, c_2 = \tilde{b}_2, \dots, c_n = \tilde{b}_i $
die Matrix ${}_C M(\alpha)_C$ obere Dreiecksgestalt hat.
Dies ist äquivalent zu
\[\alpha(c_i)\in \langle c_1, \dots, c_n \rangle \forall i=1, \dots, n \]
\begin{itemize}
\item [$i=1$:] $\alpha(c_1) = \alpha(b_1) = \lambda_1 b_1
\in \langle b_1 \rangle = \langle c_1 \rangle$
\item [$i>1$:]
\begin{align*}
& \alpha(c_i) = \alpha(\tilde{b}_i) =
\alpha(\sum_{j=2}^n \mu_{ij} b_j)
= \sum_{j=2}^n \mu_{ij} \alpha(b_j) \\
& = \sum_{j=2}^n\mu_{ij}(a_{1j} b_1 + \beta(b_j))
= (\underbrace{\sum_{j=2}^n \mu_{ij} a_{1j}}_
{\displaystyle\sigma_i})
+ \sum_{j=2}^n \mu_{ij}\beta(b_j) \\
& = \sigma_i b_1+ \beta(\sum_{j=2}^n \mu_{ij} b_j)
= \sigma_i b_1 + \beta(\tilde{b}_i) \\
& \underbrace{\in}_{\text{\ref{eq:2.2.22.2}}}
\langle b_1,\tilde{b}_2,\dots,\tilde{b}_i\rangle
= \langle c_1, \dots, c_i \rangle
\end{align*}
\end{itemize}
\end{itemize}
\end{itemize}
\end{proof}
\section{Jordan Normalform}
\begin{defin}
Eine $m\times m$ Matrix
\[J_m(\lambda) := \begin{pmatrix}
\lambda & 1 & 0 & \dots & 0 \\
0 & \ddots & \ddots & \ddots & \vdots \\
\vdots & \ddots & \ddots & \ddots & 0 \\
\vdots & \ddots & \ddots & \ddots & 1 \\
0 & \dots & \dots & 0 & \lambda
\end{pmatrix}\]
heißt \underline{Jordanblock} der Dimension $m$ zum Eigenwert $\lambda$.\\
Eine Matrix $A \in \K^{\nxn}$, die als Blockdiagonalmatrix aus Jordanblöcken besteht,
heißt \underline{Jordanmatrix}. \\
$A \in \K^{\nxn}$ besitzt eine \underline{Jordan-Normalform} wenn $P\in\K^{\nxn}$ invertierbar existiert,
sodass $P^{-1}AP$ Jordanmatrix ist.\\
$\alpha \in \homkv$ besitzt eine \underline{Jordan-Normalform} wenn eine Basis $B$ von $V$ existiert,
sodass $ {}_{B} M(\alpha)_{B} $ Jordanmatrix ist.\\
B heißt \underline{Jordanbasis} zu $A/\alpha$.
\end{defin}
\subsubsection{Beispiel}
\begin{itemize}
\item Jede Diagonalmatrix ist Jordanmatrix
\item $\begin{pmatrix}1\end{pmatrix},
\begin{pmatrix}1 & 1 \\ 0 & 1\end{pmatrix},
\begin{pmatrix}0 & 1 \\ 0 & 0\end{pmatrix},
\begin{pmatrix}1 & 1 & 0 \\ 0 & 1 & 0 \\ 0 & 0 & 2\end{pmatrix},
\xcancel{\begin{pmatrix} 1 & 1 \\ 0 & 2 \end{pmatrix}}$
\item \(
\begin{pmatrix} \tl3\br \\
& \tl2 & 1 \\
& & 2\br\end{pmatrix}
, \begin{pmatrix}
\tl0 & 1 \\
0 & 0\br \\
& & \tl-1\br
\end{pmatrix}\)
\end{itemize}
Wir wollen zeigen, dass $\alpha/A$ genau dann eine Jordan-Normalform besitzt, wenn $\alpha/A$ triangulierbar ist.
\subsubsection{Bemerkung}
\begin{itemize}
\item $\chi_{J_m(\lambda)}(\mu) = (\lambda - \mu)^m \implies \spec(J_m(\lambda)) = \{\lambda\}$ \\
$J_m(\lambda) - \lambda I = \begin{pmatrix}
0 & 1 & 0 & \dots & 0 \\
0 & \ddots & \ddots & \ddots & \vdots \\
\vdots & \ddots & \ddots & \ddots & \vdots \\
\vdots & & \ddots & \ddots & 1 \\
0 & \dots & \dots & 0 & 0
\end{pmatrix}$\\
$\implies \dim(\eig_{J_m(\lambda)}(\lambda)) = \dim(\ker(J_m(\lambda) - \lambda I)) = 1$ \\
$\implies m_g(\lambda) = 1$ und $m_a(\lambda) = m$.
\item $J_m(0)^m = 0$, das heißt $J_m(0)$ ist \underline{nilpotent}.
\begin{align*}
& J_m(0)(e_i): \begin{cases} e_{i-1} & i \in \{2, \dots, m\} \\
0 & \text{sonst}\end{cases} \\
& J_m(0)^l(e_i): \begin{cases}e_{i-l} & i \in \{l+1, \dots, m\} \\
0 & \text{sonst}\end{cases}
\end{align*}
\end{itemize}
\begin{defin}
$\alpha \in \homkv$ oder $A\in \K^{\nxn}$ heißt \underline{nilpotent} (mit Index $m$) falls
$\alpha^m = 0 / A^m = 0$ und $\forall l \in [m-1]: \alpha^l \neq 0 / A^l \neq 0$.
\end{defin}
\begin{lemma}
\label{theo:2.3.3}
Sei $\alpha \in \homkv, \dim(V)=n$ nilpotent mit Index $m$. Dann existiert eine Basis $B$ mit
\begin{equation*}
{}_B M(\alpha)_B =
\begin{pmatrix}
0 & \delta_1 & & \\
& \ddots & \ddots & \\
& & \ddots & \delta_{n-1} \\
& & & 0
\end{pmatrix}
\text{ und } \delta_i \in \{0, 1\} \forall i \in [n-1]
\end{equation*}
Das heißt ${}_B M(\alpha)_B$ ist blockdiagonal mit Jordanblöcken mit Eigenwerten $0$
\end{lemma}
\begin{proof}
Sei $V_i := \ker(\alpha^i)$. \\
Dies ergibt eine aufsteigende Kette von Unterräumen
\begin{equation*}
\underbrace{\{0\}}_{=V_0} \subseteq V_1 \subseteq \cdots \subseteq \underbrace{V_m}_{=V}
\end{equation*}
Wir bauen uns iterativ eine Basis für Komplemente $W_i$ mit $V_{i-1} \oplus W_i = V_i$.
Sei also $B^{m-1}$ Basis von $V_{m-1}$. \\
$C^m = (c_1^m, \dots, c_{r_{m}})$ Basis von $W_m$
[das heißt $C^m$ ergänzt die Basis $B^{m-1}$ zu Basis von $V^m$]. \\
\underline{Behauptung}
\begin{enumerate} [label=\arabic*)]
\item $\alpha(C^m) \subseteq V_{m-1}$
\item $\alpha(C^m)$ linear unabhängig
\item $\langle \alpha(C^m) \rangle \cap V_{m-2} = \{0\}$
\end{enumerate}
\begin{proof}[Zwischenbeweis]
\begin{itemize}
\item[1)] folgt aus $\alpha(V_{i+1}) \subseteq \alpha(V_i)$
\item[3)] Sei $\sum\limits_{i}\mu_i \alpha(c_i^m) \in V_{m-2}$
\begin{align*}
& \implies \alpha^{m-2}(\sum_{i}\mu_i \alpha(c_i^m)) = 0 \\
& \implies \alpha^{m-1} (\sum_{i} \mu_i \alpha(c_i^m)) = 0 \\
& \implies \sum \mu_i c_i^m \in V_{m-1} \\
& \underbrace{\implies}_{\mathclap{\substack{(c_i^m) \text{ liegen} \\
\text{im Komplement} \\
\text{von } V_{m-1}}}}
\mu_i = 0, \forall i \implies \sum_{i} \mu_i \alpha(c_i^m) = 0
\end{align*}
\item[2)] folgt aus 3) [da $0\in V_{m-2}$]
\end{itemize}
\end{proof}
Es folgt, dass
\[
\underbrace{V_{m-2} \oplus \langle \alpha(C^m) \rangle \oplus
\overset{\langle D^{m-1} \rangle}{\langle C^{m-1} \rangle}}_{V_{m-1}} \oplus
\overset{\langle D^m \rangle}{\langle C^m \rangle} = V
\]
Setze $D^m := C^m$ und definiere induktiv für $D^i \subseteq V_i$ die Menge
$D^{i-1} := \alpha(D^i) \cup C^{i-1} \subseteq V_{i-1}$ sodass mit einer Basis $B^{i-2}$ von
$V_{i-2}$ die Menge $B^{i-2} \cup D^{i-1}$ Basis von $V_{i-1}$ ist, also
\[
V_{i-2} \oplus \underbrace{\langle \alpha(D^i) \rangle \oplus \langle C^{i-1} \rangle}_
{\langle D^i \rangle}
= V_{i-1} \text{$\leftarrow$ das geht nach obiger Behauptung}
\]
Nach Konstruktion ist $(D^1, \dots, D^m)$ Basis von $V$.
Sie besteht aus folgenden Elementen:
\begin{align*}
\left.
\begin{array}{lll}
J_m(0) \to & \alpha^{m-1}(d_1^m), \dots, \alpha(d_1^m), & d_1^m \\
& & \vdots \\
J_m(0) \to & \alpha^{m-1}(d_{r_m}^m), \dots, \alpha(d_{r_m}^m), & d_{r_m}^m
\end{array}
\right\} \in V_m \\
\left.
\begin{array}{lll}
J_{m-1}(0) \to & \alpha^{m-1}(d_1^{m-1}), \dots, \alpha(d_1^{m-1}), & d_1^{m-1} \\
& & \vdots \\
J_{m-1}(0) \to & \alpha^{m-1}(d_{r_{m-1}}^{m-1}), \dots, \alpha(d_{r_{m-1}}^{m-1}), & d_{r_{m-1}}^{m-1}
\end{array}
\right\} \in V_{m-1} \\
\left. \begin{array}{lr}
J_1(0) \to & d_1^1 \\
& \vdots \\
J_1(0) \to & d_{r_1}^1
\end{array}
\right\} V_1 = \ker(\alpha)
\end{align*}
Wenn wir die Basiselemente von links nach rechts und von oben nach unten ordnen erhalten wir
die gewünschte Gestalt.
\end{proof}
\subsubsection{Bemerkung}
Angenommen \(\alpha - \lambda \id: V \to V\) nilpotent. Dann besitzt \(\alpha\) nach Lemma
\ref{theo:2.3.3} Jordan-Normalform.
\begin{defin}
\label{theo:2.3.4}
Sei \(V \K\)-Vektorraum, \(\dim(V) < \infty, \alpha \in \homkv\) und \(\lambda \in \spec(\alpha)\).
Für \(l \in \mathbb{N}\) definiere \(V_{l, \lambda}:= \ker((\alpha - \lambda \id)^l)\)
\end{defin}
\subsubsection{Bemerkung}
\begin{itemize}
\item $\alpha - \lambda \id|_{V_{l, \lambda}} \in \homk(V_{l, \lambda}, V_{l, \lambda})$:
\begin{align*}
\text{zu Zeigen: } v\in V_{l, \lambda} & \implies \alpha(v) - \lambda v \in V_{l, \lambda}\text{, das heißt} \\
(\alpha - \lambda \id)^l v = 0 & \implies (\alpha - \lambda \id)^{l-1} (\alpha - \lambda \id) v = 0 \\
& \implies (\alpha - \lambda \id)(v) \in V_{l, \lambda} & \square
\end{align*}
\item Nach Lemma \ref{theo:2.3.3} gibt es also Basis von $V_{l, \lambda}$ bezüglich derer
$\alpha - \lambda \id |_{V_{l, \lambda}}: V_{l, \lambda} \to V_{l, \lambda}$
Jordan-Normalform hat
\end{itemize}
\begin{lemma}
\label{theo:2.3.5}
Sei $V \K$-Vektorraum, $\dim(V) < \infty, \alpha \in \homkv$. Für $l\in\mathbb{N}$ sei
$V_l := \ker(\alpha^l)$. Dann gilt $\alpha(V_l) \subseteq V_{l-1} \subseteq V_l$ für alle
$l\in \mathbb{N}$ und es existiert genau ein $k\in \mathbb{N}_0$ mit
\[
\{0\} = V_0 \subseteq V_1 \subseteq \cdots \subseteq V_k = V_{k+1} \text{ und } V_{l+1} =
V_l, \forall l \ge k
\]
\end{lemma}
\begin{proof}
Da $\dim(V) < \infty$ muss es ein kleinstes $k$ mit $V_{k+1} = V_{k}$ geben.
Angenommen $\exists l\ge k$ mit $V_{l+1} \neq V_l$. Sei $0\neq v\in V_{l+1} \setminus V_l$
$\implies 0 = \alpha^{l+1}(v) = \alpha^{k+1}(\alpha^{l-k}(v))$ und $0\neq \alpha^l(v)
= \alpha^k (\alpha^{l-k}(v)) \implies 0\neq \alpha^{l-k}(v) \in V_{k+1}\setminus V_k$
\Lightning
\end{proof}
\begin{defin}
Sei $V_{l, \lambda}$ wie in Definition \ref{theo:2.3.4} und $k$ wie in Lemma \ref{theo:2.3.5}
Dann heißt \[
\widetilde{\eig_\alpha(\lambda)} := V_{k, \lambda} = V_{k+1, \lambda}
\]
\underline{verallgemeinerter Eigenraum} oder \underline{Hauptraum} von $\alpha$ zum Eigenwert
$\lambda$. $v \in V_{l, \lambda} \setminus V_{l-1, \lambda}$ für $1 \le l \le k$ heißt
\underline{verallgemeinerter Eigenvektor} der Ordnung $l$.
\end{defin}
\subsubsection{Idee}
\begin{itemize}
\item $\alpha|_{\widetilde{\eig_\alpha(\lambda)}}: \widetilde{\eig_\alpha(\lambda)} \to
\widetilde{\eig_\alpha(\lambda)}$
hat Jordan-Normalform.
Zerlege \begin{equation}
\label{eq:2.3.6.1}
V:= \widetilde{\eig_\alpha(\lambda_1)} \oplus \cdots \oplus \widetilde{\eig_\alpha(\lambda_r)}
\end{equation}
dann besitzt ganz $\alpha: V\to V$ Jordan-Normalform
\item Sei $V= V_1 \oplus \cdots \oplus V_r$ und $\alpha \in \homkv$. Falls $\alpha(V_i) \subseteq V_i$ für alle
$i \in [r]$, dann schreiben wir $\alpha = \alpha_1 \oplus \cdots \oplus \alpha_r$ mit
$\alpha_i = \alpha|_{V_i} \forall i \in [r]$.
Für $v= v_1 + \cdots + v_r, v_i \in V_i, \forall i \in [r]$ gilt also
$\alpha(v) = \alpha_1(v_1) + \cdots + \alpha_r(v_r)$.
Sei $B_i = \{b_1^i, \dots, b_{d_i}^i\}$ Basis von $V_i$ und $B = (B_1, \dots, B_r)$.
Dann hat ${}_B M(\alpha)_B$ Blockdiagonalgestalt mit Blöcken ${}_{B_i} M(\alpha_i)_{B_i}$, das heißt
\[
{}_B M(\alpha)_B = \begin{pmatrix}
\overbrace{{}_{B_1} M(\alpha_1)_{B_1}}^{\in \K^{d_1 \times d_1}} & & 0 \\
& \ddots & \\
0 & & \underbrace{{}_{B_r} M(\alpha_r)_{B_r}}_{\in\K^{d_r \times d_r}}
\end{pmatrix}
\]
Insbesondere gilt $\chi_\alpha = \chi_{\alpha_1} \cdot \dots \cdots \chi_{\alpha_r}$
\item Da wir schon wissen, dass $\alpha|_{\widetilde{\eig_\alpha(\lambda_i)}}$ Jordan-Normalform hat folgt \\
Jordan-Normalform für $\alpha$ wenn \ref{eq:2.3.6.1} gezeigt werden kann.
\end{itemize}
\begin{satz}
\label{theo:2.3.7}
Sei $V \K$-Vektorraum mit $\dim(V) < \infty$ und $\alpha \in \homkv$ sodass
$\chi_\alpha(\lambda) = (\lambda_1 - \lambda) \cdots (\lambda_r - \lambda)$ in Linearfaktoren
zerfällt. Dann gilt
$V = \widetilde{\eig_\alpha(\lambda_1)} \oplus \cdots \oplus \widetilde{\eig_\alpha(\lambda_r)}$
und insbesondere $\alpha = \alpha_1 \oplus \cdots \oplus \alpha_r$ mit
$\alpha_i := \alpha|_{\widetilde{\eig_\alpha(\lambda_i)}} \in \homk(\widetilde{\eig_\alpha(\lambda_i)},
\widetilde{\eig_\alpha(\lambda_i)})$
\end{satz}
\begin{proof}
Induktion nach $\dim(V)$.
\begin{itemize}
\item[$n=1$:] \checkmark
\item[$n-1 \mapsto n$:] Da $\chi_A$ in Linearfaktoren zerfällt besitzt es eine Nullstelle
$\lambda \in \spec(\alpha)$.
\begin{enumerate}[label=Fall \arabic*:]
\item $\widetilde{\eig_\alpha(\lambda)} = V$ \\
\underline{Behauptung:} $\spec(\alpha) = \{\lambda\}$
\begin{proof}[Zwischenbeweis]
Angenommen $\lambda' \neq \lambda$ und $\lambda' \in \spec(\alpha)$ und
$v\in \eig_\alpha(\lambda')$. \\
$\implies (\alpha - \lambda \id) (v) = \alpha(v) - \lambda'v + (\lambda' - \lambda) v
= (\lambda' - \lambda)(v)$ \\
$\implies (\alpha - \lambda \id)^l (v) \neq 0,\forall l \in \mathbb{N}$\Lightning \\
Daraus folgt das gewünschte Resultat
\end{proof}
\item $\widetilde{\eig_\alpha(\lambda)} \neq V$. Sei $k$ minimal mit
$\ker(\alpha - \lambda - \id)^k = \widetilde{\eig_\alpha(\lambda)}$ [Lemma \ref{theo:2.3.5}]
Setze $V_1 := \widetilde{\eig_\alpha(\lambda)}, V_2 := \im(\alpha - \lambda \id)^k$. \\
\underline{Behauptung:}
\begin{enumerate}[label=(\roman*)]
\item $\alpha(V_i) \subseteq V_i, i \in \{1, 2\}$
\item $V = V_1 \oplus V_2$
\end{enumerate}
\begin{proof}[Zwischenbeweis]
\begin{enumerate}[label=(\roman*)]
\item Wir zeigen $(\alpha - \lambda \id)(V_i) \subseteq V_i$.
\begin{itemize}
\item[$i=1$:] Sei $v\in V_1 = \ker(\alpha - \lambda \id)^k$. Dann gilt klarerweise
$(\alpha - \lambda \id)(v) \in \ker(\alpha - \lambda \id)^k \checkmark$
\item[$i=2$:] Sei $v \in \im(\alpha - \lambda \id)^k$, also
$v = (\alpha - \lambda \id)^k (w)$
$\implies (\alpha - \lambda \id)(v)
= (\alpha - \lambda \id)^k (\alpha - \lambda \id)(w) \in
\im(\alpha - \lambda \id)^k \checkmark$
\end{itemize}
\item Es gilt $\dim(V) = \dim(V_1) + \dim(V_2)$ nach der Dimensionsformel.
Es genügt also zu zeigen, dass
$V_1 \cap V_2 = \{0\}$. \\ Sei $v\in V_1 \cap V_2$
\begin{align*}
& \underbrace{\implies}_{v\in V_2} \exists w\in V: v = (\alpha - \lambda \id)^k(w) \\
& \underbrace{\implies}_{v\in V_1} (\alpha - \lambda \id)^{2k}(w) = 0 \\
& \implies w \in V_{2k, \lambda} \setminus V_{k, \lambda}
\underbrace{\implies}_{\mathclap{\text{Lemma \ref{theo:2.3.5}}}} w = \{0\}\checkmark
\end{align*}
\end{enumerate}
\end{proof}
\end{enumerate}
Es folgt $V = \underbrace{\widetilde{\eig(\lambda)}}_{V_1} \oplus V_2, \dim(V_2) < n$ und \\
$\alpha = \alpha_1 \oplus \alpha_2, \alpha_i := \alpha|_{V_i}, i\in\{1, 2\}$.
Es folgt $\chi_\alpha = \chi_{\alpha_1} \cdot \chi_{\alpha_2}$, also zerfällt $\chi_{\alpha_2}$
in Linearfaktoren. Daher können wir die Induktionsvorraussetzung anwenden,
was das gewünschte Resultat lierfert.
\end{itemize}
\end{proof}
\begin{satz}
Sei $V \K$-Vektorraum, $\dim(V) < \infty$ und $\alpha \in \homkv$ sodass $\chi_A$ in
Linearfaktoren zerfällt. Dann besitzt $\alpha$ Jordan-Normalform.
\end{satz}
\begin{proof}
Zerlege nach Satz \ref{theo:2.3.7}
$V = \widetilde{\eig_\alpha(\lambda_1)} \oplus \cdots \oplus \widetilde{\eig_\alpha(\lambda_r)}$
und \\
$\alpha = \alpha \oplus \cdots \oplus \alpha_r$.
Da $\widetilde{\eig_\alpha(\lambda_i)} = \ker(\alpha - \lambda_i \id)^{k_i}$ ist
$\alpha_i - \lambda_i \id := \alpha|_{\widetilde{\eig_\alpha(\lambda_i)}} - \lambda
\id|_{\widetilde{\eig_\alpha(\lambda_i)}} $ nilpotent. Nach Lemma \ref{theo:2.3.3} gibt es eine Basis
$B_i$ von $\widetilde{\eig_\alpha(\lambda_i)}$ sodass ${}_{B_i} M(\alpha_i)_{B_i}$ Jordan-Normalform
hat. Es folgt mit $B= (B_1, \dots, B_r)$ dass ${}_B M(\alpha)_B = \begin{pmatrix}
{}_{B_1} M(\alpha_1)_{B_1} & & \\
& \ddots & \\
& & {}_{B_r}M(\alpha_r)_{B_r}
\end{pmatrix}$ Jordanmatrix ist.
\end{proof}
\subsubsection{Berechnung der Jordan-Normalform}
\begin{enumerate}
\item Berechne $\spec(\alpha) = \{ \lambda_1, \dots, \lambda_r \}$
\item
\begin{enumerate}[label=\alph*)]
\item Haupträume berechnen: Finde $k$ minimal mit \[
\ker(\alpha - \lambda \id)^{k+1} = \ker(\alpha - \lambda \id)^k =: V_\lambda
\]
\item Für $1 \le l \le k$ bestimme $B_l = \{ b_1^l, \dots, b_{r_l}^l\}$, sodass $(B_1, \dots, B_l)$
Basis von $\ker(\alpha - \lambda \id)^l$.
\end{enumerate}
\item
\begin{enumerate}[label=\alph*)]
\item Setze zunächst $v_i^k = b_i^k, i = 1, \dots, r_k$. $D_k := (v_1^k, \dots, v_{r_k}^k)$ \\
Setze $v_i^{k-1} := (\alpha - \lambda \id)(v_i^k) \in \langle B_{k-1} \rangle,
i = 1, \dots, r_k$ \\
Ergänze gegebenenfalls $(v_1^{k-1}, \dots, v_{r_k}^{k-1},
v_{r_{k+1}}^{k-1}, \dots, v_{r_{k-1}}^{k-1})=:D_{k-1}$, sodass
$\langle D_{k-1} \rangle = \langle B_{k-1} \rangle$
\item Führe 3a) iterativ aus. \\
Setze $v_i^{l-1} := (\alpha - \lambda \id)(v_i^l), i = 1, \dots, r_l$ \\
Ergänze gegebenenfalls
$v_1^{l-1}, \dots, v_{r_l}^{l-1}, v_{r_{l+1}}^{l-1}, \dots, v_{r_{l-1}}^{l-1} =:D_{l-1}$,
sodass $\langle D_{l-1} \rangle = \langle B_{l-1} \rangle$
\end{enumerate}
\item Sei $B_\lambda = (D_1, \dots, D_k) \implies {}_{B_\lambda} M(\alpha|_{v_\lambda})_{B_\lambda}$ hat
Jordan-Normalform mit Eigenwert $\lambda$.
\item Setze $B = (B_{\lambda_1}, \dots, B_{\lambda_r}) \implies {}_B M(\alpha)_B$ hat Jordan-Normalform.
\end{enumerate}
\subsubsection{Beispiel}
\begin{align*}
& A= \begin{pmatrix}
1 & 0 & 2 & 3 & 4 \\
0 & 1 & 0 & -2 & -3 \\
0 & 0 & 1 & 0 & 2 \\
0 & 0 & 0 & 1 & -1 \\
0 & 0 & 0 & 0 & 1
\end{pmatrix}
, \chi_A(\lambda) = (\lambda - 1)^5 \\
& (A - 1\cdot I) = \begin{pmatrix}
0 & 0 & 2 & 3 & 4 \\
0 & 0 & 0 & -2 & -3 \\
0 & 0 & 0 & 0 & 2 \\
0 & 0 & 0 & 0 & -1 \\
0 & 0 & 0 & 0 & 0
\end{pmatrix}
\implies \ker(A - I) = \langle ( \underbrace{e_1, e_2}_{B_1} ) \rangle \\
& (A-I)^2 = \begin{pmatrix}
0 & 0 & 0 & 0 & 1 \\
0 & 0 & 0 & 0 & 2 \\
0 & 0 & 0 & 0 & 0 \\
0 & 0 & 0 & 0 & 0 \\
0 & 0 & 0 & 0 & 0
\end{pmatrix}
\implies \ker((A-I)^2) = \langle (\underbrace{e_1, e_2}_{B_1}, \underbrace{e_3, e_4}_{B_2}) \rangle \\
& (A-I)^3 = 0 \implies \ker((A-I)^3) =
\langle(\underbrace{e_1, e_2}_{B_1}, \underbrace{e_3, e_4}_{B_2}, \underbrace{e_5}_{B_3}) \rangle \\
& B_1 = (e_1, e_2), B_2 = (e_3, e_4), B_3 = (e_5)
\end{align*}
\begin{align*}
\begin{rcases}
v_1^3 = e_5
\end{rcases}
D_3 \\
\begin{rcases}
v_1^2 = (A-1I)(v_1^3) = \begin{pmatrix}-4 \\ -3 \\ 2 \\ -1 \\ 0\end{pmatrix} \\
v_2^2 = e_4
\end{rcases}
D_2 \\
\begin{rcases}
v_1^1 = (A - I)(v_1^2) = \begin{pmatrix}1 \\ 2 \\ 0 \\ 0 \\ 0 \end{pmatrix} \\
v_2^1 = (A - I)(v_2^2) = \begin{pmatrix}3 \\ -2 \\ 0 \\ 0 \\ 0 \end{pmatrix}
\end{rcases}
D_1
\end{align*}
\begin{align*}
(\overset{\mathrlap{\rotatebox{30}{\scriptsize$\in\ker(A-I)$}}}{v_1^1}
\underset{\mathclap{\substack{\rotatebox{180}{$\curvearrowright$} \\ A-I}}}{,}
v_1^2
\underset{\mathclap{\substack{\rotatebox{180}{$\curvearrowright$} \\ A-I}}}{,}
v_1^3,
\overset{\mathrlap{\rotatebox{30}{\scriptsize$\in\ker(A-I)$}}}{v_1^1}
\underset{\mathclap{\substack{\rotatebox{180}{$\curvearrowright$} \\ A-I}}}{,}
v_2^2) = B, {}_B M(A)_B =
\begin{pmatrix}
1 & 1 & 0 & 0 & 0 \\
0 & 1 & 1 & 0 & 0 \\
0 & 0 & 1 & 0 & 0 \\
0 & 0 & 0 & 1 & 1 \\
0 & 0 & 0 & 0 & 1
\end{pmatrix} \\
P =
\begin{pmatrix}
1 & 4 & 0 & 3 & 0 \\
2 & -3 & 0 & -2 & 0 \\
0 & 2 & 0 & 0 & 0 \\
0 & 1 & 0 & 0 & 1 \\
0 & 0 & 1 & 0 & 0
\end{pmatrix}
\implies P^{-1} A P =
\begin{pmatrix}
\tl1 & 1 & 0 & 0 & 0 \\
0 & 1 & 1 & 0 & 0 \\
0 & 0 & 1\br & 0 & 0 \\
0 & 0 & 0 & \tl1 & 1 \\
0 & 0 & 0 & 0 & 1\br
\end{pmatrix}
\end{align*}
\chapter{Euklidische und Unitäre Vektorräume}
\subsubsection{Motivation}
Wir wollen Geometrie betreiben und Längen beziehungsweise Winkel messen können.
\subsubsection{Länge}
\begin{tikzpicture}[scale=4]
\draw [-latex, very thick] (0, 0) -- (1.3, 1);
\draw [dashed] (0, 0) -- (1.3, 0) -- (1.3, 1);
\node [below] at (0.65, 0) {$x_2 - x_1$};
\node [right] at (1.3, 0.5) {$y_2 - y_1$};
\node [below left] at (0, 0) {$(x_1, y_1)$};
\node [above right] at (1.3, 1) {$(x_2, y_2)$};
\draw (1.1, 0) -- (1.1, 0.2) -- (1.3, 0.2);
\draw [fill] (0, 0) circle [radius=0.02];
\end{tikzpicture}
\( \R^2: P_1 = (x_1, y_1), P_2 = (x_2, y_2) \) \\
\( d(P_1, P_2) = \sqrt{(x_2 - x_1)^2 + (y_2 - y_1)^2} = \lvert \vect{P_1P_2} \rvert \)
\subsubsection{Winkel}
\begin{tikzpicture}[scale=0.7]
\coordinate (a) at (0, 0);
\coordinate (b) at (5, 6);
\coordinate (c) at (8, 4);
\draw [fill] (a) circle [radius=0.07];
\draw [very thick, ->] (a) -- (b);
\draw [very thick, ->] (a) -- (c);
\node [below left] at (a) {$p$};
\node [right] at (b) {$v_2$};
\node [right] at (c) {$v_2$};
\draw pic [draw, thick, angle radius=3cm, pic text=$\alpha$] {angle=c--a--b};
\end{tikzpicture} \\
$v_1 = (u_1, w_1), v_2 = (u_2, w_2), v= (u, w)$ \\
$\cos(\alpha) = \dfrac{u_1 u_2 + w_1 w_2}{\lvert w_1 \rvert \lvert w_2 \rvert}$
$\lvert v \rvert = \sqrt{u^2 + w^2}$ \\
$v_1 \cdot v_2 = u_1 u_2 + w_1 w_2$ skalares Produkt \\
$\implies d(P_1, P_2) = \sqrt{\vect{P_1 P_2} \cdot \vect{P_1 P_2}}, \cos(\sphericalangle{v_1 v_2}) =
\dfrac{v_1 v_2}{\lvert v_1 \rvert \lvert v_2 \rvert}$
\section[Skalarprodukte und Hermitesche Formen]{Skalarprodukte und Hermitesche \\ Formen}
Zunächst sei \( \K = \R \)
\begin{defin}
Sei $V$ ein $\R$-Vektorraum und $\beta: V \times V \to \R$.
$\beta$ heißt
\begin{itemize}
\item \underline{bilinear} (Bilinearform) wenn $\forall u, v, w\in V, \lambda \in \R$:
\begin{align*}
\beta(u+v, w) = \beta(u, w) + \beta(v, w), \\
\beta(u, v+w)=\beta(u, v) + \beta(u, w), \\
\beta(\lambda u, v) = \lambda \beta(u, v) = \beta(u, \lambda v)
\end{align*}
\item \underline{symmetrisch} wenn $\forall u, v \in V: \beta(u, v) = \beta(v, u)$
\item \underline{positiv definit} wenn $\forall v \in V\setminus\{0\}: \beta(v, v) > 0$
\item \underline{skalares Produkt} wenn $\beta$ symmetrisch, positiv definit (spd) und bilinear ist.
\end{itemize}
\end{defin}
\subsubsection{Bemerkung}
$v=0\in V \implies 0 \cdot v = v \implies \beta(v, v) = \beta(0 \cdot v, v) = 0 \cdot \beta(v, v) = 0$
\subsubsection{Beispiele}
\begin{itemize}
\item Sei $V = \R^n$ und $v= (v_1, \dots, v_n), w= (w_1, \dots, w_n)$ \\
$\beta_1(v, w) = \sum\limits_{i=1}^n v_i w_i= v^Tw$ ist symmetrische positiv definite Bilinearform.
\item Sei $\dim(V) = n$ und $B=(b_1, \dots, b_n)$ Basis \\
Sei für $v, w \in V: \begin{cases} {}_B \Phi(v) = (v_1, \dots, v_n) \\
{}_B \Phi(w) = (w_1, \dots, w_n)\end{cases}$ \\
$\beta_2(v, w) = \sum\limits_{i=1}^n v_i w_i = \beta_1({}_B \Phi(v), {}_B \Phi(w))$ ist spd.
\item $V= \R^2, v= \begin{pmatrix}v_1\\v_2\end{pmatrix},
w= \begin{pmatrix}w_1\\w_2\end{pmatrix}, A = \begin{pmatrix} 4 & -2 \\ -2 & 3 \end{pmatrix}$ \\
$\beta_3(v, w) = v^T A w \in \R$ \\
symmetrisch, weil
\[
\beta_3(v, w) = \beta_3(v, w)^T = (v^T A w)^T = w^T A^T v = w^T A v = \beta_3(w, v) \checkmark
\]
$\beta_3(u, v) = 4v_1w_1 - 2v_1w_2 - 2v_2w_1 + 3v_2 w_2 \implies
\beta(v, v) = (2 v_1 - v_2)^2 + 2 v_2^2 = 0
\implies v_2 = 0 \implies (2v_1)^2 = 0 \implies v_1 = 0$
\item Sei $a, b \in \R, a < b, V = \{f:[a, b] \to \R: f \text{ stetig}\}$ \\
Sei $h \in V: h(t) > 0 \forall t \in [a, b]$
\begin{align*}
& \beta_4(f, g) = \int_a^b f(t) g(t) h(t) dt \text{ bilinear, symmetrisch} \\
& \beta_4(f, f) = \int_a^b \lvert f(t) \rvert ^2 h(t) dt = 0 \implies f= 0
\end{align*}
\end{itemize}
\begin{defin}
Ein Vektorraum mit skalarem Produkt heißt \underline{Euklidischer Raum}.\\
Man schreibt oft $u \cdot v, \inner uv$ anstatt $\beta(u, v)$.
\end{defin}
Nun sei $\K = \C$
\begin{defin}
Sei $V$ ein \C-Vektorraum und $\beta: V \times V \to \C$. $\beta$ heißt \underline{hermitesche Form} wenn für
alle $u, v, w \in V, \lambda \in \C$:
\begin{enumerate}[label=\roman*)]
\item $\beta(u + v, w) = \beta(u, w) + \beta(v, w)$
\item $\beta(\lambda u, v) = \lambda \beta(u, v)$
\item $\beta(u, v) = \overline{\beta(u, v)}$
\end{enumerate}
\end{defin}
\begin{lemma}
\label{theo:3.1.4}
Sei $\beta$ hermitesche Form
\begin{enumerate}[label=\alph*)]
\item $\beta(u, v + w) = \beta(u, v) + \beta(u, w)$
\item $\beta(u, \lambda v) = \overline{\lambda} \beta(u, v)$
\item $\beta(u, u) \in \R$
\end{enumerate}
\end{lemma}
\begin{proof}
\begin{enumerate}[label=\alph*)]
\item $\beta(u, v+w) \overset{\text{iii}}{=} \overline{\beta(v+ w, u)} \overset{\text{i}}{=}
\overline{\beta(v, u)} + \overline{\beta(w, u)} \overset{\text{iii}}{=}\beta(u, v) + \beta(u, w) \checkmark$
\item $\beta(u, \lambda v) \overset{\text{iii}}{=} \overline{\beta(\lambda v, u)} \overset{\text{ii}}{=}
\overline{\lambda}\cdot\overline{\beta(v, u)} \overset{\text{iii}}{=} \overline{\lambda} \beta(u, v)$
\item $z = \overline{z} \iff z \in \R$ \\
$\beta(u, u) \overset{\text{iii}}{=} \overline{\beta(u, u)} \implies \beta(u, u) \in \R$
\end{enumerate}
\end{proof}
\begin{defin}
Sei $\beta$ hermitesche Form.
\begin{itemize}
\item $\beta$ heißt \underline{positiv definit} wenn $\forall v \in V\setminus\{0\}:
\underbrace{\beta(v, v)}_{\in\R} > 0$
\item Eine positiv definite hermitesche Form heißt \underline{skalares Produkt}
\item Ein komplexer Vektorraum mit einem skalaren Produkt heißt \underline{unitärer} \underline{Raum}.
\end{itemize}
\end{defin}
\subsubsection{Beispiel}
$V = \C^n, u = (u_1, \dots, u_n), v = (v_1, \dots, v_n)$ \\
$u \cdot v = \sum\limits_{i=1}^n u_i \overline{v_i}$ ist skalares Produkt
\par
Wir zeigen nun, dass jeder euklidische Vektorraum in einen unitären Vektorraum eingebettet werden kann.
\begin{defin}
Sei $V$ ein \R-Vektorraum.
\begin{align*}
& V_\C := \{ (u, v): u, v\in V \} \text{
[Schreibe $(u, v) =: u + \overset{\mathclap{\substack{i^2 = -1 \\ |}}}{i} \cdot v$]} \\
& (u_1, v_1) + (u_2, v_2) := (u_1 + u_2, v_1 + v_2) \text{ Addition} \\
& \lambda = (\gamma + i \delta) \in \C, \lambda \cdot (u, v) = (\gamma u - \delta v, \delta u + \gamma v)
\text{ skalare Multiplikation} \\
& \lambda(u + iv) = (\gamma + i \delta) (u + iv) = \gamma u + i \gamma v + i \delta u - \delta v \\
& \; \; =(\gamma u - \delta v) + i (\gamma v + \delta u) \\
& \implies (V_\C, +, \cdot) \text{ ist \C-Vektorraum}
\end{align*}
$V_\C$ heißt die \underline{komplexe Erweiterung} von V.
\end{defin}
\[
\text{Einbettung: }\iota: \begin{cases}V \to V_\C \\ v \mapsto (v, 0) = v + i\cdot 0 \end{cases}
\]
\begin{lemma}
\leavevmode
\begin{enumerate}[label=\alph*)]
\item $V$ ist durch die Einbettung $v \overset{\iota_V}{\mapsto} (v, 0)$ \dq in $V_\C$ enthalten\dq, das heißt
$\iota_V$ ist injektiv.
\item Seien $V, W$ \R-Vektorräume, $\alpha \in \Hom_\R(V, W)$. Dann existiert eine eindeutige komplexe
Erweiterung $\alpha_\C \in \Hom_\C(V_\C, W_\C)$ mit
\[
\forall v \in V: \alpha_\C(\iota_V(v)) = \iota_W(\alpha(v))
\]
\end{enumerate}
\end{lemma}
\begin{proof}
\begin{enumerate}[label=\alph*)]
\item $\iota_V$ ist linear \checkmark \\
$\iota_V(v) = (0, 0) \implies v = 0$ (injektiv)
\item Sei $\alpha_\C$ so eine Fortsetzung \\
$\alpha_\C(u + iv) = \alpha_\C(u) + i \alpha_\C(v) = \alpha(u) + i\alpha(v)$ \\
$\alpha_\C((u, v)) = (\alpha(u), \alpha(v))$ Dadurch ist $\alpha_\C$ eindeutig bestimmt!
\end{enumerate}
\end{proof}
\begin{defin}
$\alpha_\C$ heißt die komplexe Forsetzung von $\alpha$
\end{defin}
Auch skalare Produkte können eindeutig fortgesetzt werden.
\begin{satz}
Sei $(V, \beta)$ euklidischer \R-Vektorraum. Dann existiert genau eine hermitesche Form $\beta_\C$ auf $V_\C$,
welche $\beta$ fortsetzt:
\[
\forall v, w \in V: \beta_\C(\iota_V(v), \iota_V(w)) = \beta(v, w)
\]
\end{satz}
\begin{proof}
Ein solches $\beta_\C$ muss erfüllen, dass
\begin{align*}
\beta_\C(u_1 + i v_1, u_2 + i v_2) & = \beta_\C(u_1, u_2 + i v_2) + i \beta_\C(v_1, u_2+iv_2) \\
& = \beta_\C(u_1, u_2) + i \beta_\C(v_1, u_2)
\underset{\mathclap{\substack{| \\\text{\ref{theo:3.1.4} b)}}}}{-}
i \beta_\C(u_1, v_2) + \beta_\C(v_1, v_2) \\
& = \beta(u_1, u_2) + \beta(v_1, v_2) + i(\beta(v_1, u_2) - \beta(u_1, v_2))
\end{align*}
und dadurch ist $\beta_\C$ eindeutig bestimmt.
\end{proof}
\begin{satz}
[Cauchy-Schwarz]
\label{theo:3.1.10}
Für $u, v$ in einem euklidischen/unitären Vektorraum $V$ gilt
\[
\lvert \inner uv \rvert ^2 \le \inner uu \inner vv
\]
Gleichheit gilt genau wenn $u, v$ linear abhängig sind.
\end{satz}
\begin{proof}
$v=0 \checkmark$ \\
$v \neq 0 \implies \inner vv >0$ \\
Sei $\lambda \in \C \implies$
\begin{align*}
0 & \le \inner{u - \lambda v}{ u - \lambda v } \\
& = \inner u {u - \lambda v} - \lambda \inner v {u-\lambda v} \\
& = \inner uu - \overline{\lambda} \inner uv - \lambda \inner vu +
\underbrace{\lambda \overline{\lambda}}_{=\lvert\lambda\rvert^2} \inner vv
\end{align*}
Sei $\lambda := \dfrac{\inner uv}{\inner vv}, \overline{\lambda} =
\dfrac{\overline{\inner uv }}{\overline{\inner vv }}
=\dfrac{\inner vu}{\inner vv}$, so folgt
\begin{align*}
0 & \le \inner uu - \frac{\inner vu \inner uv }{\inner vv }
- \frac{\inner uv \inner vu }{\inner vv } +
\frac{\cancel{\inner vv } \inner uv \inner vu }
{\inner{v}{v}^{\cancel{2}}} \\
& = \inner uu - \frac{\lvert \inner uv \rvert^2}{\inner vv } \\
& \implies 0 \le \inner uu \inner vv - \lvert \inner uv \rvert ^2 \\
& \implies \inner uu \inner vv \ge \lvert \inner uv \rvert^2.
\end{align*}
Gleichheit gilt, wenn $\inner{u - \lambda v}{u - \lambda v} = 0$, also $u, v$ linear abhängig.
\end{proof}
\begin{defin}
Man nennt
\begin{itemize}
\item $\norm v := \sqrt{\inner vv }$ die \underline{Länge} oder die \underline{Norm} von
$v \in V$.
\item $\cos(\sphericalangle v w) := \dfrac{\inner vw }{\norm v \norm w }$ der
Kosinus des \underline{Winkels} zwischen $v, w \in V$. \\
(Wegen Satz \ref{theo:3.1.10} ist $\cos(\sphericalangle v w) \le 1$ und damit auch $\sphericalangle v w$
wohldefiniert!)
\item $v\in V$ heißt \underline{normiert} wenn $\norm v = 1$
\end{itemize}
\end{defin}
\begin{satz}
$\norm \cdot $ ist eine \underline{Norm}, das heißt
\begin{enumerate}[label=\alph*)]
\item $\norm v \ge 0$
\item $\norm v = 0 \implies v = 0$
\item $\norm {\lambda v} = \lvert \lambda \rvert \norm v $
\item $\norm {v + w} \le \norm v + \norm w $ (Dreiecksungleichung)
\end{enumerate}
\end{satz}
\begin{proof}
\begin{enumerate}[label=\alph*)]
\item $\norm v = (\underbrace{\inner vv }_{\in [0, \infty)})^\frac12 \ge 0$
\item $\norm v = 0 \implies \norm{v}^2 = 0 \implies \inner vv = 0 \implies v = 0$
\item $\norm{\lambda v}^2 = \inner{\lambda v}{\lambda v}= \norm{\lambda}^2
\inner vv = \lvert \lambda \rvert^2 \norm{v}^2$
\item \begin{align*}
\norm{u + v}^2 & = \inner{u + v}{u +v} \\
& = \inner u{u+v} + \inner v{u+v} = \inner uu + \inner uv +
\inner vu + \inner vv \\
& = \inner uu + \inner uv + \overline{\inner uv } + \inner vv \\
& = \inner uu + 2 \Re(\inner uv ) + \inner vv \\
& \le \inner uu + 2 \lvert \inner uv \rvert + \inner vv \\
& \le \inner uu + 2 \norm u \norm v + \inner vv \\
& = \norm{u}^2 + 2 \norm u \norm v + \norm{v}^2
= (\norm u + \norm v )^2
\end{align*}
\end{enumerate}
\end{proof}
\begin{defin}
Sei $V = (v_1, \dots, v_k)$ mit $\forall i \in [k]: v_i \neq 0$.
\begin{itemize}
\item $v, w$ heißen \underline{orthogonal}, wenn $\inner vw = 0$ [schreibe auch $v \bot w$]
\item $V$ heißt \underline{Orthogonalsystem} (OS), wenn
$\forall i, j \in [k], i\neq j: v_i \bot v_j$
\item $V$ heißt \underline{Orthonormalsystem} (ONS), wenn $V$ ein Orthogonalsystem ist
und $\forall i \in [k]: \norm{v_i}= 1$
\item $V$ heißt \underline{Orthogonalbasis} (OB), wenn $V$ ein Orthogonalsystem und eine Basis ist.
\item $V$ heißt \underline{Orthonormalbasis} (ONB), wenn $V$ ein Orthonormalsystem und eine Basis ist.
\end{itemize}
\end{defin}
\begin{satz}
Sei $(v_1, \dots, v_k)$ ein Orthogonalsystem. Dann ist $(v_1, \dots, v_k)$ linear \\unabhängig.
\end{satz}
\begin{proof}
Angenommen $\lambda_1 v_1 + \dots + \lambda_k v_k = 0$
\[
\forall i \in [k]: 0 = \lambda_1 \underbrace{\inner {v_1}{v_i}}_{=0} + \dots +
\lambda_i \underbrace{\inner {v_i}{v_i}}_{\neq0} + \dots +
\lambda_k \underbrace{\inner {v_k}{v_i}}_{=0}
= \lambda_i \underbrace{\norm{v_i}^2}_{\neq 0}
\]
$\implies \lambda_i = 0$
\end{proof}
\begin{satz}
\label{theo:3.1.15}
Sei $B=(b_1, \dots, b_n)$ Orthonormalbasis von $V, n\in \mathbb{N}\cup \{\infty\}$.
Dann gilt für alle $v, w \in V$ und $(\lambda_1, \dots, \lambda_n) = {}_B \Phi(v), (\mu_1, \dots, \mu_n)
= {}_B\Phi(w)$:
\[
\inner vw = \sum_{i=1}^n \lambda_i \overline{\mu_i}
\]
Weiters gilt $\lambda_i = \inner{v}{b_i}, b_i^*(v) = \inner v {b_i}$
\end{satz}
\begin{proof}
\begin{align*}
& \inner{b_i}{b_j} = \delta_{ij} \\
& \begin{rcases}v = \sum_{i=1}^n \lambda_i b_i \\
w = \sum_{i=1}^n \mu_i b_i\end{rcases} \implies \inner vw
= \sum_{i, j = 1}^n \inner{\lambda_i b_i}{\mu_j b_j} = \sum_{i, j = 1}^n \lambda_i \overline{\mu_j}
\inner{b_i}{b_j} = \sum_{i=1}^n \lambda_i \overline{\mu_i} \\
& {}_B \Phi(b_i) = (0, \dots, \overset{i}{1}, \dots, 0) \implies \inner v{b_i} = \sum_{j=1}^n \lambda_j
\delta_{ij} = \lambda_i
\end{align*}
\end{proof}
\begin{satz}
[Gram-Schmidt Orthonormalisierungsverfahren]
\label{theo:3.1.16}
Sei $(a_1, a_2, \dots) \subseteq V$ linear unabhängig. Dann existiert genau ein Orthonormalsystem
$(b_1, b_2, \dots)$ mit
\begin{enumerate}[label=\roman*)]
\item $\forall k: \langle a_1, \dots, a_k \rangle = \langle b_1, \dots, b_k \rangle =: U_k$
\item Die Basistransformationsmatrix $M_k$ zwischen der Basen $(a_1, \dots, a_k)$ und $(b_1, \dots, b_k)$
von $U_k$ hat positive Determinante.
\end{enumerate}
\end{satz}
\begin{proof}
$b_1, b_2, \dots$ werden induktiv definiert.
\begin{itemize}
\item $b_1 = \frac{a_1}{\norm{a_1}}, M_1 = \begin{pmatrix}\frac{1}{\norm{a_1}}\end{pmatrix}$ \\
Eindeutigkeit: Sei $\tilde b_1$ mit i), ii) $\implies \tilde b_1 = c \cdot a_1, 1 = \norm{\tilde b_1}
= \norm{c \cdot a_1} = \lvert c \rvert \norm{a_1}$ \\
$ \implies \lvert c \rvert = \dfrac{1}{\norm{a_1}} \implies \tilde M_k =(c)$
\item $(b_1, \dots, b_n)$ schon konstruiert mit i), ii) \\
Sei $c_{n+1} := a_{n+1} - \sum\limits_{j=1}^n \inner{a_{n+1}}{b_j} b_j$
\begin{align*}
& \forall i \in [n]: \inner{c_{n+1}}{b_i} = \inner{a_{n+1}}{b_i} -
\sum\limits_{j=1}^n \inner{a_{n+1}}{b_j} \underbrace{\inner{b_j}{b_i}}_{\delta_{ij}} \\
& = \inner{a_{n+1}}{b_i}
- \inner{a_{n+1}}{b_i} = 0 \implies c_{n+1} \bot \langle b_1, \dots, b_n \rangle
\end{align*}
$b_{n+1} = \dfrac{c_{n+1}}{\norm{c_{n+1}}} \implies (b_1, \dots, b_{n+1})$ Orthonormalsystem mit \\
$\langle b_1, \dots, b_n \rangle = \langle a_1, \dots, a_n \rangle$
\begin{align*}
& b_1 = \mu_{11} a_1 \\
& b_2 = \mu_{21} a_1 + \mu_{22} a_2 \\
& b_3 = \mu_{31} a_1 + \mu_{32} a_2 + \mu_{33} a_3 \\
& \vdots \\
& b_n = \mu_{n1} a_1 + \dots + \mu_{nn} a_n \\
& b_{n+1} = \mu_{n+1 1} a_1 + \dots + \mu_{n+1 n} a_n + \dfrac{1}{\norm{c_{n+1}}} a_{n+1} \\
& \implies \det(\mu_{ij}) = \det(M_n) \cdot \dfrac{1}{\norm{c_{n+1}}} > 0
\end{align*}
Eindeutigkeit: Sei $\tilde b_{n+1}$ ein weiterer Vektor mit i), ii)
\begin{align*}
& \implies \tilde b_{n+1} = \mu_1 b_1 + \dots + \mu_n b_n + \mu b_{n+1} \\
& \forall i \in [n]: 0 = \inner{\tilde b_{n+1}}{b_i} = \mu_i \implies \tilde b_{n+1} = \mu b_{n+1} \\
& 1 = \norm{\tilde b_{n+1}} = \lvert \mu \rvert \norm{b_{n+1}} = \lvert \mu \rvert \implies \lvert \mu
\rvert = 1 \\
& \det(\tilde M_{n+1}) = \det(M_n) \cdot \mu > 0 \implies \mu = 1 \land \tilde b_{n+1} = b_{n+1}
\end{align*}
\end{itemize}
\end{proof}
\begingroup
\allowdisplaybreaks
\subsubsection{Veranschaulichung im $\R^2$}
\begin{tikzpicture}[scale=3.09]
\tikzmath{
\a1 = 3;
\a2 = 1;
\a3 = 2;
\a4 = 2;
\norma1 = sqrt((\a1 * \a1 + \a2 * \a2));
\normeda1 = \a1 / \norma1;
\normeda2 = \a2 / \norma1;
\innerprod = (\normeda1 * \a3) + (\normeda2 * \a4);
\c1 = \a3 - (\innerprod * \normeda1);
\c2 = \a4 - (\innerprod * \normeda2);
\t1 = (\innerprod * \normeda1);
\t2 = (\innerprod * \normeda2);
\normc = sqrt((\c1 * \c1 + \c2 * \c2));
\b3 = \c1 / \normc;
\b4 = \c2 / \normc;
}
\draw [->] (0, 0) --node[above]{$a_1$} (\a1, \a2);
\draw [->] (0, 0) --node[above]{$a_2$} (\a3, \a4);
\draw [->, blue, thick] (0, 0) --node[below right]{$\inner{a_2}{b_1}b_1$} (\t1, \t2);
\draw [->, violet, very thick] (0, 0) --node[below right]{$\frac{a_1}{\norm{a_1}}=:b_1$} (\normeda1, \normeda2);
\draw [->, magenta, thick] (0, 0) --node[left]{$c_2:=a_2 - \inner{a_2}{b_1}b_1$} (\c1, \c2);
\draw [->, teal, very thick] (0, 0) --node[right]{$\frac{c_2}{\norm{c_2}}=:b_2$} (\b3, \b4);
\end{tikzpicture}
\subsubsection{Beispiel}
$V = \R^4, a_1 = \begin{pmatrix} 4 \\ 2 \\ -2 \\ -1 \end{pmatrix},
a_2 = \begin{pmatrix} 2 \\ 2 \\ -4 \\ -5 \end{pmatrix},
a_3 = \begin{pmatrix} 0 \\ 8 \\ -2 \\ -5 \end{pmatrix}$
\begin{align*}
& b_1 = \frac{1}{\norm{a_1}} a_1 ,\; \norm{a_1} = (4^2 + 2^2 + 2^2 + 1^2)^{\frac 12} = \sqrt{25} = 5 \\
& = \frac 15 \begin{pmatrix} 4 \\ 2 \\ -2 \\ -1 \end{pmatrix} ,\;
\inner{a_2}{b_1} = \frac 15 \begin{pmatrix} 2 \\ 2 \\ -4 \\ -5 \end{pmatrix} \cdot
\begin{pmatrix} 4 \\ 2 \\ -2 \\ -1 \end{pmatrix} = \frac 15 (8 + 4 + 8 + 5)^\frac 12 = \frac{25}5 \\
& c_2 = a_2 - \underbrace{\inner{a_2}{b_1}}_5 b_1 = \begin{pmatrix} 2 \\ 2 \\ -4 \\ -5 \end{pmatrix} -
\begin{pmatrix} 4 \\ 2 \\ -2 \\ -1 \end{pmatrix} = \begin{pmatrix} -2 \\ 0 \\ -2 \\ -4 \end{pmatrix} \\
& \norm c_2 = (4 + 4 + 16) = \sqrt{24} \\
& \implies b_2 = \frac{1}{\sqrt{24}} \begin{pmatrix} -2 \\ 0 \\ -2 \\ -4 \end{pmatrix} \\
& c_3 = a_3 - \inner{a_3}{b_1} b_1 - \inner{a_3}{b_2} b_2 = \dots =
\begin{pmatrix} -2 \\ 6 \\ 2 \\ 0\end{pmatrix} \\
& \norm{c_3} = (4 + 36 + 4)^\frac 12 = \sqrt{44} \\
& \implies b_3 = \frac 1{\sqrt{44}} \begin{pmatrix} -2 \\ 6 \\ 2 \\ 0 \end{pmatrix}
\end{align*}
\endgroup
\begin{satz}
\label{theo:3.1.17}
Sei $V$ ein euklidischer/unitärer Vektorraum mit höchstens abzählbarer Dimension.
Dann kann jedes Orthonormalsystem zu einer Orthonormalbasis von $V$ ergänzt werden.
\end{satz}
\begin{proof}
Sei $(b_1, \dots, b_k)$ ein Orthonormalsystem, $(b_1, \dots, b_k, a_{k+1}, \dots)$ eine Basis.
Satz \ref{theo:3.1.16} $\implies \exists b_{k+1}, b_{k+2}, \dots$ mit $(b_1, \dots, b_k, b_{k+1}, \dots)$
Orthonormalbasis.
\end{proof}
\begin{defin}
\leavevmode
\begin{itemize}
\item $M, N \subseteq V$ heißen \underline{orthogonal} wenn $\forall v \in M, w \in N:
\underset{\inner vw = 0}{v \bot w}$ \\
Wir schreiben $M \bot N$ \\
$[M = {v} \implies v \bot N]$
\item Für $M \subseteq V$ heißt
\[
M^\bot := \{ v\in V: v \bot M \} = \{ v \in V: \forall w \in M: \inner vw = 0 \}
\]
\underline{orthogonales Komplement} von M
\end{itemize}
\end{defin}
\subsubsection{Bemerkung}
$M^\bot$ ist immer Unterraum von $V$, selbst wenn $M$ kein Unterraum ist.
\begin{satz}
Sei $U$ $r$-dimensionaler Unterraum von $n$-dimensionalem euklidischen/\\ unitären Vektorraum $V$. Dann gilt:
\begin{enumerate}[label=\alph*)]
\item $\dim(U^\bot) = n - r$
\item $(U^\bot)^{{}^\bot} = U$
\item $V = U \oplus U^\bot$
\end{enumerate}
\end{satz}
\begin{proof}
\begin{enumerate}[label=\alph*)]
\item $(b_1, \dots, b_r)$ Orthonormalbasis von $U$.\\
$(b_1, \dots, b_r, b_{r+1}, \dots, b_n)$ Orthonormalbasis von $V$. \newline
[die existiert laut Satz \ref{theo:3.1.17}] \\
Behauptung: $U^\bot = \langle b_{r+1}, \dots, b_n \rangle$ \\
Beweis: $\subseteq$: Sei $v\in U^\bot, v = \sum_{i=1}^n \lambda_i b_i$
\[
\forall i \in [r]: 0 = \inner v{b_i} = \inner{\sum_{j=1}^n \lambda_j b_j}{b_i} =
\sum_{j=1}^n \lambda_j \underbrace{\inner{b_j}{b_i}}_{\delta_{ij}} = \lambda_i
\]
$\supseteq: v\in \langle b_{r+1}, \dots, b_n \rangle \overset{!}{\implies} v \in U^\bot$ \\
$\implies \sum_{j=r+1}^n \lambda_j b_j, u = \sum_{i=1}^r \mu_i b_i \in U$ \\
$\implies \inner vu = \sum_{j=r+1}^n \lambda_j \sum_{i=1}^r \underbrace{\inner{b_j}{b_i}}_{=0}=0$ \\
$\implies$ a)
\item $U \subseteq (U^\bot)^{{}^\bot}$ \\
Sei $v \in U \implies \forall w \in U^\bot: \inner wv = 0 \implies v \in (U^\bot)^{{}^\bot}$ \\
$(U^\bot)^{{}^\bot} \subseteq U: \dim((U^\bot)^{{}^\bot}) \overset{\text{a)}}{=} n - \dim(U^\bot)
\overset{\text{a)}}{=} n - (n-r) = r = \dim(U)$
\item $U\oplus U^\bot$: Sei $w\in U \cap U^\bot \implies \inner ww = 0 \implies w = 0$
\end{enumerate}
\end{proof}
\subsubsection{Bemerkung}
$(U^\bot)^{{}^\bot}$ gilt im Allgemeinen nicht, wenn $\dim(V) = \infty$.
\subsubsection{Beispiel}
$V = \{ f: [0, 1] \to \R, \text{ stetig} \}, \inner fg = \int_0^1 f(t) g(t) dt$\\
$U = \{ p \in V: p \text{ ist Polynom}\}$\\
$(p_1, p_2, \dots)$ ist eine Orthonormalbasis von $U$.\\
Wir zeigen: $U^\bot = \{0\} \implies (U^\bot)^{{}^\bot} =V\neq U$
\begin{satz*}[Weierstraß]
$\forall f \in V, \varepsilon > 0 \; \exists p \in U: \norm{f-p}_\infty \le \varepsilon$\\
Beweis wird hier nicht geführt.
\end{satz*}
\par
Sei $f \in V \setminus \{0\}, a := \norm f^2 = \inner ff, b = \norm f _\infty$.
Sei $p \in U: \norm{f-p}_\infty < \frac a {2b}$ \\
Behauptung: $\inner fp > 0$
\begin{align*}
\inner fp = \int_0^1 f(t)p(t) dt & = \int_0^1 f(t)[f(t) - (f(t) - p(t))]dt \\
& = \int_0^1 f(t)f(t)dt - \int_0^1 f(t)(f(t) - p(t)) dt \\
& = a - \int_0^1 f(t)[f(t)-p(t)] dt \\
\int_0^1 \underbrace{f(t)}_{\substack{\le \norm b_\infty \\ \le b}}
[\underbrace{f(t) - p(t)}_{\substack{\le \norm{f - p}_\infty \\\le \frac a {2b}}}]
& \le \int_0^1 b \cdot \frac a {2b}dt -
\int_0^1 \frac a2 dt = \frac a2 \\
\forall f \in V: \exists p \in U: \int_0^1 f(t)p(t)dt
& \neq 0 \implies U^\bot = \{\} \implies U \subsetneq (U^\bot)^{{}^\bot}
\end{align*}
\section[Adjungierte Abbildungen und normale Endomorphismen]
{Adjungierte Abbildungen und normale \\Endomorphismen}
\begin{defin}
Seien $V, W$ euklidische/unitäre Vektorräume, $\alpha \in \homk(V, W), \K \in \{\R, \C\}$. \\
$\alpha^* \in \homk(W, V)$ heißt \underline{zu $\alpha$ adjungiert}, falls
\[
\forall v \in V: \forall w \in W: \inner{\alpha(v)}{w}_W = \inner{v}{\alpha^*(w)}_V
\]
$V=W$: Gilt $\genfrac{}{}{0pt}{0}{\alpha = -\alpha^*}{\alpha = \alpha^*}$,
so heißt $\alpha$ $\genfrac{}{}{0pt}{0}{\text{\underline{anti-selbstadjungiert}}}
{\text{\underline{selbstadjungiert}}}$.
\end{defin}
\subsubsection{Bemerkung}
\begin{itemize}
\item $\alpha^*$ muss nicht existieren! \\
Beispiel: $U, V$ wie vorher. $\alpha \in \Hom_\R(U, V), \alpha(p) = p \forall p \in U$ \\
Angenommen $\exists \alpha^* \in \Hom_\R(V, U), e(t) = e^t \implies e \in V$ \\
$\alpha^* (e) = a_1 p_1 + \dots + a_m p_m$ \\
$f := e - (a_1 p_1 + \dots + a_m p_m) = e- \alpha^*(e) \neq 0$ \\
Behauptung: $f \in U^\bot (\implies f = 0$ \Lightning)
\item $i \in \{m+1, m+2, \dots \}$ %Eigentlich nicht so wirklich ein Punkt?!
\begin{align*}
& \inner{e}{p_i} = \inner{e}{\alpha(p_i)} = \inner{\alpha^*(e)}{p_i} =
\inner{a_1 p_1 + \dots + a_m p_m}{p_i} = 0 \\
& \implies \forall i = m+1, m+2, \dots: \inner{f}{p_i} = 0
\end{align*}
Außerdem: $i\in [m]: \inner{e}{p_i} = \inner{\alpha^*}{p_i}$ \newline
$\implies \inner{f}{p_i} =
\inner{e - \alpha^*(e)}{p_i} = \inner{\alpha^*(e) - \alpha^*(e)}{p_i} = 0$ \\
$\implies \inner{f}{p_i} \forall i = 1, 2, \dots \implies f \in U^\bot \implies f= \{0\}$\Lightning
\item Wenn $\alpha^*$ existiert, dann ist $\alpha^*$ eindeutig.
\end{itemize}
\begin{lemma}
Sei $\alpha \in \homk(V, W), \dim(V) < \infty$
Dann existiert $\alpha^*$: \\
Mit $\{e_1, \dots, e_n\}$ Orthonormalbasis von $\underbrace{V}_{
\mathclap{\text{Existenz gegeben wegen Satz \ref{theo:3.1.17}}}}$ gilt:
\[
\alpha^*(w) := \sum_{i=1}^n \inner{w}{\alpha(e_i)} e_i
\]
\end{lemma}
\begin{proof}
Zu Zeigen: $\forall v \in V, w \in W: \inner{\alpha(v)}{w} = \inner{v}{\alpha^*(w)}$.\\
O.B.d.A.: $v = e_j$ für $j \in [n]$
\begin{align*}
\inner{v}{\alpha^*(w)} = \inner{e_j}{\sum_{i=1}^n \inner{w}{\alpha(e_i)}e_i}
& = \sum_{i=1}^n \inner{e_j}{\inner{w}{\alpha(e_i)} e_i} \\
& = \sum_{i=1}^n \overline{\inner{w}{\alpha(e_i)}} \underbrace{\inner{e_j}{e_i}}_{\delta_ij} \\
& = \overline{\inner{w}{\alpha(e_j)}} = \inner{\alpha(e_j)}{w} = \inner{\alpha(v)}{w}
\end{align*}
\end{proof}
\begin{defin}
Sei $A \in \C^{m \times n}$.
\begin{align*}
& \overline{A} := (\overline{a}_{ij})_{i,j} & & \text{ zu $A$ \underline{konjugiert komplexe} Matrix} \\
& A^* = (\overline{A})^T & & \text{ zu $A$ \underline{adjungierte} Matrix}
\end{align*}
\end{defin}
\begin{satz}
\label{theo:3.2.4}
Sei $\alpha \in \homk(V, W), \K \in \{\R, \C\}, \dim(V), \dim(W) < \infty$
\[
\begin{rcases}
E = \{e_1, \dots, e_n\} \text{ ONB von V} \\
F = \{f_1, \dots, f_n\} \text{ ONB von W}
\end{rcases}
\implies {}_E M(\alpha^*)_F = ({}_F M(\alpha)_E)^*
\]
\end{satz}
\begin{proof}
\begin{align*}
& A = {}_F M(\alpha)_E = (a_{ij})_{\substack{i=1,\dots,m \\j=1,\dots,n}}, \,
B = {}_E M(\alpha^*)_F = (b_{ij})_{\substack{i=1,\dots,n \\j=1,\dots,m}} \\
& \alpha(e_j) = \sum_{i=1}^m a_{ij} f_i \\
& F \text{ ONB} \implies a_{ij} = \inner{\alpha(e_j)}{f_i} \\
& \alpha^*(f_j) = \sum_{i=1}^n b_{ij} e_i \implies b_{ij} = \inner{\alpha^*(f_j)}{e_i} \\
& \dots = \overline{\inner{e_i}{\alpha^*(f_j)}} = \overline{\inner{\alpha(e_i)}{f_j}} = \overline{a_{ji}} \\
& \implies B = A^*
\end{align*}
\end{proof}
\begin{lemma}
\leavevmode
\begin{enumerate}[label=\alph*)]
\item $(\alpha^*)^{{}^*} = \alpha$
\item $(\alpha + \beta)^* = \alpha^* + \beta^*$
\item $(\lambda \alpha)^* = \overline{\lambda}\alpha^*$
\item $(\beta \circ \alpha)^* = \alpha^* \circ \beta^*$
\item $\alpha \in \homkv, \dim(V) < \infty \implies \det(\alpha) = \overline{\det(\alpha^*)}$
\end{enumerate}
\end{lemma}
\begin{proof}
\begin{enumerate}[label=\alph*)]
\item $\inner{\alpha(v)}{w} = \inner{v}{\alpha^*(w)} = \overline{\inner{\alpha^*(w)}{v}} =$ \\
$\overline{\inner{w}{(\alpha^*)^{{}^*}(v)}} = \inner{(\alpha^*)^{{}^*}(v)}{w} \; \forall v \in V, w \in W$ \\
$\implies \inner{\alpha(v) - (\alpha^*)^{{}^*}(v)}{w} = 0 \; \forall v \in V, w \in W, \;
w:= \alpha(v) - (\alpha^*)^{{}^*}$ \\
$\implies \inner{\alpha(v) - (\alpha^*)^{{}^*}}{\alpha(v) - (\alpha^*)^{{}^*}} = 0 \iff
\norm{\alpha(v) - (\alpha^*)^{{}^*}} = 0 \implies \forall v \in V: \alpha(v) = (\alpha^*)^{{}^*}$
\item $ \inner{(\alpha + \beta)}{w} = \inner{v}{(\alpha + \beta)^*(w)} $ \\
$ = \inner{\alpha(v) + \beta(v)}{w} = \inner{\alpha(v)}{w} + \inner{\beta(v)}{w} = \inner{v}{\alpha^*(w)} +
\inner{v}{\beta^*(w)} = \inner{v}{\alpha^*(w) + \beta^*(w)}$
\item $ \inner{(\lambda \alpha)(v)}{w} = \inner{v}{(\lambda \alpha)^*(w)} $\\
$ = \lambda \inner{\alpha(v)}{w} = \lambda \inner{v}{\alpha^*(w)} = \inner{v}{\overline{\lambda} \alpha^*(w)}$
\item $ \inner{\beta \circ \alpha(v)}{w} = \inner{\alpha(v)}{\beta^*(w)} = \inner{v}{\alpha^* \circ \beta^*(w)}$ \\
$ = \inner{v}{(\beta \circ \alpha)^*(w)} $
\item Sei $E$ Orthonormalbasis, $A = {}_E M(\alpha)_E = (a_{ij}) \in \C^{\nxn}$
\[
\overline{\det(\alpha)} = \sum_{\pi \in S_n} \sgn(\pi) \overline{a}_{1\pi(1)} \cdots \overline{a}_{n\pi(n)}
= \det(\overline{A}) = \det(\overline A^T) = \det(A^*)
\]
$ =\det({}_E M(\alpha^*)_E) = \det(\alpha^*) $ %Eigentlich noch in dem oberen environment
\end{enumerate}
\end{proof}
\begin{defin}
$\alpha \in \homkv$ mit $V$ euklidisch/unitär heißt \underline{normal}, wenn $\alpha^*$ existiert und
\[
\alpha \circ \alpha^* = \alpha^* \circ \alpha
\]
\end{defin}
\begin{satz}
\label{theo:3.2.7}
\[
\alpha \text{ normal} \iff \inner{\alpha(v)}{\alpha(w)} = \inner{\alpha^*(v)}{\alpha^*(w)}
\]
\end{satz}
\begin{proof}
\begin{itemize}
\item[$\implies$:]$\inner{\alpha(v)}{\alpha(w)} = \inner{v}{\alpha^*(\alpha(w))}
\underbrace{=}_{\alpha \text{ normal}} \inner{v}{\alpha(\alpha^*(w))}$ \\
$ \inner{\alpha^*(v)}{\alpha^*(w)} = \inner{v}{(\alpha^*)^{{}^*} (\alpha^*(v))} = \inner{v}{\alpha(\alpha^*(w))}$
\end{itemize}
\end{proof}
\begin{lemma}
\[
\alpha \text{ normal } \implies \ker(\alpha) = \ker(\alpha^*)
\]
\end{lemma}
\begin{proof}
\begin{align*}
& \norm{\alpha(v)}^2 = \inner{\alpha(v)}{\alpha(v)} = \inner{\alpha^*(v)}{\alpha^*(v)} = \norm{\alpha^*(v)}^2 \\
& v \in \ker(\alpha) \iff \alpha(v) = 0 \iff \norm{\alpha(v)} = 0 \iff \norm{\alpha^*(v)} = 0 \\
& \iff \alpha^*(v) = 0 \iff v \in \ker(\alpha^*)
\end{align*}
\end{proof}
\begin{satz}
\label{theo:3.2.9}
$\alpha$ normal:
\begin{enumerate}[label=\alph*)]
\item $\alpha$ und $\alpha^*$ besitzen die selben Eigenvektoren.
\item $v \in \eig_\alpha(\lambda)\implies v \in \eig_{\alpha^*}(\overline \lambda)$
\end{enumerate}
\end{satz}
\begin{proof}
$v \in \eig_\alpha(\lambda)$
\begin{align*}
\norm{\alpha(v) - \lambda v}^2 & = \inner{\alpha(v) - \lambda v}{\alpha(v) - \lambda v} \\
& = \inner{\alpha(v)}{\alpha(v)} - \lambda \inner{v}{\alpha(v)} -
\overline\lambda \inner{\alpha(v)}{v} + \lambda \overline \lambda
\inner vv \\
& \underbrace{=}_{\alpha\text{ normal}}
\inner{\alpha^*(v)}{\alpha^*(v)} - \lambda \overline{\inner{\alpha(v)}{v}}
-\overline{\lambda} \inner{v}{\alpha^*(v)} + \lambda \overline \lambda
\inner vv \\
& = \inner{\alpha^*(v)}{\alpha^*(v)} - \lambda \overline{\inner{v}{\alpha^*(v)}}
- \overline \lambda \inner{v}{\alpha^*(v)}
+ \lambda \overline \lambda \inner vv \\
& =\inner{\alpha^*(v)}{\alpha^*(v)} - \lambda \inner{\alpha^*(v)}{v}
- \overline \lambda \inner{v}{\alpha^*(v)} + \lambda \overline\lambda
\inner vv \\
& = \inner{\alpha^*(v) - \overline \lambda v}{\alpha^*(v) - \overline{\lambda}
v} = \norm{\alpha^*(v) - \overline \lambda v}^2
\end{align*}
\begin{align*}
& v \in \eig_\alpha(\lambda) \iff \alpha(v) - \lambda v = 0 \iff \norm{\alpha(v) - \lambda v}^2 = 0 \\
& \iff \norm{\alpha^*(v) - \overline \lambda v}^2 = 0 \iff \alpha^*(v) - \overline \lambda v = 0
\iff v \in \eig_{\alpha^*}(\overline\lambda)
\end{align*}
\end{proof}
\begin{satz}
[Spektralsatz für normale Abbildungen]
\label{theo:3.2.10}
$\alpha \in \Hom_\C(V, V), V$ unitär mit $\dim(V) = n < \infty$. Dann gilt:
\[
\exists \text{ Orthonormalbasis aus Eigenvektoren von } \alpha \iff \alpha \text{ normal}
\]
\end{satz}
\begin{proof}
\leavevmode
\begin{itemize}
\item[$\impliedby$:] $\alpha$ normal
\begin{itemize}
\item[$n=1$:] $\exists$ Eigenvektor $e_1 \in V \setminus\{0\}$ mit $\alpha(e_1) = \lambda e_1$.\\
o.B.d.A.: $\norm{e_1} = 1 \implies v$ ist Orthonormalbasis aus Eigenvektoren
\item[$n-1 \to n$:] $\exists$ Eigenvektor $e_1 \in V \setminus\{0\}$ mit $\alpha(e_1) = \lambda e_1$.\\
o.B.d.A.: $\norm{e_1} = 1 \; U= \langle e_1 \rangle ^\bot$
\begin{itemize}
\item $V = \langle e_1 \rangle \oplus U, \alpha(U) \overset{\text{!}}{\subseteq} U,
\alpha(\langle e_1 \rangle) \subseteq \overset{\checkmark}{\langle}e_1 \rangle$
\end{itemize}
$\implies \alpha = \alpha|_{\langle e_1 \rangle} \oplus \alpha|_U$ \\
Sei $v \in U\implies 0 = \inner{v}{e_1}
\;\;\; [e_1 \in \eig_\alpha(\lambda) \iff e_1 \in \eig_{\alpha^*}(\overline\lambda)]$
\begin{align*}
\inner{\alpha(v)}{e_1} & = \inner{v}{\alpha^*(e_1)} = \inner{v}{\overline \lambda e_1} \\
& = \lambda \inner{v}{e_1} = 0 \\
\implies \alpha(v) \in U & \implies \alpha(U) \subseteq U \checkmark \\
& \implies \alpha|_U \in \Hom(U, U), \dim(U) = n-1 \\
\end{align*}
\begin{align*}
& \overset{\mathclap{\substack{\text{Induktionsvorraussetzung} \\|}}}
{\implies} \exists
\text{ ONB } (e_2, \dots, e_n) \text{ von $U$ aus Eigenvektoren von } \alpha \\
& \implies (e_1, \dots, e_n) \text{ ist ONB von $V$ aus Eigenvektoren von } \alpha
\end{align*}
\end{itemize}
\item[$\implies$:] Sei $(e_1, \dots, e_n)$ Orthonormalbasis aus Eigenvektoren von $\alpha$. Seien weiters
$\lambda_1, \dots, \lambda_n \in \C$ die zugehörigen Eigenwerte.
\[
\alpha:\begin{cases} V & \to V \\
v = \sum_{i=1}^n \mu_i e_i & \mapsto \sum_{i=1}^n \lambda_i \mu_i e_i\end{cases}
\]
Definiere
\[
\beta:\begin{cases} V & \to V \\ v = \sum_{i=1}^n \mu_i e_i &\mapsto \sum_{i=1}^n \overline \lambda_i
\mu_i e_i\end{cases} \implies \beta = \alpha^*
\]
\[
\begin{aligned}
\alpha^*(\alpha(v)) & = \alpha^*( \sum_{i=1}^n \lambda_i \mu_i e_i )
= \sum_{i=1}^n \overline\lambda_i \lambda_i \mu_i e_i
= \sum_{i=1}^n \lvert \lambda_i \rvert^2 \mu_i e_i \\
\alpha(\alpha^*(v)) & = \alpha( \sum_{i=1}^n \overline\lambda_i \mu_i e_i )
= \sum_{i=1}^n \lambda_i \overline\lambda_i \mu_i e_i
= \sum_{i=1}^n \lvert \lambda_i \rvert^2 \mu_i e_i
\end{aligned}
\implies \alpha \text{ normal}
\]
\end{itemize}
\end{proof}
\subsubsection{Bemerkung}
Im Reellen/Euklidischen Fall gilt dieser Satz genau dann, wenn $\alpha$ diagonalisierbar ist.
\begin{satz}
\label{theo:3.2.11}
Sei $V$ ein unitärer Vektorraum mit $\dim(V) = n < \infty$ und $\alpha \in \Hom_\C(V, V)$ selbstadjungiert.
(Das heißt $\alpha = \alpha^*$) \\
Dann gilt:
\begin{enumerate}[label=\alph*)]
\item Alle Eigenwerte von $\alpha$ sind reell.
\item $V$ besitzt eine Orthonormalbasis aus Eigenvektoren von $\alpha$.
\item Eigenvektoren zu unterschiedlichen Eigenwerten sind orthogonal.
\end{enumerate}
\end{satz}
\begin{proof}
$\alpha$ ist normal: $\alpha \circ \alpha^* = \alpha \circ \alpha = \alpha^* \circ \alpha$
\begin{enumerate}[label=\alph*)]
\item Sei $\lambda$ Eigenwert von $\alpha$ mit Eigenvektor $v\in V\setminus\{0\}$\\
$\overset{\text{\ref{theo:3.2.7}}}{\implies} v$ ist Eigenvektor von $\alpha^*$ mit Eigenwert
$\overline{\lambda}$ \\
$\implies \lambda v = \alpha(v) = \alpha^*(v) = \overline{\lambda} v \implies
(\lambda - \overline{\lambda}) v = 0 \overset{v\neq0}{\implies} \lambda = \overline{\lambda}$ \\
$\implies \lambda \in \R$
\item Folgt direkt aus Satz \ref{theo:3.2.10} \& $\alpha$ normal.
\item Sei $\alpha(v_1) = \lambda_1 v_1, \alpha(v_2)=\lambda_2 v_2, \lambda_1 \neq \lambda_2$
\begin{align*}
& \lambda_1 \inner{v_1}{v_2} = \inner{\lambda_1 v_1}{v_2} = \inner{\alpha(v_1)}{v_2} =
\inner{v_1}{\alpha^*(v_2)} = \inner{v_1}{\alpha(v_2)} \\
& = \inner{v_1}{\lambda_2 v_2} = \overline{\lambda_2} \inner{v_1}{v_2} = \lambda_2 \inner{v_1}{v_2} \\
& \implies \underbrace{(\lambda_1 - \lambda_2)}_{\neq 0} \inner{v_1}{v_2} = 0 \implies \inner{v_1}{v_2}=0
\end{align*}
\end{enumerate}
\end{proof}
\begin{lemma}
Sei $V$ ein euklidischer Vektorraum und $\alpha \in \Hom_\R(V, V)$ normal. Dann ist
$\alpha_\C \in \Hom_\C (V_\C, V_\C)$ auch normal.
\end{lemma}
\begin{proof}
Sei $\alpha \in \Hom(V, V), \alpha_\C$ die komplexe Erweiterung von $\alpha$. Seien weiters $v, v' \in V$ mit
$v = u+iw, v' = u' +iw', u, w, u', w' \in \R$.
\begin{equation}
\label{eq:3.2.12.1}
\begin{split}
\inner{\alpha_\C(v)}{\alpha_\C(v')} & = \inner{\alpha(u) + i\alpha(w)}{\alpha(u') + i\alpha(w')} \\
& \begin{multlined}=\inner{\alpha(u)}{\alpha(u')} +
i \inner{\alpha(w)}{\alpha(u')}
- i \inner{\alpha(u)}{\alpha(w')} \\
+ (-i)(-i)\inner{\alpha(w)}{\alpha(w')}\end{multlined} \\
& \begin{multlined}= \inner{\alpha^*(u)}{\alpha^*(u')} + i \inner{\alpha^*(w)}{\alpha^*(u')}
- i \inner{\alpha^*(u)}{\alpha^*(w')} \\
+ (-i)(-i)\inner{\alpha^*(w)}{\alpha^*(w')} \end{multlined} \\
& = \inner{\alpha^*(u)+i\alpha^*(w)}{\alpha^*(u')} +
\inner{\alpha^*(u) + i\alpha^*(w)}{i\alpha^*(w')} \\
& = \inner{\alpha^*(u) + i\alpha^*(w)}{\alpha^*(u') + i\alpha^*(w')} \\
& = \inner{(\alpha^*)_\C(v)}{(\alpha^*)_\C(v')}
\end{split}
\end{equation}
Bleibt zu Zeigen, dass $(\alpha^*)_\C = (\alpha_\C)^*$:
\begin{align*}
\inner{\alpha_\C(v)}{v'} & = \inner{\alpha(u) + i \alpha(w)}{u' + i w'} \\
& = \inner{\alpha(u)}{u'} + i \inner{\alpha(w)}{u'} -i \inner{\alpha(u)}{w'} +
i (-i) \inner{\alpha(w)}{w'} \\
& = \inner{u}{\alpha^*(u')} + i \inner{w}{\alpha^*(u')}
-i \inner{u}{\alpha^*(w')} +i (-i) \inner{w}{\alpha^*(w')} \\
& = \inner{u+iw}{\alpha^*(u')} + \inner{u+iw}{i\alpha^*(w')} \\
& = \inner{u + iw}{\alpha^*(u') + i \alpha^*(w')} = \inner{v}{(\alpha^*)_\C(v')}
\end{align*}
Das heißt $(\alpha^*)_\C$ ist tatsächlich die adjungierte Abbildung von $\alpha_\C$ und mit
\ref{eq:3.2.12.1} folgt nach Satz \ref{theo:3.2.7}, dass $\alpha_\C$ normal ist.
\end{proof}
\begin{lemma}
\label{theo:3.2.13}
Sei $V$ ein euklidischer Vektorraum und $\alpha \in \Hom_\R(V, V)$ normal. Sei $v_\C \in V_\C$
$\underbrace{\text{normierter}}_{\text{d.h. }\norm{v_\C}_{V_\C}=1}$ Eigenvektor von $\alpha_\C$ zum Eigenwert
$\lambda \in \C\setminus \R$. \\
Dann ist $\overline{v_\C}$ normierter Eigenvektor von $\alpha_\C$ zu Eigenwert $\overline{\lambda}$.
Insbesondere sind $v_\C, \overline{v_\C}$ orthogonal.
\end{lemma}
\begin{proof}
Sei $u + iw \in V_\C$ mit $u, w\in V$. $\alpha_\C(u + iw) = \alpha(u) + i\alpha(w)$ \\
$v_\C = u + iw$ ist normiert
\begin{align*}
\implies 1 = \inner{u+iw}{u+iw} & = \inner uu_V + i \inner wu_V - i \inner uw_V + \inner ww_V \\
& = \inner uu_V + i \inner wu_V - i \inner wu_V + \inner ww_V \\
& = \inner uu_V + \inner ww_V \\
\implies \inner{u - iw}{u - iw} & = \inner uu_V + \inner{-w}{-w}_V = \inner uu_V \inner ww_V = 1 \\
\implies \norm{\overline{v_\C}} = 1
\end{align*}
$\lambda = \gamma + i \delta$
\[
\alpha_\C(v_\C) = \lambda v_\C \implies \alpha(u) + i \alpha(w) = (\gamma + i \delta) (u + iw) =
(\gamma u - \delta w) + i(\delta u + \gamma w)
\]
\[
\begin{aligned}
\alpha(u) = \gamma u + \delta w \\
\alpha(w) = \delta u + \gamma w
\end{aligned}
\implies
\begin{aligned}
\alpha_\C (\overline{v_\C}) = \alpha(u) + i \alpha(-w) & = (\gamma u + \delta w) + i
(\delta u - \gamma w) \\
& = (\gamma - i \delta)(u - iw)
= \overline{\lambda}\overline{v_\C}
\end{aligned}
\]
$\implies \overline{v_\C}$ ist Eigenvektor von $\alpha_\C$ zum Eigenvektor $\overline\lambda$
$\overset{\text{\ref{theo:3.2.9}}}{\implies} \overline{v_\C}$ ist Eigenvektor von $\alpha^*_\C$ zum
Eigenwert $\lambda$.
\begin{align*}
& \lambda \inner{v_\C}{\overline{v_\C}} = \inner{\alpha(v_\C)}{\overline{v_\C}} =
\inner{v_\C}{\alpha^*_\C(\overline{v_\C})} = \inner {v_\C}{\lambda \overline{v_\C}}
= \overline{\lambda} \inner{v_\C}{\overline{v_\C}} \\
& \implies (\underbrace{\lambda - \overline\lambda}
_{\mathrlap{\neq 0\text{, weil } \lambda \in \C \setminus \R}})
\inner{v_\C}{\overline{v_\C}}= 0 \implies \inner{v_\C}{\overline{v_\C}} = 0
\end{align*}
\end{proof}
\begin{satz}
\label{theo:3.2.14}
Sei $V$ ein euklidischer Vektorraum mit $\dim(V) = n < \infty$
\[
\alpha \in \Hom_\R(V, V) \text{ normal} \iff \exists \text{ ONB } B = (e_1, \dots, e_n) \text{ von } V
\text{ mit }
\]
{\setcounter{MaxMatrixCols}{20}
\[
{}_B M(\alpha)_B = \begin{pmatrix}
\lambda_1 \\
& \lambda_2 \\
& & \ddots \\
& & & \lambda_k \\
& & & & \tl \gamma_1 & \mathllap{-}\delta_1 \\
& & & & \delta_1 & \gamma_1 \br \\
& & & & & & \tl \gamma_2 & \mathllap{-}\delta_2 \\
& & & & & & \delta_2 & \gamma_2 \br \\
& & & & & & & & \ddots \\
& & & & & & & & & \tl \gamma_r & \mathllap{-}\delta_r \\
& & & & & & & & & \delta_r & \gamma_r \br
\end{pmatrix}
\]
}
wobei $\spec(\alpha_\C) = \{\underbrace{\lambda_1, \dots, \lambda_k}_{\in \R},
\underbrace{\lambda_{k+1}, \dots, \lambda_{k+r}}_{\in\C\setminus\R} \}$ und
$\lambda_{k+j} = \gamma_j + i \delta_j$
\subsubsection{Bemerkung}
Jedem Kästchen $\eta(\gamma, \delta)$ entspricht ein Paar $\lambda, \overline\lambda$ konjugiert komplexer
Eigenwerte von $\alpha_\C$. $\gamma = \Re(\lambda), \delta = \Im(\lambda)$
\end{satz}
\begin{proof}
\leavevmode
\begin{itemize}
\item[$n=1$:] \checkmark
\item[$n-1 \to n$:] Wenn $\alpha$ reellen Eigenwert besitzt, kann man wie im Beweis von Satz \ref{theo:3.2.10}
vorgehen. Wenn nicht: Sei $v_\C \in V_\C$ ein Eigenvektor von $\alpha_\C$ zum Eigenwert
$\lambda = \delta + i\gamma \in \C \setminus \R$.\\
Lemma \ref{theo:3.2.13}: $\overline{v_\C}$ ist Eigenvektor von $\alpha_\C$ zum Eigenwert $\overline\lambda$
und \\ $\inner{v_\C}{\overline{v_\C}} = 0$ \\
Setze
\[
\begin{rcases}
a = \frac{1}{\sqrt2}(v_\C + \overline{v_\C}) \in V & v_\C = u + iw \\
b = \frac{1}{i\sqrt2}(v_\C - \overline{v_\C}) \in V & \overline{v_\C} = u - iw
\end{rcases}
\implies \begin{aligned}
v_\C + \overline{v_\C} = 2u \\
\frac 1i(v_\C - \overline{v_\C}) = 2w
\end{aligned}
\]
\tl UE\br $\implies \norm a = \norm b = 1$ und $\inner ab = 0$ \\
Weiters
\[
\begin{aligned}
\alpha(a) & = \frac{1}{\sqrt{2}}(\alpha_\C(v_\C) + \alpha_\C(\overline {v_\C}))
= \frac{1}{\sqrt{2}}(\lambda v_\C + \overline \lambda \overline {v_\C}) \\
& = \frac{1}{\sqrt{2}}((\delta + i \gamma)(u + iw) + (\delta - i \gamma)(u - iw)) \\
& = \frac{1}{\sqrt{2}}((\delta u - \gamma w) + i(\delta w + \gamma u) +
(\delta u - \gamma w) - i(\delta w + \gamma u)) \\
& = \frac{1}{\sqrt{2}}(\delta 2 u - \gamma 2 w)
= \delta\frac{2u}{\sqrt{2}} - \gamma \frac{2w}{\sqrt{2}} \\
& = \delta a - \gamma b \\
\alpha(b) & = \frac{1}{i\sqrt{2}}(\alpha_\C(v_\C) - \alpha_\C(\overline{v_\C}))
= \frac{1}{i\sqrt{2}}(\lambda v_\C - \overline \lambda \overline{v_\C}) \\
& = \frac{1}{i\sqrt{2}}((\delta + i \gamma)(u + iw) - (\delta - i \gamma)(u - iw)) \\
& = \frac{1}{i\sqrt{2}}((\delta u - \gamma w) + i(\delta w + \gamma u) -
((\delta u - \gamma w) - i(\delta w + \gamma u))) \\
& = \frac{1}{i\sqrt{2}}(2i \delta w + 2i \gamma u)
= \delta \frac{2iw}{i\sqrt{2}} + \gamma \frac{2iu}{i\sqrt{2}}
= \delta \frac{2w}{\sqrt{2}} + \gamma \frac{2u}{\sqrt{2}} \\
& = \delta b + \gamma a
\end{aligned}
\]
\item[$\impliedby$:] Da $B$ Orthonormalbasis ist, folgt aus Satz \ref{theo:3.2.4} und
$\lambda_i, \gamma_i, \delta_i \in \R$, dass
${}_B M(\alpha^*)_B = {}_B M(\alpha)_B^* = \overline{{}_B M(\alpha)_B}^T = {}_B {M(\alpha)_B}^T$. Da
${}_B M(\alpha^*)_B$ eine Blockdiagonalmatrix ist, reicht es aus die multiplikative Kommutativität für
die einzelnen Blöcke zu zeigen:
\begin{align*}
& \lambda_i \lambda_i = \lambda_i \lambda_i \checkmark \\
& \begin{pmatrix} \gamma & -\delta \\ \delta & \gamma \end{pmatrix}
\begin{pmatrix} \gamma & \delta \\ -\delta & \gamma \end{pmatrix}
= \begin{pmatrix} \gamma^2 + \delta^2 & 0 \\ 0 & \delta^2 +\gamma^2 \end{pmatrix}
= \begin{pmatrix} \gamma & \delta \\ -\delta & \gamma \end{pmatrix}
\begin{pmatrix} \gamma & -\delta \\ \delta & \gamma \end{pmatrix}
\end{align*}
\end{itemize}
\end{proof}
\begin{satz}
Sei $V$ ein euklidischer/unitärer Vektorraum, $\alpha \in \homkv$ \\
anti-selbstadjungiert. Dann gilt
\begin{enumerate}[label=\alph*)]
\item $\lambda \in \spec(\alpha) \implies \Re(\lambda) = 0$
\item $\alpha_\C$ besitzt eine Orthonormalbasis aus Eigenvektoren.
\item Ist $V$ euklidisch, so sind die Diagonalelemente der Matrix ${}_B M(\alpha)_B$ gleich $0$, wobei
$B$ die Basis aus Satz \ref{theo:3.2.14} ist.
\end{enumerate}
\end{satz}
\begin{proof}
\begin{enumerate}[label=\alph*)]
\item $\alpha$ ist normal $\implies$ wegen Satz \ref{theo:3.2.9}
$v \in \eig_\alpha(\lambda)\implies v \in \eig_{\alpha^*}(\overline \lambda)$
Mit $0 \neq v \in \eig_{\alpha}(\lambda)$:
\[
\alpha(v) = \lambda v = -\alpha^*(v) = -\overline \lambda v \implies \lambda = -\overline \lambda
\implies \Re (\lambda) = 0
\]
\item $\alpha$ ist normal, $\alpha^*= -\alpha$
\item Folgt aus dem Satz~\ref{theo:3.2.14}, sowie a).
\end{enumerate}
\end{proof}
\section{Orthogonale und unitäre Abbildungen}
\begin{defin}
Seien $V, W$ beide euklidische/unitäre Vektorräume, $\alpha \in \Hom(V, W)$ heißt \underline{orthogonal}/%
\underline{unitär} wenn
\[
\forall v, w \in V: \inner{\alpha(v)}{\alpha(w)}_W = \inner vw_V
\]
\end{defin}
\subsubsection{Bemerkung}
Das sind genau die Längen- und Winkelerhaltenden Abbildungen.
\begin{satz}
\label{theo:3.3.2}
Seien $V, W$ euklidische/unitäre Vektorräume und $\alpha \in \Hom(V, W)$. Dann sind äquivalent:
\begin{enumerate}[label=\alph*)]
\item $\alpha$ ist orthogonal/unitär
\item $\forall v \in V: \norm v_V = 1 \implies \norm{\alpha(v)}_W = 1$
\item $\forall v \in V: \norm v_V = \norm{\alpha(v)}_W$
\item $( e_1, \dots, e_n ) \subseteq V \text{ ONS } \implies
( \alpha(e_1), \dots, \alpha(e_n) ) \subseteq W \text{ ONS.}$
\end{enumerate}
\end{satz}
\begin{proof}
\leavevmode
\begin{itemize}
\item[a) $\implies$ b):] \checkmark
\item[b) $\implies$ c):] Es gilt für $v \in V\setminus \{0\}:$
$\norm{\frac{v}{\norm v}} = 1 \implies \norm{\frac{\alpha(v)}{\norm v}} = 1$\\
$\implies \norm{\alpha(v)} = \norm v$
\item[c) $\implies$ d):] $\inner vw = \frac 14(\norm{v+w}^2 - \norm{v-w}^2 +i\norm{v+iw}^2 - i\norm{v-iw}^2)$
\item[d) $\implies$ a):] Sei $v, w \in V$.
\begin{enumerate}[label=\arabic*. Fall:]
\item $v = 0 \implies \inner{\alpha(v)}{\alpha(w)} = \inner{0}{\alpha(w)} = 0 \checkmark$
\item $w = \lambda v \implies \inner{\alpha(w)}{\alpha(v)} = \lambda \inner{\alpha(v)}{\alpha(v)}
= \lambda \norm{\alpha(v)}^2$. \\
Sei $l := \frac{v}{\norm v} \overset{\text{d)}}{\implies} \alpha(l)$ ist ONS
$\implies \norm{\alpha(l)} = 1 \implies \norm{\alpha(v)} = \norm v$. \\
Es folgt $\inner{\alpha(v)}{\alpha(w)} = \inner vw \checkmark$.
\item $v, w$ linear unabhängig. Sei $(e_1, e_2)$ ONS mit $\langle\{e_1, e_2\}\rangle
= \langle\{ v, w \}\rangle$.
(Gram-Schmidt liefert Existenz)
\begin{align}
\implies & (\alpha(e_1), \alpha(e_2)) \text{ ist ONS} \nonumber \\ \nonumber
& v = \mu_1 e_1 + \mu_2 e_2 \\ \nonumber
& w = \tau_1 e_1 + \tau_2 e_2 \\ \nonumber
\implies & \alpha(v) = \mu_1 \alpha(e_1) + \mu_2 \alpha(e_2) \\
& \alpha(w) = \tau_1 \alpha(e_1) + \tau_2 \alpha(e_2) \label{eq:3.3.2.1}
\end{align}
\[
\underset{\text{\ref{theo:3.1.15}}}{\implies}
\inner vw = \mu_1 \overline{\tau_1} + \mu_2 \overline{\tau_2}
\underset{\mathclap{\substack{| \\ (\alpha(e_1), \alpha(e_2)) \text{ ONS \& \ref{eq:3.3.2.1}}}}}{=}
\inner{\alpha(v)}{\alpha(w)}
\]
\end{enumerate}
\end{itemize}
Beweis für $\inner vw = \frac 14(\norm{v+w}^2 - \norm{v-w}^2 +i\norm{v+iw}^2 - i\norm{v-iw}^2)$:
\begin{align*}
& \frac 14(\norm{v+w}^2 - \norm{v-w}^2 +i\norm{v+iw}^2 - i\norm{v-iw}^2) \\
& \begin{multlined}= \frac 14 (\inner{v+w}{v+w} -
\inner{v-w}{v-w}) + \frac i4 (\inner{v+iw}{v+iw} \\- \inner{v-iw}{v-iw})\end{multlined} \\
& \begin{multlined}= \frac 14 (\cancel{\inner vv} + \inner vw + \inner wv + \cancel{\inner ww}
- \cancel{\inner vv} + \inner vw + \inner wv - \cancel{\inner ww}) \\
+ \frac i4 (\cancel{\inner vv} + \inner v{iw} + \inner {iw}v + \cancel{\inner {iw}{iw}}\\
- \cancel{\inner vv} - \inner v{-iw} - \inner {-iw}v - \cancel{\inner {-iw}{-iw}})
\end{multlined} \\
& = \frac 14 (2 \inner vw + 2 \inner wv) +
\frac i4 (-i \inner vw + i \inner wv -i \inner vw + i \inner wv) \\
& = \frac 14 (2 \inner vw + 2 \inner wv) + \frac 14 (\inner vw - \inner wv + \inner vw - \inner wv) \\
& = \frac 14 (2 \inner vw + 2 \inner wv + 2 \inner vw - 2 \inner wv) \\
& = \frac 14 4 \inner vw = \inner vw
\end{align*}
\end{proof}
\begin{korollar}
\label{theo:3.3.3}
\leavevmode
\begin{enumerate}[label=\alph*)]
\item $\alpha$ orthogonal $\implies \alpha_\C$ unitär
\item $\alpha$ orthogonal/unitär $\implies \alpha$ injektiv.
\end{enumerate}
\end{korollar}
\begin{proof}
\begin{enumerate}[label=\alph*)]
\item Folgt direkt aus Satz~\ref{theo:3.3.2}:
\begin{align*}
\norm{\underset{\substack{\rotatebox{90}{=} \\u+iv}}{v_\C}}
= 1 & \iff \norm u^2 + \norm v^2 = 1 \\
& \implies \norm{\alpha_\C(v_\C)} = \norm{\alpha(u)}^2 + \norm{\alpha(v)}^2 = 1
\end{align*}
\item $\alpha(v) = 0 \implies \norm{\alpha(v)} = 0 \implies \norm v = 0 \implies v = 0$
\end{enumerate}
\end{proof}
\begin{defin}
\leavevmode
\begin{itemize}
\item $A \in \R^{\nxn}$ heißt \underline{orthogonal} wenn $A^{-1} = A^T$.
\item $A \in \C^{\nxn}$ heißt \underline{unitär} wenn $A^{-1} = A^* = \overline{A}^T$.
\item $O(n, \R) := \{ A \in \R^{\nxn}: \det(A)\neq 0 \land A^{-1} = A^T \}$
\item $U(n, \C) := \{ A \in \C^{\nxn}: \det(A)\neq 0 \land A^{-1} = A^* \}$
\end{itemize}
\end{defin}
\subsubsection{Beispiele}
\[
\begin{pmatrix} 0 & 1 & 0 \\ 1 & 0 & 0 \\ 0 & 0 & 1\end{pmatrix}, \;
\begin{pmatrix} \sin\varphi & \cos \varphi \\ -\cos\varphi & \sin \varphi \end{pmatrix}, \;
\frac 13 \begin{pmatrix} 2 & 1 & 2 \\ -2 & 2 & 1 \\ 1 & 2 & -2\end{pmatrix}\text{orthogonal}
\]
\[
\frac{1}{\sqrt{2}}\begin{pmatrix} 1 & i \\ -i & -1 \end{pmatrix}
\text{unitär}
\]
\begin{satz}
Es sind äquivalent für $A \in \K^{\nxn}, \K \in \{\R, \C\}$:
\begin{enumerate}[label=\alph*)]
\item $A$ ist orthogonal/unitär.
\item $(a_{1\_}, a_{2\_}, \dots, a_{n\_})$ bilden ONS in $\K^n$.
\item $(a_{\_1}, a_{\_2}, \dots, a_{\_n})$ bilden ONS in $\K^n$.
\end{enumerate}
\end{satz}
\begin{proof}
\leavevmode
\begin{itemize}
\item[a) $\iff$ b):] heißt, dass $\inner{a_{i\_}}{a_{j\_}}_{\K^n} = \delta_{ij}$.\\
Gleichzeitig gilt $(\inner{a_{i\_}}{a_{j\_}}_{\K^n})_{i,j=1,\dots,n} = A A^*$ \\
Also ist b) gleichbedeutend mit $A A^* = I$, also $A^{-1} = A^*$.
\item[a) $\iff$ c):] genauso, nur mit $A^* A$
\end{itemize}
\end{proof}
\begin{satz}
Sei $V$ ein euklidischer/unitärer Vektorraum mit $\dim(V)<\infty$ und $\alpha\in\Hom(V, V)$. Dann gilt
\[
\alpha\text{ ist orthogonal/unitär} \iff \alpha^{-1}=\alpha^*
\]
\end{satz}
\begin{proof}
\begin{itemize}
\item[$\implies$:] Seien $v,w \in V$.
$\alpha^{-1}$ existiert wegen Korollar~\ref{theo:3.3.3}b)
\begin{align*}
\inner{v}{\alpha^*(w) - \alpha^{-1}} & = \inner{v}{\alpha^*(w)} - \inner{v}{\alpha^{-1}(w)} \\
& = \inner{\alpha(v)}{w} - \inner{v}{\alpha^{-1}(w)} \\
& = \inner{\alpha(v)}{w} - \inner{\alpha(v)}{\alpha(\alpha^{-1}(w))} \\
& = \inner{\alpha(v)}{w} - \inner{\alpha(v)}{w} = 0
\end{align*}
\item[$\impliedby$:] Sei $\alpha^* = \alpha^{-1}, u,v,w\in V, v = \alpha(w)$
\[
\implies \inner uw = \inner{u}{\alpha^{-1}(v)} = \inner{u}{\alpha^*(v)} = \inner{\alpha(u)}{v}
=\inner{\alpha(u)}{\alpha(w)} \checkmark
\]
\end{itemize}
\end{proof}
\begin{satz}
Sei $B$ Orthonormalbasis, $\alpha \in \Hom(V, V), A= {}_B M(\alpha)_B$. Dann gilt:
\begin{enumerate}[label=\alph*)]
\item $\alpha$ orthogonal $\iff A$ orthogonal.
\item $\alpha$ unitär $\iff A$ unitär.
\end{enumerate}
\end{satz}
\begin{proof}
Satz \ref{theo:3.2.4}: ${}_B M(\alpha^*)_B = A^*$ \\
${}_B M(\alpha^{-1})_B = A^{-1}$
\end{proof}
\begin{satz}
\leavevmode
\begin{enumerate}[label=\alph*)]
\item $O(n, \R)$ ist Untergruppe von $\GL(n, \R)$.
\item $U(n, \C)$ ist Untergruppe von $\GL(n, \C)$.
\item $A \in O(n, \R) \implies \det(A) \in \{1, -1\}$.
\item $A \in U(n, \C) \implies \lvert \det(A) \rvert = 1
\implies \det(A) = e^{i\alpha}, \alpha \in [ 0, 2\pi ]$.
\end{enumerate}
\end{satz}
\begin{proof}
\begin{enumerate}[label=\alph*)]
\item $I \in O(n, \R), A,B \in O(n, \R)$ \\
$(AB)^* = B^*A^* = B^{-1}A^{-1} = (AB)^{-1} \implies AB \in =(n, \R)$ \\
$(A^{-1})^{{}^*} = (A^*)^{{}^*} = A \implies A^{-1} \in O(n, \R)$
\item Genauso
\item $A^{-1} = A^T$.
\begin{align*}
1 & = \det(A A^{-1}) = \det(A)\det(A^{-1}) = \det(A) \det(A^T) = \det(A) \det(A) \\
& = \det(A)^2 \implies \det(A) \in \{-1, 1\}
\end{align*}
\item $\det(\overline A) = \overline{\det(A)} \implies \det(A^*) = \overline{\det(A)}$.
\begin{align*}
& 1 = \det(A) \det(A^{-1})=\det(A) \det(A^*) = \det(A) \overline{\det(A)} \\
& \implies \lvert \det(A) \rvert=1
\end{align*}
\end{enumerate}
\end{proof}
\subsubsection{Polarzerlegung}
$z = \overbrace{e^{i\varphi}}^{\text{Betrag 1}} \underbrace{\lvert z \rvert}_{\mathclap{\text{positiv reell}}}$\\
Betrag 1 $\cong$ unitär, positiv $\cong$ selbstadjungiert mit positiven Eigenwerten.
\begin{satz}
[Polarzerlegung]
\label{theo:3.3.9}
Sei $V$ ein euklidischer/unitärer Vektorraum, $\dim(V)<\infty$ und sei $\alpha \in \Hom(V,V)$. Dann existiert
eine orthogonale/unitäre Abbildung $\beta$ und eine selbstadjungierte Abbildung $\gamma$ mit lauter
nichtnegativen reellen Eigenwerten, sodass $\alpha = \beta \circ \gamma$. Falls $\alpha$ Automorphismus ist,
so sind alle Eigenwerte von $\gamma$ positiv, und $\gamma, \beta$ eindeutig bestimmt.
\end{satz}
\begin{proof}
Zunächst: $\alpha$ Automorphismus.
\begin{itemize}
\item $\alpha^*$ ist auch Automorphismus.
\begin{align*}
\text{Sei }\alpha^*(v) = 0 & \implies \forall w \in V:
\inner{w}{\alpha^*(v)} = 0 \\
& \implies \forall w \in V: \inner{\alpha(V)}{v} = 0 \\
& \implies v \in \underbrace{\im(\alpha)}_v{}^\bot = \{0\} \\
& \implies v = 0
\end{align*}
\item $\alpha^*\circ \alpha$ ist Automorphismus und selbstadjungiert.
\[
(\alpha^*\alpha)^{{}^*} = \alpha^*(\alpha^*)^{{}^*} = \alpha^* \alpha
\]
\item Satz \ref{theo:3.2.11} ist $\spec(\alpha^*\alpha) \subseteq \R\setminus\{0\}$.
$\exists$ ONB $(e_1, \dots, e_n)$ von V aus Eigenvektoren, $(\lambda_1, \dots, \lambda_n) \subseteq
\R\setminus\{0\}$ Eigenwerte.
Behauptung: $\lambda_i > 0 \forall i \in [n]$
\[
\lambda_i = \lambda_i \inner{e_i}{e_i} = \inner{\lambda_ie_i}{e_i} =
\inner{\alpha^*\alpha(e_i)}{e_i} = \inner{\alpha(e_i)}{\alpha(e_i)} > 0
\]
\item $\gamma: \begin{cases} v & \to V \\
v = \sum_{i=1}^n \mu_i e_i & \mapsto \sum_{i=1}^n \mu_i \sqrt{\lambda_i} e_i\end{cases}$
${}_B M(\alpha^* \alpha)_B = \begin{pmatrix} \lambda_1 \\ & \ddots \\ & & \lambda_n\end{pmatrix}$
${}_B M(\gamma)_B = \begin{pmatrix} \sqrt{\lambda_1} \\ & \ddots \\ & & \sqrt{\lambda_n}\end{pmatrix}$
$\implies \gamma$ selbstadjungiert \\
$\implies \gamma^2 = \alpha^* \alpha$
\item $\alpha = \beta \circ \gamma \implies \beta = \alpha \circ \gamma^{-1}$ Behauptung: $\beta$ unitär, das
heißt $\beta^{-1} = \beta^*$
\begin{align*}
\beta^{-1} & = (\alpha \gamma^{-1})^{{}^{-1}} = \gamma \alpha^{-1} = \gamma^{-1} \gamma^2 \alpha^{-1}
= \gamma^{-1} \alpha^* \alpha \alpha^{-1} = \gamma^{-1} \alpha^* \\
& = (\alpha(\gamma^{-1})^*)^{{}^*} = (\alpha \gamma^{-1})^{{}^*} = \beta^*
\end{align*}
$\implies \beta$ unitär.
\end{itemize}
\underline{Eindeutigkeit:} $\alpha = \beta' \circ \gamma'$
\[
\gamma^2 = \alpha^* \alpha = (\gamma')^* \underbrace{\beta'^* \beta'}_{\id}\gamma' = (\gamma')^* \gamma'
= (\gamma')^2
\]
$\implies \gamma, \gamma'$ haben dieselben Eigenwerte und Eigenvektoren. \\
$\implies \gamma = \gamma' \implies \beta = \beta'$ \\
\underline{$\alpha$ nicht injektiv}:
\begin{itemize}
\item $W := \ker(\alpha)^\bot \implies \alpha|_W$ ist injektiv. \\
Sei $v, w \in W, \alpha(v) = \alpha(w) \implies \alpha(v - w) = 0 \implies v - w \in \ker(\alpha)
= W^\bot \cap W = \{0\} \implies v = w$
$\implies \alpha|_W = \beta_W \circ \gamma_W$ mit $\beta, \gamma \in \Hom(W, W); \beta_W$ unitär,
$\gamma_W$ selbstadjungiert mit positiven Eigenwerten.
\item Sei $( e_1, \dots, e_k )$ ONB von $W$, $(e_1, \dots, e_k, \dots, e_n)$ ONB von $V$.
\item $\pi:\begin{cases} V & \to W \\
v = \sum_{i=1}^n \lambda_i e_i & \to \sum_{i=1}^k \lambda_i e_i\end{cases}$
orthogonale Projektion auf $W$. \\
$\pi$ ist selbstadjungiert:
\begin{align*}
& \inner{\pi(v)}w = \inner{\sum_{i=1}^k \lambda_i e_i}{\sum_{j=1}^n \mu_j e_j} = \sum_{i=1}^k \lambda_i
\overline{\mu_i} \\
& \inner{v}{\pi(w)} = \inner{\sum_{i=1}^n \lambda_i e_i}{\sum_{j=1}^k \mu_j e_j} = \sum_{i=1}^k \lambda_i
\overline{\mu_i}
\end{align*}
$\gamma := \pi^* \circ \gamma_W \circ \pi = \pi \circ \gamma_W \circ \pi$
\begin{align*}
& v\in W & & \implies \gamma(v) = \gamma_W(v) \\ % Hier fehlt noch was aus der VO, nachschauen
& v \in W^\bot = \ker(\alpha) & & \implies \gamma(v) = 0
\end{align*}
$\beta := \underset{W}{\beta_W} \oplus \underset{W^\bot}{I}$ ist orthogonal/unitär. \\
$\implies \alpha = \beta \circ \gamma$
\end{itemize}
\end{proof}
\begin{defin}
$\alpha \in \Hom(V, V)$ heißt
$\genfrac{}{}{0pt}{0}{\text{\underline{positiv definit}}}{\text{\underline{positiv semi-definit}}}$, wenn
$\forall v \in V\setminus \{0\}: \inner{\alpha(v)}{v}\genfrac{}{}{0pt}{0}{>}{\ge}0$
\end{defin}
\begin{lemma}
Sei $\alpha$ selbstadjungiert. Dann gilt
\begin{align*}
\alpha \text{ positiv definit} & \iff \text{ Alle Eigenwerte positiv} \\
\alpha \text{ positiv semi-definit} & \iff \text{ Alle Eigenwerte nicht-negativ}
\end{align*}
\end{lemma}
\begin{proof}
Sei $(e_1, \dots, e_n)$ ONB aus Eigenvektoren,
$(\lambda_1, \dots, \lambda_n), \lambda_i \in \R$ Eigenwerte, $v = \sum_{i=1}^n
\mu_i e_i$
\begin{align*}
\inner{\alpha(v)}v = \inner{\sum_{i=1}^n \lambda_i \mu_i e_i}{\sum_{j=1}^n \mu_j e_j} =
\sum_{i=1}^n \lambda_i \mu_i \overline{\mu_i} = \sum_{i=1}^n \lambda_i \underbrace{\lvert \mu_i\rvert}_{\ge 0}{}^2
\end{align*}
Angenommen $\exists i \in [n]:\lambda_j \le 0 \implies \inner{\alpha(\lambda_j)}{e_j} = \lambda_j \le 0
\implies \alpha$ nicht positiv definit. \\
Angenommen $\forall i \in [n]: \lambda_i > 0 \implies \inner{\alpha(v)}v = \sum_{i=1}^n \lambda_i
\lvert \mu_i \rvert^2 > 0$.
\end{proof}
\subsubsection{Bemerkung}
In der Polarzerlegung ist $\beta$ orthogonal/unitär und $\gamma$ selbstadjungiert \& positiv (semi-)definit.
\section[Hauptachsentheorem für symmetrische/hermitesche Matrizen]{Hauptachsentheorem für \\symmetrische/hermitesche Matrizen}
\subsubsection{Ziel}
Klassifizierung aller Skalarprodukte.
\begin{defin}
\begin{align*}
A \in & \R^{\nxn} \text{heißt} & & \text{\underline{symmetrisch}, wenn} & & A = A^T \\
A \in & \C^{\nxn} \text{heißt} & & \text{\underline{hermitesch}, wenn} & & A = A^* \\
A \in & \R^{\nxn} \text{heißt} & & \text{\underline{schiefsymmetrisch}, wenn} & & A = -A^T \\
A \in & \C^{\nxn} \text{heißt} & & \text{\underline{schiefhermitesch}, wenn} & & A = A^*
\end{align*}
\end{defin}
\begin{satz}
$V$ euklidischer/unitärer Vektorraum, $\dim(V)< \infty$. Dann gilt:
\begin{itemize}
\item $\alpha$ selbstadjungiert $\iff \exists$ ONB $B$ mit ${}_B M(\alpha)_B$ \\
symmetrisch/hermitesch.
\item $\alpha$ anti-selbstadjungiert $\iff \exists$ ONB $B$ mit ${}_B M(\alpha)_B$ \\
schiefsymmetrisch/schiefhermitesch.
\end{itemize}
\end{satz}
\begin{satz}
\label{theo:3.4.3}
$A$ symmetrisch/hermitesch. \\
$\implies \exists$ orthogonale/unitäre Matrix $P$ mit $D = P^{-1} A P$ reelle Diagonalmatrix
\end{satz}
\begin{proof}
Sei $E = \{e_1, \dots, e_n\} \subseteq \K^n$ kanonische Basis, \\
$\varphi_A : \begin{cases} \K^n & \to \K^n \\v & \mapsto Av\end{cases}
\implies {}_E M(\varphi_A)_E = A \implies {}_E M(\varphi_A^*)_E = A^* = A$ \\
$\implies \varphi_A$ selbstadjungiert.
$\implies \exists$ ONB $B=(b_1, \dots, b_n)$ von $\K^n$ aus ~Eigenvektoren von $\varphi_A$, Eigenwerte sind
reell.
$\implies {}_B M(\varphi_A)_B = D = \diag(\lambda_1, \dots, \lambda_n)$ \\
$\implies {}_B M(\varphi_A)_B = \underbrace{{}_B M(\id)_E}_{P^{-1}}
\underbrace{{}_E M(\varphi_A)_E}_{A} \underbrace{{}_E M(\id)_B}_{\underbrace{(b_1, \dots, b_n)}_{P}}
\implies D = P^{-1} A P$
\end{proof}
\begin{korollar}
$A$ symmetrisch/hermitesch $\implies$ Eigenvektoren zu verschiedenen Eigenwerten sind orthogonal.
\end{korollar}
\subsubsection{Berechnung der Hauptachsentransformation}
$A \in \K^{\nxn}$
\begin{enumerate}[label=\arabic*)]
\item $\chi_A(\lambda) = \prod_{j=1}^r (\lambda_j - \lambda)^{d_j} $ ($\sum d_j = n$; algebraische Vielfachheit = geometrische Vielfachheit)
\item Für jedes $j \in [r]:$ Berechne Basis $B_j = (b_1^j, \dots, b_{d_j}^j)$ von $\ker(A - \lambda_j I)$
\item Orthogonalisiere $B_j$ zu ONS $E_j = (e_1^j, \dots, e_{d_j}^j)$ mittels Gram-Schmidt Verfahren.
\item $B = \bigcup_{j=1}^r E_j$ ist die gesuchte Orthonormalbasis.\\
Insbesondere $B^{-1} A B = \begin{pmatrix} \lambda_1 \\ & \ddots \\ & & \lambda_n\end{pmatrix}$ \\
$B^{-1} = \overline B^T$
\end{enumerate}
Polarzerlegung $A \in \K^{\nxn}, A^* A$ symmetrisch/hermitesch
\[
\implies A^* A = P^* \begin{pmatrix} \lambda_1 \\ & \ddots \\ & & \lambda_n \end{pmatrix} P
\]
\[
(A^* A)^{\frac 12} = P^* \begin{pmatrix} \sqrt{\lambda_1} \\ & \ddots \\ & & \sqrt{\lambda_n} \end{pmatrix} P
\underset{\mathclap{O=AS^{-1}}}{=} S \implies A = \underset{\mathclap{\substack{| \\ \text{orthogonal/unitär}}}}{O} S
\]
\begin{satz}
Sei $V$ ein reeller/komplexer Vektorraum mit $\dim(V) = n, B=(b_1, \dots, b_n)$ Basis. Für $A \in K^{\nxn}$ ist
\begin{equation}
\label{eq:3.4.5.1}
\inner vw := {}_B \Phi(v)^T A {}_B \overline{\Phi(w)}
\end{equation}
Genau dann ein Skalarprodukt, wenn $A$ symmetrisch/hermitesch und\\
$\underbrace{\text{positiv definit}}_{\substack{\forall \lambda \in \spec(A): \lambda > 0 \\
\rotatebox{90}{\tiny$\iff$} \\ \forall v \in V\setminus\{0\}: \inner{Av}{v} > 0}}$ ist. \\
Umgekehrt: Sei $\inner ..$ Skalarprodukt und $B$ Basis
\[
\implies \inner vw = {}_B \Phi(v)^T \underbrace{(\inner{b_i}{b_j})_{i,j=1}^n}_A {}_B \overline{\Phi(w)}
\]
($A = P^* \left( \begin{smallmatrix} \lambda_1 \\ & \ddots \\ & & \lambda_n\end{smallmatrix} \right)
P \implies \inner vw = {}_B \Phi(v)^T P^*
\left( \begin{smallmatrix} \lambda_1 \\ & \ddots \\ & & \lambda_n\end{smallmatrix} \right)
P {}_B \overline{\Phi(w)}$
$\implies$ in geeigneter Basis ist jedes Skalarprodukt durch $\sum_{i=1}^n \mu_i \overline \lambda_i$ gegeben.)
\end{satz}
\begin{proof}
Nur für $\K=\C$. Angenommen \ref{eq:3.4.5.1} ist Skalarprodukt.
\begin{align*}
\implies & \underset{\rotatebox{90}{$=$}}{\inner{b_i}{b_j}} = {}_B \Phi(b_i)^T A {}_B \overline{\Phi(b_j)}
= e_i^T A e_j = a_{ij} \\
& \overline{\inner{b_j}{b_i}} = \overline{a_{ji}} \\
\implies & a_{ij} = \overline{a_{ji}} \implies A \text{ hermitesch}
\end{align*}
Weiters muss $A$ positiv definit sein: Angenommen $\exists x \in \C^n \setminus \{0\}: x^T Ax = 0
\implies v := \sum x_i b_i$, das heißt ${}_B \Phi(v) = x$ erfüllt $\inner vv = {}_B \Phi(v)^T A {}_B
\overline{\Phi(v)} = x^T A \overline x = 0$ \\
Sei $A$ hermitesch \& positiv definit. Klarerweise gilt dann für \\
$\inner uv := {}_B \Phi(u)^T A {}_B \overline{\Phi(v)}$:
\begin{align*}
\inner{u+v}{w} & = \inner uw + \inner vw \\
\inner uv & = \overline{\inner vu} \\
\inner{\lambda u}{v} & = \lambda \inner uv
\end{align*}
Bleibt zu zeigen, dass $\forall v \in V\setminus\{0\}: \inner vv > 0$ \\
Satz \ref{theo:3.4.3} $\implies \exists$ unitäre Matrix $U$ mit
$A = \underset{\substack{\rotatebox{90}{$=$}\\U^{-1}}}{U^*}
\underbrace{\diag(\lambda_1, \dots, \lambda_n)}_\Sigma U, \lambda_i \in (0, \infty)\forall i \in [n]$
\begin{align*}
& \inner vv={}_B \Phi(v)^T A {}_B \overline{\Phi(v)} = {}_B \Phi(v)^T U^* \Sigma U {}_B \overline{\Phi(v)}
= \underset{\substack{\rotatebox{90}{$=$} \\\sum \lambda_i \lvert x_i \rvert^2 > 0}}
{(\overline U {}_B\Phi(v))^T \Sigma \overline{\overline U {}_B (v)}} \\
& v \neq0 \implies \overline U {}_B \Phi(v) =
\left( \begin{smallmatrix} x_1 \\ \vdots \\ x_n \end{smallmatrix} \right) \neq 0
\end{align*}
\end{proof}
\begin{defin}
Sei $A \in \K^{\nxn}$ eine symmetrische/hermitesche Matrix.
\begin{itemize}
\item \[
t(A) := \lvert \{\lambda \in \spec(A): \lambda > 0 \} \rvert
\]
heißt \underline{Trägheitsindex} von $A$.
\item $A, B$ heißen \underline{kongruent} wenn eine invertierbare Matrix $Q \in \K^{\nxn}$ existiert mit
$B = Q^* A Q$
\end{itemize}
\end{defin}
\subsubsection{Bemerkung}
$M_B(\sigma), M_{B'}(\sigma)$ sind kongruent (\& umgekehrt)
\begin{satz}
[Trägheitssatz von Sylvester]
Sei $A \in \K^{\nxn}$ symmetrisch/hermitesch mit $\rg(A) = r, t(A)=t$. Dann gilt:
\begin{enumerate}[label=\alph*)]
\item Es gibt $S \in \K^{\nxn}$ invertierbar mit
\[
S^* A S =
\diag(\underbrace{1, \dots, 1}_t, \underbrace{-1, \dots, -1}_{r-t}, \underbrace{0, \dots, 0}_{n-r})
\]
$A$ ist zu dieser Matrix kongruent.
\item $A, B$ kongruent $\iff t(A) = t(B) \land \rg(A) = \rg(B)$
\end{enumerate}
\subsubsection{Bemerkung}
Trägheitsindex und Rang charakterisieren symmetrische Bilinearformen / hermitesche Sesquilinearformen komplett.
\end{satz}
\begin{proof}
Nur $\K = \C$
\begin{enumerate}[label=\alph*)]
\item Satz \ref{theo:3.4.3} $\implies \exists P$ unitär mit $P^* AP = \diag(\lambda_1, \dots, \lambda_n)$
o.B.d.A. $\lambda_1, \dots, \lambda_t > 0, \lambda_{t+1}, \dots, \lambda_{r} < 0, \lambda_{r+1}, \dots,
\lambda_n = 0$. \\
Setze $T = \begin{pmatrix}
\frac{1}{\sqrt{\lvert \lambda_1 \rvert}} \\
& \ddots \\
& & \frac{1}{\sqrt{\lvert \lambda_r \rvert}} \\
& & & 1 \\
& & & & \ddots \\
& & & & & 1
\end{pmatrix}$
$\implies S := PT$ ist invertierbar.
\begin{align*}
S^* A S & = T \underbrace{P^* A P}_{\mathclap{\diag{\lambda_1, \dots, \lambda_n}}} T = T
\begin{pmatrix}
\lambda_1 \\
& \ddots \\
& & \lambda_n
\end{pmatrix} T \\
& = \begin{pmatrix}
1 \\
& \ddots \\
& & 1 \\
& & & -1 \\
& & & & \ddots \\
& & & & & -1 \\
& & & & & & 0 \\
& & & & & & & \ddots \\
& & & & & & & & 0
\end{pmatrix}
\end{align*}
\item $A, B$ kongruent, das heißt $\exists Q$ invertierbar mit $B = Q^* A Q$\\
$ \implies \rg(B) = \rg(A)$.
Satz \ref{theo:3.4.3} $\implies \exists P_1, P_2$ unitär mit
\begin{align*}
P_1^* A P_1 & = \begin{pmatrix}
\lambda_1 \\
& \ddots \\
& & \lambda_n
\end{pmatrix} = D \\
P_2^* A P_2 & = \begin{pmatrix}
\mu_1 \\
& \ddots \\
& & \mu_n
\end{pmatrix} = G
\end{align*}
$t(A) := t, t(B) := s$ \\
Ordne so, dass $\ontop{\lambda_1, \dots, \lambda_t > 0,
\lambda_{t+1}, \dots, \lambda_r < 0,
\lambda_{r+1}, \dots, \lambda_n = 0}
{\mu_1, \dots, \mu_s > 0, \mu_{s+1}, \dots, \mu_r < 0, \mu_{r+1}, \dots, \mu_n =0}$
Sethe $a_i := \sqrt{\lvert \lambda_i \rvert}, b_i := \sqrt{\lvert \mu_i \rvert}$
\begin{equation}
\label{eq:3.4.6.1}
x^* D x = \sum_{j=1}^t a_j^2 \lvert x_j \rvert^2 - \sum_{j=t+1}^r a_j^2 \lvert x_j \lvert^2
\end{equation}
$C := P_2 Q^{-1} P_1, y := Cx$
\begin{equation}
\label{eq:3.4.6.2}
x^* D x = x^* C^* G C x = y^* G y = \sum_{j=1}^s b_j^2 \lvert y_j \rvert^2 - \sum_{j=s+1}^r b_j^2
\lvert y_j \rvert^2
\end{equation}
Angenommen $t<s$:
\begin{equation}
\label{eq:3.4.6.3}
\begin{rcases} x_i = 0 & i = 1, \dots, t \\
y_i = \sum_{j=1}^n C_{ij} x_j = 0 & i = s+1, \dots, n
\end{rcases}
\end{equation}
$\implies <n \text{ Gleichungen in }n$
Variablen $\implies \exists z \in \C^n \setminus \{0\}:$ \ref{eq:3.4.6.3} ist für $z$ erfüllt.
\begin{align*}
z^* D z & \overset{\text{\ref{eq:3.4.6.1}}}{=} - \sum_{j=t+1}^r a_j^2 \lvert z_j \rvert^2 < 0 \\
z^* D z & \overset{\text{\ref{eq:3.4.6.2}}}{=} \sum_{j=1}^s b_j^2 \abs{y_j}^2 > 0
& & \text{\Lightning}
\end{align*}
$\implies s = t$
\end{enumerate}
\end{proof}
\subsubsection{Berechnung des Trägheitsindex:}
Sei $A$ symmetrisch hermitesch, $\det(A) \neq 0 \implies \chi_A(\lambda) = a_1 \lambda^n + \dots +
\underset{\mathrlap{\rotatebox{325}{\scriptsize$\neq 0$}}}{a_0}$ mit $a_i \in \R$ ist Polynom mit lauter
reellen Nullstellen.
\begin{satz}
[Vorzeichenregel von Descartes]
Sei $p$ Polynom mit $p(0) \neq 0$, reellen Koeffizienten und lauter reellen Nullstellen. Dann gilt
\[
\abs{ \{ \lambda: p(\lambda) = 0 \land \lambda > 0 \} }
= \abs{ i \in \{0, \dots n-1 \}: a_i a_{i+1} < 0 }
\]
\end{satz}
\section{Bilinearformen und Sesquilinearformen}
\begin{defin}
\leavevmode
\begin{itemize}
\item $\sigma: V\times V \to \K$ mit $\forall u,v,w \in V, \lambda \in \K$:
\begin{align}
\sigma(v+w, u) & = \sigma(v, u) + \sigma(w, u) \nonumber \\
\sigma(v, w+u) & = \sigma(v, w) + \sigma(v, u) \nonumber \\
\sigma(\lambda v, w) & = \lambda \sigma(v, w) \nonumber \\
\sigma(v, \lambda w) & = \lambda \sigma(v, w) \label{eq:3.5.1.1}
\end{align}
heißt \underline{Bilinearform}.
\item Falls $\K = \C$ und anstelle von \ref{eq:3.5.1.1} gilt, dass
$\sigma(v, \lambda w) = \overline \lambda \sigma(v,w)$, so heißt $\sigma$ \underline{Sesquilinearform}.
\item Eine Bilinearform heißt \underline{symmetrisch}, wenn $\sigma(u, v) = \sigma(v, u)$ und \\
\underline{alternierend}, wenn $\sigma(u, v) = -\sigma(v, u)$.
\item Eine Sesquilinearform heißt \underline{hermitesch}, wenn $\sigma(u, v) = \overline{\sigma(v, u)}$
\end{itemize}
\end{defin}
\subsubsection{Beispiel}
\begin{itemize}
\item Euklidisches Skalarprodukt ist symmetrische Bilinearform.
\item Unitäres Skalarprodukt ist hermitesche Bilinearform.
\item \begin{equation} \label{eq:3.5.1.2}
\sigma(x, y) = x_1 y_1 + x_1 y_2 + x_2 y_1 - 5 x_2 y_2 = \begin{pmatrix} x_1 & x_2 \end{pmatrix}
\begin{pmatrix} 1 & 1 \\ 1 & -5 \end{pmatrix} \begin{pmatrix} y_1 \\ y_2 \end{pmatrix}
\end{equation}
$V=\R^2$
$ q(x) := \sigma(x, x) = x_1^2 + x_1 x_2 + x_2 x_1 - 5x_2^2, \R^2 \to \R$
\end{itemize}
Sei $ B= (b_1, \dots, b_n)$ Basis, so ist $M_B(\sigma) := (\sigma(b_i, b_j))_{i,j=1}^n$
\begin{lemma}
\label{theo:3.5.2}
\leavevmode
\begin{enumerate}[label=\alph*)]
\item Es gilt für $\sigma$ Bilinearform und $B$ Basis, dass
\[
\sigma(u, v) = {}_B \Phi(u)^T M_B(\sigma) {}_B \Phi(v) \; \forall u, v
\]
\item Es gilt für $\sigma$ Sequilinearform, dass
\[
\sigma(u, v) = {}_B \Phi(u)^T M_B(\sigma) {}_B \overline{\Phi(v)} \; \forall u, v
\]
\item Sei $B'$ eine weitere Basis und $\K = \R$
\[
M_{B'}(\sigma) = {{}_B M(\id)_{B'}}^T \, M_B(\sigma) \, {}_B M(\id)_{B'}
\]
\item Sei $B'$ eine weitere Basis und $\K = \C$
\[
M_{B'}(\sigma) = {{}_B M(\id)_{B'}}^T \, M_B(\sigma) \, {}_B \overline{M(\id)}_{B'}
\]
\item $\sigma$ symmetrisch/hermitesch $\iff M_B(\sigma)^* = M_B(\sigma)$
\end{enumerate}
\end{lemma}
\begin{proof}
\begin{enumerate}[label=\alph*)]
\item Analog wie b)
\item $u = \sum \lambda_i b_i, v=\sum \mu_j b_j, A = M_B(\sigma)$
\begin{align*}
\sigma(u, v) & = \sigma(\sum \lambda_i b_i, \sum \mu_j b_j) = \sum_{i=1}^n \lambda_i
\underbrace{\sum_{j=1}^n \underbrace{\sigma(b_i, b_j)}_{a_{ij}} \overline \mu_j}
_{A {}_B\overline{\Phi(v)}} \\
& = {}_B \Phi(u)^T M_B(\sigma) {}_B \overline{\Phi(v)}
\end{align*}
\item Analog wie d)
\item $b'_i = \sum_k a_{ki} b_k, M_{B'} = (\sigma(b'_i, b'_j))_{i,j}$
\begin{align*}
\sigma(b'_i, b'_j) & = \sigma(\sum_k a_{ki} b_k, \sum_l a_{lj} b_l) = \sum_k a_{ki} \sum_l
\underbrace{\sigma(b_i,b_j)}_{M_B(\sigma)} \overline{a_{lj}} \\
& = (A^T M_B(\sigma)\overline A)_{ij}
\end{align*}
\item \begin{align*}
\underset{\rotatebox{70}{$=$}}{\sigma(v, w)}
& = \underset{\rotatebox{110}{$=$}}{\overline{\sigma(w, v)}} \\
{}_B \Phi(v)^T M_B(\sigma) {}_B \overline{\Phi(w)}
& = \left({}_B\overline{\Phi(w)}^T \overline{M_B(\sigma)} {}_B \Phi(v)\right)^T \\
& = {}_B \Phi(v)^T \overline{M_B(\sigma)}^T {}_B \overline{\Phi(w)} \\
\implies M_B(\sigma) = M_B(\sigma)^*
\end{align*}
\end{enumerate}
\end{proof}
\begin{satz}
Sei $V$ euklidischer/unitärer Vektorraum und $\sigma$ symmetrische/hermitesche Bilinear-/Sesquilinearform.
Dann existiert eine Orthonormalbasis $B$ mit \\ $M_B(\sigma)$ reelle Diagonalmatrix.
\end{satz}
\begin{proof}
Nach Lemma \ref{theo:3.5.2} e) gilt $M_B(\sigma)^* = M_B(\sigma)$ und daher nach Satz \ref{theo:3.4.3}
gibt es orthogonale/unitäre Matrix $U$ mit
\[
U^* M_B(\sigma) U = \left(\begin{smallmatrix}
\lambda_1 \\ & \ddots \\ & & \lambda_n
\end{smallmatrix}\right), \lambda_i \in \R \forall i \in [n]
\]
Behauptung folgt dann aus Lemma \ref{theo:3.5.2} d).
\end{proof}
\subsubsection{Beispiel}
$\sigma$ wie in \ref{eq:3.5.1.2}, $A = \begin{pmatrix} 1 & 1 \\ 1 & -5 \end{pmatrix}$ \\
$\chi_a(\lambda) = \det\begin{pmatrix} 1-\lambda & 1 \\ 1 & -5-\lambda \end{pmatrix}
= (\lambda -1)(5 + \lambda) - 1$ \\
Nullstellen: $\lambda_1, \lambda_2 = -2 \pm \sqrt{10}$\\
$b_1 = \frac{1}{\sqrt{20+6\sqrt{10}}} \begin{pmatrix}
3 + \sqrt{10} \\ 1 \end{pmatrix},
b_2 = \frac{1}{\sqrt{20-6\sqrt{10}}} \begin{pmatrix}3 - \sqrt{10} \\ 1 \end{pmatrix}$ \\
$M_B(\sigma) = \begin{pmatrix} -2 + \sqrt{10} & 0 \\ 0 & -2 -\sqrt{10} \end{pmatrix}$ \\
$ \implies q(\tilde x_1, \tilde x_2) = \lambda_1 \tilde x_1^2 + \lambda_2 \tilde x_2^2 \equiv c$
\begin{tikzpicture}[scale=1.8]
\tikzmath{
\bfactor1 = 1 / sqrt(20 + 6* sqrt(10));
\bfactor2 = 1 / sqrt(20 - 6* sqrt(10));
\bx1 = \bfactor1 * (3 + sqrt(10));
\by1 = \bfactor1;
\bx2 = \bfactor2 * (3 - sqrt(10));
\by2 = \bfactor2;
\brichtung1 = \by1 / \bx1;
\brichtung2 = \by2 / \bx2;
}
\begin{axis}[
title=\scriptsize{Niveaulinien von $q(x)$},
xlabel={$x_1$},
ylabel={$x_2$},
ymin=-2.7,ymax=2.7,
xmin=-3.3,xmax=3.3,
view={0}{90},
axis lines=middle,
tick label style={font=\tiny},
label style={font=\scriptsize},
]
\addplot3 [
contour gnuplot={
levels={0,-1,-4,-9,-16,-25,-36,1,4,9},
contour label style={every node/.append style={text=ForestGreen}},
label distance = 90pt,
},
samples=80,
domain=-3.3:3.3,
domain y=-2.7:2.7,
contour/draw color={ForestGreen},
]
{x^2 + 2*x*y - 5*y^2};
\draw [->, red, thick] (0,0) -- (\bx1, \by1) node[above]{\footnotesize$b_1$};
\draw [->, red, thick] (0,0) -- (\bx2, \by2) node[above right]{\footnotesize$b_2$};
\addplot [
domain=-3.3:3.3,
color=red,
style={dash pattern=on 3pt off 2pt on 15pt off 2pt},
]
{x * \brichtung1};
\addplot [
domain=-3.3:3.3,
color=red,
style={dash pattern=on 3pt off 2pt on 15pt off 2pt},
]
{x * \brichtung2};
\end{axis}
\end{tikzpicture}
\begin{defin}
Sei $\rho: V \to \K$ heißt \underline{quadratische Form} wenn $\forall u, v \in V, \lambda \in \K:$
\begin{enumerate}[label=\alph*)]
\item $\rho(\lambda v) = \lambda^2 \rho(v)$
\item $ \sigma(u, v) := \rho(u + v) - \rho(u) - \rho (v)$ ist eine (symmetrische) Bilinearform
\end{enumerate}
\end{defin}
\begin{lemma}
Sei $\operatorname{char}(\K) \neq 2$. Dann entsprechen die quadratischen Formen und symmetrischen
Bilinearformen eineindeutig.
\end{lemma}
\begin{proof}
$\rho$ quadratische Form $\implies \sigma(v, w) = \rho(u + v) - \rho(u) - \rho(v)$ ist symmetrische
Bilinearform. \\
Sei $\sigma$ symmetrische Bilinearform, $\rho(v) := \frac 12 \sigma(v, v)$.
\begin{align*}
\rho(\lambda v) = \frac 12 \sigma(\lambda v, \lambda v) & = \lambda^2 \frac 12 \sigma(v, v) =
\lambda^2 \rho(v) \implies \text{a)} \\
\rho(u+v) - \rho(u) - \rho(v) & = \frac 12(\sigma(u + v, u + v) - \sigma(u, u) - \sigma(v, v)) \\
& \begin{multlined}
= \frac 12(\cancel{\sigma(u, u)} + \sigma(u, v) + \sigma(v, u)
+ \cancel{\sigma(v, v)} \\
- \cancel{\sigma(u, u)} - \cancel{\sigma(v, v)})\end{multlined} \\
& = \sigma(u, v) \text{ ist symmetrische Bilinearform.}
\end{align*}
\end{proof}
\begin{defin}
$V \C$-VR. $\rho: V \to \R$ heißt \underline{hermitesche Form} wenn $\forall u, v \in V, \lambda \in \C$:
\begin{enumerate}[label=\alph*)]
\item $\rho(\lambda v) = \abs{\lambda}^2 \rho(v)$
\item $\rho(u+v) + \rho(u -v) = 2(\rho(u) + \rho(v))$
\item $\sigma(u,v) := \frac 12 (\rho(u+v) + i\rho(u +iv) - (1+i)(\rho(u) + \rho(v)))$ ist hermitesche
Sesquilinearform.
\end{enumerate}
\end{defin}
\begin{lemma}
Hermitesche Formen und hermitesche Sesquilinearformen entsprechen einander eineindeutig
\end{lemma}
\begin{proof}
\leavevmode
\begin{itemize}
\item $\rho$ hermitesche Form, $\sigma$ wie oben in c) $\implies \sigma$ hermitesche Sesquilinearform.
\item $\sigma$ hermitesche Sesquilinearform, $\rho(v) := \frac 12 \sigma(v, v) \overset{\text{\tl UE\br}}
\implies \rho$ ist hermitesche Form.
\end{itemize}
\end{proof}
\subsubsection{Bemerkung}
$\sigma$ heißt Polarform von $\rho$
\section[Die Singulärwertzerlegung und die Pseudoinverse]{Die Singulärwertzerlegung und die \\Pseudoinverse}
Wollen Normalform für $\alpha \in \homk(V, W)$ mit $V, W$ euklidisch/unitär herleiten.
Polarzerlegung: $W = V \implies \exists$ ONBs $B, B'$ von $V$ mit
\[
{}_B M(\alpha)_B = \begin{pmatrix}s_1 \\
& \ddots \\
& & s_r \\
& & & 0 \\
& & & & \ddots \\
& & & & & 0\end{pmatrix}, s_1, \dots, s_n > 0
\]
Das gilt für $V, W$ allgemein.
\begin{satz}
[Singulärwertzerlegung]
Sei $A \in \R^{m \times n} / \C^{m \times n}$. Dann gibt es orthogonale/unitäre Matrizen $U, V$ sowie
$s_1, \dots, s_r \in (0, \infty)$ mit
\[
A = \underbrace{U}_{\K^{m\times m}} \underbrace{\begin{pmatrix}s_1 \\
& \ddots \\
& & s_r \\
& & & 0 \\
& & & & \ddots \\
& & & & & 0\end{pmatrix}}_{\K^{m \times m}} \underbrace{V}_{\K^{m \times n}}
\]
$s_1, \dots, s_r$ heißen positive \underline{Singulärwerte} von $A$.
\end{satz}
\begin{proof}
\begin{itemize}
\item $A^* A \in \K^{\nxn}$ selbstadjungiert und positiv semi-definit.
Eigenwerte $\lambda_1, \dots, \lambda_n \in [0, \infty)$, ONB $b_1, \dots, b_n$ aus Eigenvektoren.
Sei $\lambda_1, \dots, \lambda_r \in (0, \infty), \lambda_{r+1} = \dots = \lambda_n = 0$
$s_i := \sqrt{\lambda_i}, i\in [n]$
\item $\overbrace{\frac 1{s_1} A b_1}^{b_1'}, \dots, \overbrace{\frac 1{s_r} A b_r}^{b_r'}$ ist
Orthonormalsystem in $\K^m$.
\[
\overline{\inner{Ab_i}{Ab_j}}_{\K^m} = \overline{b_i}^T A^* A b_j = \lambda_j \overline{b_i}^T b_j
= \lambda_j \overline{\inner{b_i}{b_j}}_{\K^n} = \lambda_j \delta_{ij} \in \R
\]
\item Ergänze $b_1', \dots, b_r'$ zu Orthonormalbasis $b_1', \dots, b_r', \dots, b_m'$ von $\K^m$. \\
Sei $\varphi_A: x \mapsto A\cdot x \implies {}_{B'} M(\varphi_A)_B = \left( \begin{smallmatrix}
s_1 \\
& \ddots \\
& & s_r \\
& & & 0 \\
& & & & \ddots \\
& & & & & 0
\end{smallmatrix} \right)$ \\
$v = \sum \mu_i b_i \implies Av = \sum \mu_i \underbrace{A b_i}_{s_i b_i'} = \sum \mu_i s_i b_i'$
\end{itemize}
\end{proof}
\end{document}